You are on page 1of 65

Great Pacific Life Assurance vs.

Court of Appeals; 316 SCRA 678 FACTS: Great Pacific Life Assurance Corporation (Grepalife) executed a contract of group life insurance with Development Bank of the Philippines (DBP) wherein Grepalife agreed to insure the lives of eligible housing loan mortgagors of DBP.

form of contract so that in the event of the unexpected demise of the mortgagor during the subsistence of the mortgage contract, the proceeds from such insurance will be applied to the payment of the mortgage debt, thereby relieving the heirs of the mortgagor from paying the obligation. In a similar vein, ample protection is given to the mortgagor under such a concept so that in the event of death, the mortgage obligation will be extinguished by the application of the insurance proceeds to the mortgage indebtedness. In this type of policy insurance, the mortgagee is simply an appointee of the insurance fund. Such loss-payable clause does not make the mortgagee a party to the contract.

One such loan mortgagor is Dr. Wilfredo Leuterio. In an application form, Dr. Leuterio answered questions concerning his test, attesting among others that he does not have any heart conditions and that he is in good health to the best of his knowledge.

However, after about a year, Dr. Leuterio died due to massive cerebral hemorrhage. When DBP submitted a death claim to Grepalife, the latter denied the claim, alleging that Dr. Leuterio did not disclose he had been suffering from hypertension, which caused his death. Allegedly, such non-disclosure constituted concealment that justified the denial of the claim.

The insured, being the person with whom the contract was made, is primarily the proper person to bring suit thereon. Subject to some exceptions, insured may thus sue, although the policy is taken wholly or in part for the benefit of another person, such as a mortgagee.

And since a policy of insurance upon life or health may pass by transfer, will or succession to any person, whether he has an insurable interest or not, and such person may recover it whatever the insured might have recovered, the widow of the decedent Dr. Leuterio may file the suit against the insurer, Grepalife. SUNLIFE ASSURANCE COMPANY OF CANADA vs. CA G.R. No. 105135 June 22, 1995 Element of concealment non disclosure of material fact that could mislead the insurerand affect in forming his estimates of the proposed insurance policy or in making inquiries. Facts: Robert John B. Bacani procured a life insurance contract for himself from Sunlife. He was issued a Policy valued atP100,000.00, with double indemnity in case of accidental death. The designated beneficiary was his mother, BernardaBacani.In his application for insurance Robert was asked if within 5 years he (a) consulted any doctor or other healthpractitioner (b) subjected to different test i.e. blood, x-rays etc. (c) attended or been admitted to any hospital or othermedical facility. Robert answered yes in letter a. but limited his answer to a consultation with a certain Dr. Reinaldo D.Raymundo of the Chinese General Hospital on February 1986, for cough and flu complications.Sunlife discovered that two weeks prior to

Hence, the widow of the late Dr. Leuterio filed a complaint against Grepalife for Specific Performance with Damages. Both the trial court and the Court of Appeals found in favor of the widow and ordered Grepalife to pay DBP.

ISSUE: Whether the CA erred in holding Grepalife liable to DBP as beneficiary in a group life insurance contract from a complaint filed by the widow of the decedent/mortgagor HELD: The rationale of a group of insurance policy of mortgagors, otherwise known as the mortgage redemption insurance, is a device for the protection of both the mortgagee and the mortgagor. On the part of the mortgagee, it has to enter into such

Roberts application for insurance, that Robert was examined and confinedat the Lung Center of the Philippines, where he was diagnosed for renal failure. During his confinement, the deceasedwas subjected to urinalysis, ultra-sonography and hematology tests.Robert died in a plane crash. Bernarda filed a claim with sunlife, seeking the benefits of the insurance policy taken byher son. Sunlife conducted an investigation and its findings prompted it to reject the claim.Sunlife informed Bernarda that Robert did not disclose material facts relevant to the issuance of the policy, thusrendering the contract of insurance voidable. A check representing the total premiums paid in the amount of P10,172.00 was attached to said letter.Bernarda subsequently filed an action for specific performance against Sunlife. Sunlife filed a counter claim and a listof exhibits consisting of medical records furnished by the Lung Center of the Philippines. Bernarda filed a "ProposedStipulation with Prayer for Summary Judgment" where they manifested that they "have no evidence to refute the documentary evidence of concealment/misrepresentation by the decedent of his health condition. Sunlife also filed amotion for summary judgement. Trial court ruled in favor of Bernarda and concluded that although there was concealment and misrepresentation thefacts concealed by the insured were made in good faith and under a belief that they need not be disclosed. Moreover,it held that the health history of the insured was immaterial since the insurance policy was "non-medical". Court of Appeals affirmed the decision and stated that the cause of death was unrelated to the facts concealed by the insured.Issue: WON there was concealment and can goodfaith be used as a defense.Ruling: Yes there was concealment and No the defense of goodfaith is not applicable.Rationale: Section 26 of The Insurance Code requires a party to a contract of insurance to communicate to the other, in goodfaith, all facts within his knowledge which are material to the contract and as to which he makes no warranty, and which the other has no means of ascertaining and thus it provides that A neglect to communicate that whicha party knows and ought to communicate, is called concealment.Materiality is to be determined not by the event, but solely by the probable and reasonable influence of the facts upon the party to whom communication is due, in forming his estimate of the disadvantages of the proposed contract or in making his inquiriesThe information which the insured failed to disclose were material and relevant to the approval andissuance of the insurance policy. The matters concealed would have definitely affected Sunlife's actionon Roberts application, either by approving it with the corresponding adjustment for a higher premiumor rejecting the same.

Moreover, a disclosure may have warranted a medical examination of the insured by Sunlifein order for it to reasonably assess the risk involved in accepting the application. It is well settled that the insured need not die of the disease he had failed to disclose to the insurer. It issufficient that his non-disclosure misled the insurer in forming his estimates of the risks of the proposedinsurance policy or in making inquiries. Thus, " good faith" is no defense in concealment . The insured's failure to disclose the fact that he was hospitalizedfor two weeks prior to filing his application for insurance, raises grave doubts about his bonafides . It appears that suchconcealment was deliberate on his part.

[G.R. No. 125678. March 18, 2002] PHILAMCARE HEALTH SYSTEMS, INC., petitioner, vs. CA Ernani Trinos, deceased husband of respondent Julita Trinos, applied for a health care coverage with petitioner Philamcare Health Systems, Inc. In the standard application form, he answered no to the following question: Have you or any of your family members ever consulted or been treated for high blood pressure, heart trouble, diabetes, cancer, liver disease, asthma or peptic ulcer? (If Yes, give details).[1] The application was approved for a period of one year from March 1, 1988 to March 1, 1989. Accordingly, he was issued Health Care Agreement No. P010194. Under the agreement, respondents husband was entitled to avail of hospitalization benefits, whether ordinary or emergency, listed therein. He was also entitled to avail of outpatient benefits such as annual physical examinations, preventive health care and other out-patient services. Upon the termination of the agreement, the same was extended for another year from March 1, 1989 to March 1, 1990, then from March 1, 1990 to June 1, 1990. The amount of coverage was increased to a maximum sum of P75,000.00 per disability.[2]

During the period of his coverage, Ernani suffered a heart attack and was confined at the Manila Medical Center (MMC) for one month beginning March 9, 1990. While her husband was in the hospital, respondent tried to claim the benefits under the health care agreement. However, petitioner denied her claim saying that the Health Care Agreement was void. According to petitioner, there was a concealment regarding Ernanis medical history. Doctors at the MMC allegedly discovered at the time of Ernanis confinement that he was hypertensive, diabetic and asthmatic, contrary to his answer in the application form. Thus, respondent paid the hospitalization expenses herself, amounting to about P76,000.00. After her husband was discharged from the MMC, he was attended by a physical therapist at home. Later, he was admitted at the Chinese General Hospital. Due to financial difficulties, however, respondent brought her husband home again. In the morning of April 13, 1990, Ernani had fever and was feeling very weak. Respondent was constrained to bring him back to the Chinese General Hospital where he died on the same day. On July 24, 1990, respondent instituted with the Regional Trial Court of Manila, Branch 44, an action for damages against petitioner and its president, Dr. Benito Reverente, which was docketed as Civil Case No. 90-53795. She asked for reimbursement of her expenses plus moral damages and attorneys fees. After trial, the lower court ruled against petitioners, viz: WHEREFORE, in view of the forgoing, the Court renders judgment in favor of the plaintiff Julita Trinos, ordering: 1. Defendants to pay and reimburse the medical and hospital coverage of the late Ernani Trinos in the amount of P76,000.00 plus interest, until the amount is fully paid to plaintiff who paid the same; 2. Defendants to pay the reduced amount of moral damages of P10,000.00 to plaintiff; 3. Defendants to pay the reduced amount of P10,000.00 as exemplary damages to plaintiff; 4. Defendants to pay attorneys fees of P20,000.00, plus costs of suit. SO ORDERED.[3]

On appeal, the Court of Appeals affirmed the decision of the trial court but deleted all awards for damages and absolved petitioner Reverente.[4] Petitioners motion for reconsideration was denied.[5] Hence, petitioner brought the instant petition for review, raising the primary argument that a health care agreement is not an insurance contract; hence the incontestability clause under the Insurance Code[6] does not apply. Petitioner argues that the agreement grants living benefits, such as medical checkups and hospitalization which a member may immediately enjoy so long as he is alive upon effectivity of the agreement until its expiration one-year thereafter. Petitioner also points out that only medical and hospitalization benefits are given under the agreement without any indemnification, unlike in an insurance contract where the insured is indemnified for his loss. Moreover, since Health Care Agreements are only for a period of one year, as compared to insurance contracts which last longer,[7] petitioner argues that the incontestability clause does not apply, as the same requires an effectivity period of at least two years. Petitioner further argues that it is not an insurance company, which is governed by the Insurance Commission, but a Health Maintenance Organization under the authority of the Department of Health. Section 2 (1) of the Insurance Code defines a contract of insurance as an agreement whereby one undertakes for a consideration to indemnify another against loss, damage or liability arising from an unknown or contingent event. An insurance contract exists where the following elements concur: 1. The insured has an insurable interest; 2. The insured is subject to a risk of loss by the happening of the designated peril; 3. The insurer assumes the risk; 4. Such assumption of risk is part of a general scheme to distribute actual losses among a large group of persons bearing a similar risk; and 5. In consideration of the insurers promise, the insured pays a premium.[8] Section 3 of the Insurance Code states that any contingent or unknown event, whether past or future, which may damnify a person having an insurable interest against him, may be insured against. Every person has an insurable interest in the life and health of himself. Section 10 provides:

Every person has an insurable interest in the life and health: (1) of himself, of his spouse and of his children;

the eligibility for health care coverage of the Proposed Members and that the acceptance of any Agreement issued on this application shall be a ratification of any correction in or addition to this application as stated in the space for Home Office Endorsement.[11] (Underscoring ours) In addition to the above condition, petitioner additionally required the applicant for authorization to inquire about the applicants medical history, thus: I hereby authorize any person, organization, or entity that has any record or knowledge of my health and/or that of __________ to give to the PhilamCare Health Systems, Inc. any and all information relative to any hospitalization, consultation, treatment or any other medical advice or examination. This authorization is in connection with the application for health care coverage only. A photographic copy of this authorization shall be as valid as the original.[12] (Underscoring ours) Petitioner cannot rely on the stipulation regarding Invalidation of agreement which reads: Failure to disclose or misrepresentation of any material information by the member in the application or medical examination, whether intentional or unintentional, shall automatically invalidate the Agreement from the very beginning and liability of Philamcare shall be limited to return of all Membership Fees paid. An undisclosed or misrepresented information is deemed material if its revelation would have resulted in the declination of the applicant by Philamcare or the assessment of a higher Membership Fee for the benefit or benefits applied for.[13] The answer assailed by petitioner was in response to the question relating to the medical history of the applicant. This largely depends on opinion rather than fact, especially coming from respondents husband who was not a medical doctor. Where matters of opinion or judgment are called for, answers made in good faith and without intent to deceive will not avoid a policy even though they are untrue.[14] Thus, (A)lthough false, a representation of the expectation, intention, belief, opinion, or judgment of the insured will not avoid the policy if there is no actual fraud in inducing the acceptance of the risk, or its acceptance at a lower rate of premium, and this is likewise the rule although the statement is material to the risk, if the statement is obviously of the foregoing character, since in such case the insurer is not justified in relying upon such statement, but is obligated to make further inquiry. There is a clear distinction between such a case and one in which the insured is fraudulently and

(2) of any person on whom he depends wholly or in part for education or support, or in whom he has a pecuniary interest; (3) of any person under a legal obligation to him for the payment of money, respecting property or service, of which death or illness might delay or prevent the performance; and (4) of any person upon whose life any estate or interest vested in him depends.

In the case at bar, the insurable interest of respondents husband in obtaining the health care agreement was his own health. The health care agreement was in the nature of non-life insurance, which is primarily a contract of indemnity.[9] Once the member incurs hospital, medical or any other expense arising from sickness, injury or other stipulated contingent, the health care provider must pay for the same to the extent agreed upon under the contract. Petitioner argues that respondents husband concealed a material fact in his application. It appears that in the application for health coverage, petitioners required respondents husband to sign an express authorization for any person, organization or entity that has any record or knowledge of his health to furnish any and all information relative to any hospitalization, consultation, treatment or any other medical advice or examination.[10] Specifically, the Health Care Agreement signed by respondents husband states: We hereby declare and agree that all statement and answers contained herein and in any addendum annexed to this application are full, complete and true and bind all parties in interest under the Agreement herein applied for, that there shall be no contract of health care coverage unless and until an Agreement is issued on this application and the full Membership Fee according to the mode of payment applied for is actually paid during the lifetime and good health of proposed Members; that no information acquired by any Representative of PhilamCare shall be binding upon PhilamCare unless set out in writing in the application; that any physician is, by these presents, expressly authorized to disclose or give testimony at anytime relative to any information acquired by him in his professional capacity upon any question affecting

intentionally states to be true, as a matter of expectation or belief, that which he then knows, to be actually untrue, or the impossibility of which is shown by the facts within his knowledge, since in such case the intent to deceive the insurer is obvious and amounts to actual fraud.[15] (Underscoring ours) The fraudulent intent on the part of the insured must be established to warrant rescission of the insurance contract.[16] Concealment as a defense for the health care provider or insurer to avoid liability is an affirmative defense and the duty to establish such defense by satisfactory and convincing evidence rests upon the provider or insurer. In any case, with or without the authority to investigate, petitioner is liable for claims made under the contract. Having assumed a responsibility under the agreement, petitioner is bound to answer the same to the extent agreed upon. In the end, the liability of the health care provider attaches once the member is hospitalized for the disease or injury covered by the agreement or whenever he avails of the covered benefits which he has prepaid. Under Section 27 of the Insurance Code, a concealment entitles the injured party to rescind a contract of insurance. The right to rescind should be exercised previous to the commencement of an action on the contract.[17] In this case, no rescission was made. Besides, the cancellation of health care agreements as in insurance policies require the concurrence of the following conditions: 1. Prior notice of cancellation to insured;

insurance contract, ambiguity must be strictly interpreted against the insurer and liberally in favor of the insured, especially to avoid forfeiture.[21] This is equally applicable to Health Care Agreements. The phraseology used in medical or hospital service contracts, such as the one at bar, must be liberally construed in favor of the subscriber, and if doubtful or reasonably susceptible of two interpretations the construction conferring coverage is to be adopted, and exclusionary clauses of doubtful import should be strictly construed against the provider.[22] Anent the incontestability of the membership of respondents husband, we quote with approval the following findings of the trial court: (U)nder the title Claim procedures of expenses, the defendant Philamcare Health Systems Inc. had twelve months from the date of issuance of the Agreement within which to contest the membership of the patient if he had previous ailment of asthma, and six months from the issuance of the agreement if the patient was sick of diabetes or hypertension. The periods having expired, the defense of concealment or misrepresentation no longer lie.[23] Finally, petitioner alleges that respondent was not the legal wife of the deceased member considering that at the time of their marriage, the deceased was previously married to another woman who was still alive. The health care agreement is in the nature of a contract of indemnity. Hence, payment should be made to the party who incurred the expenses. It is not controverted that respondent paid all the hospital and medical expenses. She is therefore entitled to reimbursement. The records adequately prove the expenses incurred by respondent for the deceaseds hospitalization, medication and the professional fees of the attending physicians.[24] WHEREFORE, in view of the foregoing, the petition is DENIED. The assailed decision of the Court of Appeals dated December 14, 1995 is AFFIRMED.SO ORDERED.

2. Notice must be based on the occurrence after effective date of the policy of one or more of the grounds mentioned; 3. Must be in writing, mailed or delivered to the insured at the address shown in the policy; 4. Must state the grounds relied upon provided in Section 64 of the Insurance Code and upon request of insured, to furnish facts on which cancellation is based.[18] None of the above pre-conditions was fulfilled in this case. When the terms of insurance contract contain limitations on liability, courts should construe them in such a way as to preclude the insurer from non-compliance with his obligation.[19] Being a contract of adhesion, the terms of an insurance contract are to be construed strictly against the party which prepared the contract the insurer.[20] By reason of the exclusive control of the insurance company over the terms and phraseology of the

G.R. No. 92492

June 17, 1993

THELMA VDA. DE CANILANG, vs CA.

In a decision dated 5 November 1985, Insurance Commissioner Armando Ansaldo ordered Great Pacific to pay P19,700 plus legal interest and P2,000.00 as attorney's fees after holding that: 1. the ailment of Jaime Canilang was not so serious that, even if it had been disclosed, it would not have affected Great Pacific's decision to insure him; 2. Great Pacific had waived its right to inquire into the health condition of the applicant by the issuance of the policy despite the lack of answers to "some of the pertinent questions" in the insurance application; 3. there was no intentional concealment on the part of the insured Jaime Canilang as he had thought that he was merely suffering from a minor ailment and simple cold; 10 and 4. Batas Pambansa Blg. 847 which voids an insurance contract, whether or not concealment was intentionally made, was not applicable to Canilang's case as that law became effective only on 1 June 1985. On appeal by Great Pacific, the Court of Appeals reversed and set aside the decision of the Insurance Commissioner and dismissed Thelma Canilang's complaint and Great Pacific's counterclaim. The Court of Appealed found that the use of the word "intentionally" by the Insurance Commissioner in defining and resolving the issue agreed upon by the parties at pre-trial before the Insurance Commissioner was not supported by the evidence; that the issue agreed upon by the parties had been whether the deceased insured, Jaime Canilang, made a material concealment as the state of his health at the time of the filing of insurance application, justifying respondent's denial of the claim. The Court of Appeals also found that the failure of Jaime Canilang to disclose previous medical consultation and treatment constituted material information which should have been communicated to Great Pacific to enable the latter to make proper inquiries. The Court of Appeals finally held that the Ng Gan Zee case which had involved misrepresentation was not applicable in respect of the case at bar which involves concealment. Petitioner Thelma Canilang is now before this Court on a Petition for Review on Certiorari alleging that: 1. . . . the Honorable Court of Appeals, speaking with due respect, erred in not holding that the issue in the case agreed upon between the parties before the

On 18 June 1982, Jaime Canilang consulted Dr. Wilfredo B. Claudio and was diagnosed as suffering from "sinus tachycardia." The doctor prescribed the following fro him: Trazepam, a tranquilizer; and Aptin, a beta-blocker drug. Mr. Canilang consulted the same doctor again on 3 August 1982 and this time was found to have "acute bronchitis." On next day, 4 August 1982, Jaime Canilang applied for a "non-medical" insurance policy with respondent Great Pacific Life Assurance Company ("Great Pacific") naming his wife, Thelma Canilang, as his beneficiary. 1 Jaime Canilang was issued ordinary life insurance Policy No. 345163, with the face value of P19,700, effective as of 9 August 1982. On 5 August 1983, Jaime Canilang died of "congestive heart failure," "anemia," and "chronic anemia." 2 Petitioner, widow and beneficiary of the insured, filed a claim with Great Pacific which the insurer denied on 5 December 1983 upon the ground that the insured had concealed material information from it. Petitioner then filed a complaint against Great Pacific with the Insurance Commission for recovery of the insurance proceeds. During the hearing called by the Insurance Commissioner, petitioner testified that she was not aware of any serious illness suffered by her late husband 3 and that, as far as she knew, her husband had died because of a kidney disorder. 4 A deposition given by Dr. Wilfredo Claudio was presented by petitioner. There Dr. Claudio stated that he was the family physician of the deceased Jaime Canilang 5 and that he had previously treated him for "sinus tachycardia" and "acute bronchitis." 6 Great Pacific for its part presented Dr. Esperanza Quismorio, a physician and a medical underwriter working for Great Pacific. 7 She testified that the deceased's insurance application had been approved on the basis of his medical declaration. 8 She explained that as a rule, medical examinations are required only in cases where the applicant has indicated in his application for insurance coverage that he has previously undergone medical consultation and hospitalization. 9

Insurance Commission is whether or not Jaime Canilang "intentionally" made material concealment in stating his state of health; 2. . . . at any rate, the non-disclosure of certain facts about his previous health conditions does not amount to fraud and private respondent is deemed to have waived inquiry thereto. 11 The medical declaration which was set out in the application for insurance executed by Jaime Canilang read as follows: MEDICAL DECLARATION I hereby declare that: (1) I have not been confined in any hospital, sanitarium or infirmary, nor receive any medical or surgical advice/attention within the last five (5) years. (2) I have never been treated nor consulted a physician for a heart condition, high blood pressure, cancer, diabetes, lung, kidney, stomach disorder, or any other physical impairment. (3) I am, to the best of my knowledge, in good health.

Illegible Signature of Applicant. 12 We note that in addition to the negative statements made by Mr. Canilang in paragraph 1 and 2 of the medical declaration, he failed to disclose in the appropriate space, under the caption "Exceptions," that he had twice consulted Dr. Wilfredo B. Claudio who had found him to be suffering from "sinus tachycardia" and "acute bronchitis." The relevant statutory provisions as they stood at the time Great Pacific issued the contract of insurance and at the time Jaime Canilang died, are set out in P.D. No. 1460, also known as the Insurance Code of 1978, which went into effect on 11 June 1978. These provisions read as follows: Sec. 26. A neglect to communicate that which a party knows and ought to communicate, is called a concealment. xxx xxx xxx

EXCEPTIONS:

Sec. 28. Each party to a contract of insurance must communicate to the other, in good faith, all factors within his knowledge which are material to the contract and as to which he makes no warranty, and which the other has not the means of ascertaining. (Emphasis supplied) Under the foregoing provisions, the information concealed must be information which the concealing party knew and "ought to [have] communicate[d]," that is to say, information which was "material to the contract." The test of materiality is contained in Section 31 of the Insurance Code of 1978 which reads: Sec. 31. Materially is to be determined not by the event, but solely by the probable and reasonable influence of the facts upon the party to whom the communication is due, in forming his estimate of the disadvantages of the proposed contract, or in making his inquiries. (Emphasis supplied) "Sinus tachycardia" is considered present "when the heart rate exceeds 100 beats per minute." 13 The symptoms of this condition include pounding in the chest and

GENERAL DECLARATION I hereby declare that all the foregoing answers and statements are complete, true and correct. I hereby agree that if there be any fraud or misrepresentation in the above statements material to the risk, the INSURANCE COMPANY upon discovery within two (2) years from the effective date of insurance shall have the right to declare such insurance null and void. That the liabilities of the Company under the said Policy/TA/Certificate shall accrue and begin only from the date of commencement of risk stated in the Policy/TA/Certificate, provided that the first premium is paid and the Policy/TA/Certificate is delivered to, and accepted by me in person, when I am in actual good health. Signed at Manila his 4th day of August, 1992.

sometimes faintness and weakness of the person affected. The following elaboration was offered by Great Pacific and set out by the Court of Appeals in its Decision: Sinus tachycardia is defined as sinus-initiated; heart rate faster than 100 beats per minute. (Harrison' s Principles of Internal Medicine, 8th ed. [1978], p. 1193.) It is, among others, a common reaction to heart disease, including myocardial infarction, and heart failure per se. (Henry J.L. Marriot, M.D., Electrocardiography, 6th ed., [1977], p. 127.) The medication prescribed by Dr. Claudio for treatment of Canilang's ailment on June 18, 1982, indicates the condition that said physician was trying to manage. Thus, he prescribed Trazepam, (Philippine Index of Medical Specialties (PIMS), Vol. 14, No. 3, Dec. 1985, p. 112) which is anti-anxiety, anti-convulsant, muscle-relaxant; and Aptin, (Idem, p. 36) a cardiac drug, for palpitations and nervous heart. Such treatment could have been a very material information to the insurer in determining the action to be take on Canilang's application for life insurance coverage. 14 We agree with the Court of Appeals that the information which Jaime Canilang failed to disclose was material to the ability of Great Pacific to estimate the probable risk he presented as a subject of life insurance. Had Canilang disclosed his visits to his doctor, the diagnosis made and medicines prescribed by such doctor, in the insurance application, it may be reasonably assumed that Great Pacific would have made further inquiries and would have probably refused to issue a non-medical insurance policy or, at the very least, required a higher premium for the same coverage. 15 The materiality of the information withheld by Great Pacific did not depend upon the state of mind of Jaime Canilang. A man's state of mind or subjective belief is not capable of proof in our judicial process, except through proof of external acts or failure to act from which inferences as to his subjective belief may be reasonably drawn. Neither does materiality depend upon the actual or physical events which ensue. Materiality relates rather to the "probable and reasonable influence of the facts" upon the party to whom the communication should have been made, in assessing the risk involved in making or omitting to make further inquiries and in accepting the application for insurance; that "probable and reasonable influence of the facts" concealed must, of course, be determined objectively, by the judge ultimately. The insurance Great Pacific applied for was a "non-medical" insurance policy. In Saturnino v. Philippine-American Life Insurance Company, 16 this Court held that: . . . if anything, the waiver of medical examination [in a non-medical insurance contract] renders even more material the information required of the applicant

concerning previous condition of health and diseases suffered, for such information necessarily constitutes an important factor which the insurer takes into consideration in deciding whether to issue the policy or not . . . . 17 (Emphasis supplied) The Insurance Commissioner had also ruled that the failure of Great Pacific to convey certain information to the insurer was not "intentional" in nature, for the reason that Jaime Canilang believed that he was suffering from minor ailment like a common cold. Section 27 of the Insurance Code of 1978 as it existed from 1974 up to 1985, that is, throughout the time range material for present purposes, provided that: Sec. 27. insurance. A concealment entitles the injured party to rescind a contract of

The preceding statute, Act No. 2427, as it stood from 1914 up to 1974, had provided: Sec. 26. A concealment, whether intentional or unintentional, entitles the injured party to rescind a contract of insurance. (Emphasis supplied) Upon the other hand, in 1985, the Insurance Code of 1978 was amended by B.P. Blg. 874. This subsequent statute modified Section 27 of the Insurance Code of 1978 so as to read as follows: Sec. 27. A concealment whether intentional or unintentional entitles the injured party to rescind a contract of insurance. (Emphasis supplied) The unspoken theory of the Insurance Commissioner appears to have been that by deleting the phrase "intentional or unintentional," the Insurance Code of 1978 (prior to its amendment by B.P. Blg. 874) intended to limit the kinds of concealment which generate a right to rescind on the part of the injured party to "intentional concealments." This argument is not persuasive. As a simple matter of grammar, it may be noted that "intentional" and "unintentional" cancel each other out. The net result therefore of the phrase "whether intentional or unitentional" is precisely to leave unqualified the term "concealment." Thus, Section 27 of the Insurance Code of 1978 is properly read as referring to "any concealment" without regard to whether such concealment is intentional or unintentional. The phrase "whether intentional or unintentional" was in fact superfluous. The deletion of the phrase "whether intentional or unintentional" could not have had the effect of imposing an affirmative requirement that a concealment must be intentional if it is to entitle the injured party to rescind a contract of insurance. The restoration in 1985 by B.P. Blg. 874 of the

phrase "whether intentional or unintentional" merely underscored the fact that all throughout (from 1914 to 1985), the statute did not require proof that concealment must be "intentional" in order to authorize rescission by the injured party. In any case, in the case at bar, the nature of the facts not conveyed to the insurer was such that the failure to communicate must have been intentional rather than merely inadvertent. For Jaime Canilang could not have been unaware that his heart beat would at times rise to high and alarming levels and that he had consulted a doctor twice in the two (2) months before applying for non-medical insurance. Indeed, the last medical consultation took place just the day before the insurance application was filed. In all probability, Jaime Canilang went to visit his doctor precisely because of the discomfort and concern brought about by his experiencing "sinus tachycardia." We find it difficult to take seriously the argument that Great Pacific had waived inquiry into the concealment by issuing the insurance policy notwithstanding Canilang's failure to set out answers to some of the questions in the insurance application. Such failure precisely constituted concealment on the part of Canilang. Petitioner's argument, if accepted, would obviously erase Section 27 from the Insurance Code of 1978. It remains only to note that the Court of Appeals finding that the parties had not agreed in the pretrial before the Insurance Commission that the relevant issue was whether or not Jaime Canilang had intentionally concealed material information from the insurer, was supported by the evidence of record, i.e., the Pre-trial Order itself dated 17 October 1984 and the Minutes of the Pre-trial Conference dated 15 October 1984, which "readily shows that the word "intentional" does not appear in the statement or definition of the issue in the said Order and Minutes." 18 WHEREFORE, the Petition for Review is DENIED for lack of merit and the Decision of the Court of Appeals dated 16 October 1989 in C.A.-G.R. SP No. 08696 is hereby AFFIRMED. No pronouncement as to the costs.SO ORDERED.

recovery of the proceeds from their late father's policy. The facts of the case as found by the Court of Appeals are: Petitioners appeal from the Decision of the Insurance Commissioner dismissing herein petitioners' complaint against respondent Philippine American Life Insurance Company for the recovery of the proceeds of Policy No. 1082467 in the amount of P 80,000.00. On September 23,1973, Tan Lee Siong, father of herein petitioners, applied for life insurance in the amount of P 80,000.00 with respondent company. Said application was approved and Policy No. 1082467 was issued effective November 6,1973, with petitioners the beneficiaries thereof (Exhibit A). On April 26,1975, Tan Lee Siong died of hepatoma (Exhibit B). Petitioners then filed with respondent company their claim for the proceeds of the life insurance policy. However, in a letter dated September 11, 1975, respondent company denied petitioners' claim and rescinded the policy by reason of the alleged misrepresentation and concealment of material facts made by the deceased Tan Lee Siong in his application for insurance (Exhibit 3). The premiums paid on the policy were thereupon refunded . Alleging that respondent company's refusal to pay them the proceeds of the policy was unjustified and unreasonable, petitioners filed on November 27, 1975, a complaint against the former with the Office of the Insurance Commissioner, docketed as I.C. Case No. 218. After hearing the evidence of both parties, the Insurance Commissioner rendered judgment on August 9, 1977, dismissing petitioners' complaint. (Rollo, pp. 91-92) The Court of Appeals dismissed ' the petitioners' appeal from the Insurance Commissioner's decision for lack of merit Hence, this petition.

TAN v. CA G.R. No. 48049 June 29, 1989 The petitioners raise the following issues in their assignment of errors, to wit: This is a petition for review on certiorari of the Court of Appeals' decision affirming the decision of the Insurance Commissioner which dismissed the petitioners' complaint against respondent Philippine American Life Insurance Company for the A. The conclusion in law of respondent Court that respondent insurer has the right to rescind the policy contract when insured is already dead is not in accordance with existing law and applicable jurisprudence.

B. The conclusion in law of respondent Court that respondent insurer may be allowed to avoid the policy on grounds of concealment by the deceased assured, is contrary to the provisions of the policy contract itself, as well as, of applicable legal provisions and established jurisprudence. C. The inference of respondent Court that respondent insurer was misled in issuing the policy are manifestly mistaken and contrary to admitted evidence. (Rollo, p. 7) The petitioners contend that the respondent company no longer had the right to rescind the contract of insurance as rescission must allegedly be done during the lifetime of the insured within two years and prior to the commencement of action. The contention is without merit. The pertinent section in the Insurance Code provides: Section 48. Whenever a right to rescind a contract of insurance is given to the insurer by any provision of this chapter, such right must be exercised previous to the commencement of an action on the contract. After a policy of life insurance made payable on the death of the insured shall have been in force during the lifetime of the insured for a period of two years from the date of its issue or of its last reinstatement, the insurer cannot prove that the policy is void ab initio or is rescindable by reason of the fraudulent concealment or misrepresentation of the insured or his agent. According to the petitioners, the Insurance Law was amended and the second paragraph of Section 48 added to prevent the insurance company from exercising a right to rescind after the death of the insured. The so-called "incontestability clause" precludes the insurer from raising the defenses of false representations or concealment of material facts insofar as health and previous diseases are concerned if the insurance has been in force for at least two years during the insured's lifetime. The phrase "during the lifetime" found in Section 48 simply means that the policy is no longer considered in force after the insured has died. The key phrase in the second paragraph of Section 48 is "for a period of two years."

As noted by the Court of Appeals, to wit: The policy was issued on November 6,1973 and the insured died on April 26,1975. The policy was thus in force for a period of only one year and five months. Considering that the insured died before the two-year period had lapsed, respondent company is not, therefore, barred from proving that the policy is void ab initio by reason of the insured's fraudulent concealment or misrepresentation. Moreover, respondent company rescinded the contract of insurance and refunded the premiums paid on September 11, 1975, previous to the commencement of this action on November 27,1975. (Rollo, pp. 99-100) xxx xxx xxx

The petitioners contend that there could have been no concealment or misrepresentation by their late father because Tan Lee Siong did not have to buy insurance. He was only pressured by insistent salesmen to do so. The petitioners state: Here then is a case of an assured whose application was submitted because of repeated visits and solicitations by the insurer's agent. Assured did not knock at the door of the insurer to buy insurance. He was the object of solicitations and visits. Assured was a man of means. He could have obtained a bigger insurance, not just P 80,000.00. If his purpose were to misrepresent and to conceal his ailments in anticipation of death during the two-year period, he certainly could have gotten a bigger insurance. He did not. Insurer Philamlife could have presented as witness its Medical Examiner Dr. Urbano Guinto. It was he who accomplished the application, Part II, medical. Philamlife did not. Philamlife could have put to the witness stand its Agent Bienvenido S. Guinto, a relative to Dr. Guinto, Again Philamlife did not. (pp. 138139, Rollo) xxx xxx xxx

This Honorable Supreme Court has had occasion to denounce the pressure and practice indulged in by agents in selling insurance. At one time or another most of us have been subjected to that pressure, that practice. This court took judicial cognizance

of the whirlwind pressure of insurance selling-especially of the agent's practice of 'supplying the information, preparing and answering the application, submitting the application to their companies, concluding the transactions and otherwise smoothing out all difficulties. We call attention to what this Honorable Court said in Insular Life v. Feliciano, et al., 73 Phil. 201; at page 205: It is of common knowledge that the selling of insurance today is subjected to the whirlwind pressure of modern salesmanship. Insurance companies send detailed instructions to their agents to solicit and procure applications. These agents are to be found all over the length and breadth of the land. They are stimulated to more active efforts by contests and by the keen competition offered by the other rival insurance companies. They supply all the information, prepare and answer the applications, submit the applications to their companies, conclude the transactions, and otherwise smooth out all difficulties. The agents in short do what the company set them out to do. The Insular Life case was decided some forty years ago when the pressure of insurance salesmanship was not overwhelming as it is now; when the population of this country was less than one-fourth of what it is now; when the insurance companies competing with one another could be counted by the fingers. (pp. 140-142, Rollo) xxx xxx xxx

The insurer has two years from the date of issuance of the insurance contract or of its last reinstatement within which to contest the policy, whether or not, the insured still lives within such period. After two years, the defenses of concealment or misrepresentation, no matter how patent or well founded, no longer lie. Congress felt this was a sufficient answer to the various tactics employed by insurance companies to avoid liability. The petitioners' interpretation would give rise to the incongruous situation where the beneficiaries of an insured who dies right after taking out and paying for a life insurance policy, would be allowed to collect on the policy even if the insured fraudulently concealed material facts. The petitioners argue that no evidence was presented to show that the medical terms were explained in a layman's language to the insured. They state that the insurer should have presented its two medical field examiners as witnesses. Moreover, the petitioners allege that the policy intends that the medical examination must be conducted before its issuance otherwise the insurer "waives whatever imperfection by ratification." We agree with the Court of Appeals which ruled: On the other hand, petitioners argue that no evidence was presented by respondent company to show that the questions appearing in Part II of the application for insurance were asked, explained to and understood by the deceased so as to prove concealment on his part. The same is not well taken. The deceased, by affixing his signature on the application form, affirmed the correctness of all the entries and answers appearing therein. It is but to be expected that he, a businessman, would not have affixed his signature on the application form unless he clearly understood its significance. For, the presumption is that a person intends the ordinary consequence of his voluntary act and takes ordinary care of his concerns. [Sec. 5(c) and (d), Rule 131, Rules of Court]. The evidence for respondent company shows that on September 19,1972, the deceased was examined by Dr. Victoriano Lim and was found to be diabetic and hypertensive; that by January, 1973, the deceased was complaining of progressive weight loss and abdominal pain and was diagnosed to be suffering from hepatoma, (t.s.n. August 23, 1976, pp. 8-10; Exhibit 2). Another physician, Dr. Wenceslao Vitug, testified that the deceased came to see him on December 14, 1973 for consolation and claimed to have been diabetic for five years. (t.s.n., Aug. 23,1976, p. 5; Exhibit 6) Because of the concealment made by the deceased of his consultations and treatments for hypertension, diabetes and liver disorders, respondent company

In the face of all the above, it would be unjust if, having been subjected to the whirlwind pressure of insurance salesmanship this Court itself has long denounced, the assured who dies within the two-year period, should stand charged of fraudulent concealment and misrepresentation." (p. 142, Rollo) The legislative answer to the arguments posed by the petitioners is the "incontestability clause" added by the second paragraph of Section 48.

was thus misled into accepting the risk and approving his application as medically standard (Exhibit 5- C) and dispensing with further medical investigation and examination (Exhibit 5-A). For as long as no adverse medical history is revealed in the application form, an applicant for insurance is presumed to be healthy and physically fit and no further medical investigation or examination is conducted by respondent company. (t.s.n., April 8,1976, pp. 6-8). (Rollo, pp. 96-98) There is no strong showing that we should apply the "fine print" or "contract of adhesion" rule in this case. (Sweet Lines, Inc. v. Teves, 83 SCRA 361 [1978]). The petitioners cite: It is a matter of common knowledge that large amounts of money are collected from ignorant persons by companies and associations which adopt high sounding titles and print the amount of benefits they agree to pay in large black-faced type, following such undertakings by fine print conditions which destroy the substance of the promise. All provisions, conditions, or exceptions which in any way tend to work a forfeiture of the policy should be construed most strongly against those for whose benefit they are inserted, and most favorably toward those against whom they are meant to operate. (Trinidad v. Orient Protective Assurance Assn., 67 Phil. 184) There is no showing that the questions in the application form for insurance regarding the insured's medical history are in smaller print than the rest of the printed form or that they are designed in such a way as to conceal from the applicant their importance. If a warning in bold red letters or a boxed warning similar to that required for cigarette advertisements by the Surgeon General of the United States is necessary, that is for Congress or the Insurance Commission to provide as protection against high pressure insurance salesmanship. We are limited in this petition to ascertaining whether or not the respondent Court of Appeals committed reversible error. It is the petitioners' burden to show that the factual findings of the respondent court are not based on substantial evidence or that its conclusions are contrary to applicable law and jurisprudence. They have failed to discharge that burden. WHEREFORE, the petition is hereby DENIED for lack of merit. The questioned decision of the Court of Appeals is AFFIRMED. SO ORDERED.

PRUDENTIAL GUARANTEE v. TRANS-ASIA SHIPPING LINES, INC., G.R. No. 151890

This is a consolidation of two separate Petitions for Review on Certiorari filed by petitioner Prudential Guarantee and Assurance, Inc. (PRUDENTIAL) in G.R. No. 151890 and Trans-Asia Shipping Lines, Inc. (TRANS-ASIA) in G.R. No. 151991, assailing the Decision[1] dated 6 November 2001 of the Court of Appeals in CA G.R. CV No. 68278, which reversed the Judgment[2] dated 6 June 2000 of the Regional Trial Court (RTC), Branch 13, Cebu City in Civil Case No. CEB-20709. The 29 January 2002 Resolution[3] of the Court of Appeals, denying PRUDENTIALs Motion for Reconsideration and TRANS-ASIAs Partial Motion for Reconsideration of the 6 November 2001 Decision, is likewise sought to be annulled and set aside. The Facts The material antecedents as found by the court a quo and adopted by the appellate court are as follows: Plaintiff [TRANS-ASIA] is the owner of the vessel M/V Asia Korea. In consideration of payment of premiums, defendant [PRUDENTIAL] insured M/V Asia Korea for loss/damage of the hull and machinery arising from perils, inter alia, of fire and explosion for the sum of P40 Million, beginning [from] the period [of] July 1, 1993 up to July 1, 1994. This is evidenced by Marine Policy No. MH93/1363 (Exhibits A to A-11). On October 25, 1993, while the policy was in force, a fire broke out while [M/V Asia Korea was] undergoing repairs at the port of Cebu. On October 26, 1993 plaintiff [TRANS-ASIA] filed its notice of claim for damage sustained by the vessel. This is evidenced by a letter/formal claim of even date (Exhibit B). Plaintiff [TRANS-ASIA] reserved its right to subsequently notify defendant [PRUDENTIAL] as to the full amount of the claim upon final survey and determination by average adjuster Richard Hogg International (Phil.) of the damage sustained by reason of fire. An adjusters report on the fire in question was submitted by Richard Hogg International together with the U-Marine Surveyor Report (Exhibits 4 to 4-115). On May 29, 1995[,] plaintiff [TRANS-ASIA] executed a document denominated Loan and Trust receipt, a portion of which read (sic):

Received from Prudential Guarantee and Assurance, Inc., the sum of PESOS THREE MILLION ONLY (P3,000,000.00) as a loan without interest under Policy No. MH 93/1353 [sic], repayable only in the event and to the extent that any net recovery is made by Trans-Asia Shipping Corporation, from any person or persons, corporation or corporations, or other parties, on account of loss by any casualty for which they may be liable occasioned by the 25 October 1993: Fire on Board. (Exhibit 4) In a letter dated 21 April 1997 defendant [PRUDENTIAL] denied plaintiffs claim (Exhibit 5). The letter reads: After a careful review and evaluation of your claim arising from the abovecaptioned incident, it has been ascertained that you are in breach of policy conditions, among them WARRANTED VESSEL CLASSED AND CLASS MAINTAINED. Accordingly, we regret to advise that your claim is not compensable and hereby DENIED. This was followed by defendants letter dated 21 July 1997 requesting the return or payment of the P3,000,000.00 within a period of ten (10) days from receipt of the letter (Exhibit 6).[4]

without interest and without prejudice to the final evaluation of the claim, including the amounts of P500,000.00, for survey fees and P200,000.00, representing attorneys fees. The Ruling of the Trial Court On 6 June 2000, the court a quo rendered Judgment[8] finding for (therein defendant) PRUDENTIAL. It ruled that a determination of the parties liabilities hinged on whether TRANS-ASIA violated and breached the policy conditions on WARRANTED VESSEL CLASSED AND CLASS MAINTAINED. It interpreted the provision to mean that TRANS-ASIA is required to maintain the vessel at a certain class at all times pertinent during the life of the policy. According to the court a quo, TRANS-ASIA failed to prove compliance of the terms of the warranty, the violation thereof entitled PRUDENTIAL, the insured party, to rescind the contract.[9] Further, citing Section 107[10] of the Insurance Code, the court a quo ratiocinated that the concealment made by TRANS-ASIA that the vessel was not adequately maintained to preserve its class was a material concealment sufficient to avoid the policy and, thus, entitled the injured party to rescind the contract. The court a quo found merit in PRUDENTIALs contention that there was nothing in the adjustment of the particular average submitted by the adjuster that would show that TRANSASIA was not in breach of the policy. Ruling on the denominated loan and trust receipt, the court a quo said that in substance and in form, the same is a receipt for a loan. It held that if TRANS-ASIA intended to receive the amount of P3,000,000.00 as advance payment, it should have so clearly stated as such. The court a quo did not award PRUDENTIALs claim for P500,000.00, representing expert survey fees on the ground of lack of sufficient basis in support thereof. Neither did it award attorneys fees on the rationalization that the instant case does not fall under the exceptions stated in Article 2208[11] of the Civil Code. However, the court a quo granted PRUDENTIALs counterclaim stating that there is factual and legal basis for TRANS-ASIA to return the amount of P3,000,000.00 by way of loan without interest. The decretal portion of the Judgment of the RTC reads: WHEREFORE, judgment is hereby rendered DISMISSING the complaint for its failure to prove a cause of action.

Following this development, on 13 August 1997, TRANS-ASIA filed a Complaint[5] for Sum of Money against PRUDENTIAL with the RTC of Cebu City, docketed as Civil Case No. CEB-20709, wherein TRANS-ASIA sought the amount of P8,395,072.26 from PRUDENTIAL, alleging that the same represents the balance of the indemnity due upon the insurance policy in the total amount of P11,395,072.26. TRANS-ASIA similarly sought interest at 42% per annum citing Section 243[6] of Presidential Decreee No. 1460, otherwise known as the Insurance Code, as amended. In its Answer,[7] PRUDENTIAL denied the material allegations of the Complaint and interposed the defense that TRANS-ASIA breached insurance policy conditions, in particular: WARRANTED VESSEL CLASSED AND CLASS MAINTAINED. PRUDENTIAL further alleged that it acted as facts and law require and incurred no liability to TRANS-ASIA; that TRANS-ASIA has no cause of action; and, that its claim has been effectively waived and/or abandoned, or it is estopped from pursuing the same. By way of a counterclaim, PRUDENTIAL sought a refund of P3,000,000.00, which it allegedly advanced to TRANS-ASIA by way of a loan

On defendants counterclaim, plaintiff is directed to return the sum of P3,000,000.00 representing the loan extended to it by the defendant, within a period of ten (10) days from and after this judgment shall have become final and executory.[12]

The Ruling of the Court of Appeals

On appeal by TRANS-ASIA, the Court of Appeals, in its assailed Decision of 6 November 2001, reversed the 6 June 2000 Judgment of the RTC. On the issue of TRANS-ASIAs alleged breach of warranty of the policy condition CLASSED AND CLASS MAINTAINED, the Court of Appeals ruled that PRUDENTIAL, as the party asserting the non-compensability of the loss had the burden of proof to show that TRANS-ASIA breached the warranty, which burden it failed to discharge. PRUDENTIAL cannot rely on the lack of certification to the effect that TRANS-ASIA was CLASSED AND CLASS MAINTAINED as its sole basis for reaching the conclusion that the warranty was breached. The Court of Appeals opined that the lack of a certification does not necessarily mean that the warranty was breached by TRANS-ASIA. Instead, the Court of Appeals considered PRUDENTIALs admission that at the time the insurance contract was entered into between the parties, the vessel was properly classed by Bureau Veritas, a classification society recognized by the industry. The Court of Appeals similarly gave weight to the fact that it was the responsibility of Richards Hogg International (Phils.) Inc., the average adjuster hired by PRUDENTIAL, to secure a copy of such certification to support its conclusion that mere absence of a certification does not warrant denial of TRANS-ASIAs claim under the insurance policy. In the same token, the Court of Appeals found the subject warranty allegedly breached by TRANS-ASIA to be a rider which, while contained in the policy, was inserted by PRUDENTIAL without the intervention of TRANS-ASIA. As such, it partakes of a nature of a contract dadhesion which should be construed against PRUDENTIAL, the party which drafted the contract. Likewise, according to the Court of Appeals, PRUDENTIALs renewal of the insurance policy from noon of 1 July 1994 to noon of 1 July 1995, and then again, until noon of 1 July 1996 must be deemed a waiver by PRUDENTIAL of any breach of warranty committed by TRANS-ASIA.

Further, the Court of Appeals, contrary to the ruling of the court a quo, interpreted the transaction between PRUDENTIAL and TRANS-ASIA as one of subrogation, instead of a loan. The Court of Appeals concluded that TRANS-ASIA has no obligation to pay back the amount of P3,000.000.00 to PRUDENTIAL based on its finding that the aforesaid amount was PRUDENTIALs partial payment to TRANSASIAs claim under the policy. Finally, the Court of Appeals denied TRANSASIAs prayer for attorneys fees, but held TRANS-ASIA entitled to double interest on the policy for the duration of the delay of payment of the unpaid balance, citing Section 244[13] of the Insurance Code. Finding for therein appellant TRANS-ASIA, the Court of Appeals ruled in this wise: WHEREFORE, the foregoing consideration, We find for Appellant. The instant appeal is ALLOWED and the Judgment appealed from REVERSED. The P3,000,000.00 initially paid by appellee Prudential Guarantee Assurance Incorporated to appellant Trans-Asia and covered by a Loan and Trust Receipt dated 29 May 1995 is HELD to be in partial settlement of the loss suffered by appellant and covered by Marine Policy No. MH93/1363 issued by appellee. Further, appellee is hereby ORDERED to pay appellant the additional amount of P8,395,072.26 representing the balance of the loss suffered by the latter as recommended by the average adjuster Richard Hogg International (Philippines) in its Report, with double interest starting from the time Richard Hoggs Survey Report was completed, or on 13 August 1996, until the same is fully paid. All other claims and counterclaims are hereby DISMISSED. All costs against appellee.[14]

Not satisfied with the judgment, PRUDENTIAL and TRANS-ASIA filed a Motion for Reconsideration and Partial Motion for Reconsideration thereon, respectively, which motions were denied by the Court of Appeals in the Resolution dated 29 January 2002. The Issues Aggrieved, PRUDENTIAL filed before this Court a Petition for Review, docketed as G.R. No. 151890, relying on the following grounds, viz:

I. THE AWARD IS GROSSLY UNCONSCIONABLE.

THE COURT OF APPEALS ERRED IN HOLDING THAT THE LOAN AND TRUST RECEIPT EXECUTED BY TRANS-ASIA IS AN ADVANCE ON THE POLICY, THUS CONSTITUTING PARTIAL PAYMENT THEREOF. VII.

II. THE COURT OF APPEALS ERRED IN HOLDING THAT THERE WAS NO VIOLATION BY TRANS-ASIA OF A MATERIAL WARRANTY, NAMELY, WARRANTY CLAUSE NO. 5, OF THE INSURANCE POLICY. THE COURT OF APPEALS ERRED IN HOLDING THAT THE ACCEPTANCE BY PRUDENTIAL OF THE FINDINGS OF RICHARDS HOGG IS INDICATIVE OF A WAIVER ON THE PART OF PRUDENTIAL OF ANY VIOLATION BY TRANS-ASIA OF THE WARRANTY.

III. THE COURT OF APPEALS ERRED IN HOLDING THAT PRUDENTIAL, AS INSURER HAD THE BURDEN OF PROVING THAT THE ASSURED, TRANSASIA, VIOLATED A MATERIAL WARRANTY.

VIII. THE COURT OF APPEALS ERRRED (sic) IN REVERSING THE TRIAL COURT, IN FINDING THAT PRUDENTIAL UNJUSTIFIABLY REFUSED TO PAY THE CLAIM AND IN ORDERING PRUDENTIAL TO PAY TRANS-ASIA P8,395,072.26 PLUS DOUBLE INTEREST FROM 13 AUGUST 1996, UNTIL [THE] SAME IS FULLY PAID.[15]

IV. THE COURT OF APPEALS ERRED IN HOLDING THAT THE WARRANTY CLAUSE EMBODIED IN THE INSURANCE POLICY CONTRACT WAS A MERE RIDER. Similarly, TRANS-ASIA, disagreeing in the ruling of the Court of Appeals filed a Petition for Review docketed as G.R. No. 151991, raising the following grounds for the allowance of the petition, to wit: I. THE HONORABLE COURT OF APPEALS ERRED IN NOT AWARDING ATTORNEYS FEES TO PETITIONER TRANS-ASIA ON THE GROUND THAT SUCH CAN ONLY BE AWARDED IN THE CASES ENUMERATED IN ARTICLE 2208 OF THE CIVIL CODE, AND THERE BEING NO BAD FAITH ON THE PART OF RESPONDENT PRUDENTIAL IN DENYING HEREIN PETITIONER TRANS-ASIAS INSURANCE CLAIM. II. THE DOUBLE INTEREST REFERRED TO IN THE DECISION DATED 06 NOVEMBER 2001 SHOULD BE CONSTRUED TO MEAN DOUBLE INTEREST BASED ON THE LEGAL INTEREST OF 12%, OR INTEREST AT THE RATE OF 24% PER ANNUM.[16]

V. THE COURT OF APPEALS ERRED IN HOLDING THAT THE ALLEGED RENEWALS OF THE POLICY CONSTITUTED A WAIVER ON THE PART OF PRUDENTIAL OF THE BREACH OF THE WARRANTY BY TRANS-ASIA.

VI.

recovery of TRANS-ASIA under the policy, the violation of which entitled PRUDENTIAL to rescind the contract under Sec. 74[21] of the Insurance Code. In our Resolution of 2 December 2002, we granted TRANS-ASIAs Motion for Consolidation[17] of G.R. Nos. 151890 and 151991;[18] hence, the instant consolidated petitions. In sum, for our main resolution are: (1) the liability, if any, of PRUDENTIAL to TRANS-ASIA arising from the subject insurance contract; (2) the liability, if any, of TRANS-ASIA to PRUDENTIAL arising from the transaction between the parties as evidenced by a document denominated as Loan and Trust Receipt, dated 29 May 1995; and (3) the amount of interest to be imposed on the liability, if any, of either or both parties. Ruling of the Court Prefatorily, it must be emphasized that in a petition for review, only questions of law, and not questions of fact, may be raised.[19] This rule may be disregarded only when the findings of fact of the Court of Appeals are contrary to the findings and conclusions of the trial court, or are not supported by the evidence on record.[20] In the case at bar, we find an incongruence between the findings of fact of the Court of Appeals and the court a quo, thus, in our determination of the issues, we are constrained to assess the evidence adduced by the parties to make appropriate findings of facts as are necessary. I. A. PRUDENTIAL failed to establish that TRANS-ASIA violated and breached the policy condition on WARRANTED VESSEL CLASSED AND CLASS MAINTAINED, as contained in the subject insurance contract. The warranty condition CLASSED AND CLASS MAINTAINED was explained by PRUDENTIALs Senior Manager of the Marine and Aviation Division, Lucio Fernandez. The pertinent portions of his testimony on direct examination is reproduced hereunder, viz: ATTY. LIM Q Please tell the court, Mr. Witness, the result of the evaluation of this claim, what final action was taken? A It was eventually determined that there was a breach of the policy condition, and basically there is a breach of policy warranty condition and on that basis the claim was denied. Q To refer you (sic) the policy warranty condition, I am showing to you a policy here marked as Exhibits 1, 1-A series, please point to the warranty in the policy which you said was breached or violated by the plaintiff which constituted your basis for denying the claim as you testified. A Warranted Vessel Classed and Class Maintained.

ATTY. LIM Witness pointing, Your Honor, to that portion in Exhibit 1-A which is the second page of the policy below the printed words: Clauses, Endorsements, Special Conditions and Warranties, below this are several typewritten clauses and the witness pointed out in particular the clause reading: Warranted Vessel Classed and Class Maintained. COURT Q Will you explain that particular phrase?

In resisting the claim of TRANS-ASIA, PRUDENTIAL posits that TRANSASIA violated an express and material warranty in the subject insurance contract, i.e., Marine Insurance Policy No. MH93/1363, specifically Warranty Clause No. 5 thereof, which stipulates that the insured vessel, M/V ASIA KOREA is required to be CLASSED AND CLASS MAINTAINED. According to PRUDENTIAL, on 25 October 1993, or at the time of the occurrence of the fire, M/V ASIA KOREA was in violation of the warranty as it was not CLASSED AND CLASS MAINTAINED. PRUDENTIAL submits that Warranty Clause No. 5 was a condition precedent to the

A Yes, a warranty is a condition that has to be complied with by the insured. When we say a class warranty, it must be entered in the classification society.

COURT Slowly. WITNESS (continued) A A classification society is an organization which sets certain standards for a vessel to maintain in order to maintain their membership in the classification society. So, if they failed to meet that standard, they are considered not members of that class, and thus breaching the warranty, that requires them to maintain membership or to maintain their class on that classification society. And it is not sufficient that the member of this classification society at the time of a loss, their membership must be continuous for the whole length of the policy such that during the effectivity of the policy, their classification is suspended, and then thereafter, they get reinstated, that again still a breach of the warranty that they maintained their class (sic). Our maintaining team membership in the classification society thereby maintaining the standards of the vessel (sic). ATTY. LIM Q Can you mention some classification societies that you know? A Well we have the Bureau Veritas, American Bureau of Shipping, D&V Local Classification Society, The Philippine Registration of Ships Society, China Classification, NKK and Company Classification Society, and many others, we have among others, there are over 20 worldwide. [22]

In our rule on evidence, TRANS-ASIA, as the plaintiff below, necessarily has the burden of proof to show proof of loss, and the coverage thereof, in the subject insurance policy. However, in the course of trial in a civil case, once plaintiff makes out a prima facie case in his favor, the duty or the burden of evidence shifts to defendant to controvert plaintiffs prima facie case, otherwise, a verdict must be returned in favor of plaintiff.[23] TRANS-ASIA was able to establish proof of loss and the coverage of the loss, i.e., 25 October 1993: Fire on Board. Thereafter, the burden of evidence shifted to PRUDENTIAL to counter TRANS-ASIAs case, and to prove its special and affirmative defense that TRANS-ASIA was in violation of the particular condition on CLASSED AND CLASS MAINTAINED. We sustain the findings of the Court of Appeals that PRUDENTIAL was not successful in discharging the burden of evidence that TRANS-ASIA breached the subject policy condition on CLASSED AND CLASS MAINTAINED. Foremost, PRUDENTIAL, through the Senior Manager of its Marine and Aviation Division, Lucio Fernandez, made a categorical admission that at the time of the procurement of the insurance contract in July 1993, TRANS-ASIAs vessel, M/V Asia Korea was properly classed by Bureau Veritas, thus: Q Kindly examine the records particularly the policy, please tell us if you know whether M/V Asia Korea was classed at the time (sic) policy was procured perthe (sic) insurance was procured that Exhibit 1 on 1st July 1993 (sic). WITNESS A I recall that they were classed.

At the outset, it must be emphasized that the party which alleges a fact as a matter of defense has the burden of proving it. PRUDENTIAL, as the party which asserted the claim that TRANS-ASIA breached the warranty in the policy, has the burden of evidence to establish the same. Hence, on the part of PRUDENTIAL lies the initiative to show proof in support of its defense; otherwise, failing to establish the same, it remains self-serving. Clearly, if no evidence on the alleged breach of TRANS-ASIA of the subject warranty is shown, a fortiori, TRANS-ASIA would be successful in claiming on the policy. It follows that PRUDENTIAL bears the burden of evidence to establish the fact of breach.

ATTY. LIM Q A With what classification society? I believe with Bureau Veritas.[24]

As found by the Court of Appeals and as supported by the records, Bureau Veritas is a classification society recognized in the marine industry. As it is undisputed that TRANS-ASIA was properly classed at the time the contract of insurance was entered into, thus, it becomes incumbent upon PRUDENTIAL to show evidence that the

status of TRANS-ASIA as being properly CLASSED by Bureau Veritas had shifted in violation of the warranty. Unfortunately, PRUDENTIAL failed to support the allegation. We are in accord with the ruling of the Court of Appeals that the lack of a certification in PRUDENTIALs records to the effect that TRANS-ASIAs M/V Asia Korea was CLASSED AND CLASS MAINTAINED at the time of the occurrence of the fire cannot be tantamount to the conclusion that TRANS-ASIA in fact breached the warranty contained in the policy. With more reason must we sustain the findings of the Court of Appeals on the ground that as admitted by PRUDENTIAL, it was likewise the responsibility of the average adjuster, Richards Hogg International (Phils.), Inc., to secure a copy of such certification, and the alleged breach of TRANS-ASIA cannot be gleaned from the average adjusters survey report, or adjustment of particular average per M/V Asia Korea of the 25 October 1993 fire on board. We are not unmindful of the clear language of Sec. 74 of the Insurance Code which provides that, the violation of a material warranty, or other material provision of a policy on the part of either party thereto, entitles the other to rescind. It is generally accepted that [a] warranty is a statement or promise set forth in the policy, or by reference incorporated therein, the untruth or non-fulfillment of which in any respect, and without reference to whether the insurer was in fact prejudiced by such untruth or non-fulfillment, renders the policy voidable by the insurer.[25] However, it is similarly indubitable that for the breach of a warranty to avoid a policy, the same must be duly shown by the party alleging the same. We cannot sustain an allegation that is unfounded. Consequently, PRUDENTIAL, not having shown that TRANSASIA breached the warranty condition, CLASSED AND CLASS MAINTAINED, it remains that TRANS-ASIA must be allowed to recover its rightful claims on the policy. B. Assuming arguendo that TRANS-ASIA violated the policy condition on WARRANTED VESSEL CLASSED AND CLASS MAINTAINED, PRUDENTIAL made a valid waiver of the same.

Third, after the loss, Prudential renewed the insurance policy of Trans-Asia for two (2) consecutive years, from noon of 01 July 1994 to noon of 01 July 1995, and then again until noon of 01 July 1996. This renewal is deemed a waiver of any breach of warranty.[26]

PRUDENTIAL finds fault with the ruling of the appellate court when it ruled that the renewal policies are deemed a waiver of TRANS-ASIAs alleged breach, averring herein that the subsequent policies, designated as MH94/1595 and MH95/1788 show that they were issued only on 1 July 1994 and 3 July 1995, respectively, prior to the time it made a request to TRANS-ASIA that it be furnished a copy of the certification specifying that the insured vessel M/V Asia Korea was CLASSED AND CLASS MAINTAINED. PRUDENTIAL posits that it came to know of the breach by TRANS-ASIA of the subject warranty clause only on 21 April 1997. On even date, PRUDENTIAL sent TRANS-ASIA a letter of denial, advising the latter that their claim is not compensable. In fine, PRUDENTIAL would have this Court believe that the issuance of the renewal policies cannot be a waiver because they were issued without knowledge of the alleged breach of warranty committed by TRANSASIA.[27] We are not impressed. We do not find that the Court of Appeals was in error when it held that PRUDENTIAL, in renewing TRANS-ASIAs insurance policy for two consecutive years after the loss covered by Policy No. MH93/1363, was considered to have waived TRANS-ASIAs breach of the subject warranty, if any. Breach of a warranty or of a condition renders the contract defeasible at the option of the insurer; but if he so elects, he may waive his privilege and power to rescind by the mere expression of an intention so to do. In that event his liability under the policy continues as before.[28] There can be no clearer intention of the waiver of the alleged breach than the renewal of the policy insurance granted by PRUDENTIAL to TRANS-ASIA in MH94/1595 and MH95/1788, issued in the years 1994 and 1995, respectively. To our mind, the argument is made even more credulous by PRUDENTIALs lack of proof to support its allegation that the renewals of the policies were taken only after a request was made to TRANS-ASIA to furnish them a copy of the certificate attesting that M/V Asia Korea was CLASSED AND CLASS MAINTAINED. Notwithstanding PRUDENTIALs claim that no certification was issued to that effect, it renewed the policy, thereby, evidencing an intention to waive TRANS-

The Court of Appeals, in reversing the Judgment of the RTC which held that TRANS-ASIA breached the warranty provision on CLASSED AND CLASS MAINTAINED, underscored that PRUDENTIAL can be deemed to have made a valid waiver of TRANS-ASIAs breach of warranty as alleged, ratiocinating, thus:

ASIAs alleged breach. Clearly, by granting the renewal policies twice and successively after the loss, the intent was to benefit the insured, TRANS-ASIA, as well as to waive compliance of the warranty. The foregoing finding renders a determination of whether the subject warranty is a rider, moot, as raised by the PRUDENTIAL in its assignment of errors. Whether it is a rider will not effectively alter the result for the reasons that: (1) PRUDENTIAL was not able to discharge the burden of evidence to show that TRANS-ASIA committed a breach, thereof; and (2) assuming arguendo the commission of a breach by TRANS-ASIA, the same was shown to have been waived by PRUDENTIAL. II. A. The amount of P3,000,000.00 granted by PRUDENTIAL to TRANS- ASIA via a transaction between the parties evidenced by a document denominated as Loan and Trust Receipt, dated 29 May 1995 constituted partial payment on the policy.

promptly prosecute suit against such persons, corporation or corporations through whose negligence the aforesaid loss was caused or who may otherwise be responsible therefore, with all due diligence, in our own name, but at the expense of and under the exclusive direction and control of PRUDENTIAL GUARANTEE AND ASSURANCE INC. TRANS-ASIA SHIPPING CORPORATION[29] PRUDENTIAL largely contends that the Loan and Trust Receipt executed by the parties evidenced a loan of P3,000,000.00 which it granted to TRANS-ASIA, and not an advance payment on the policy or a partial payment for the loss. It further submits that it is a customary practice for insurance companies in this country to extend loans gratuitously as part of good business dealing with their assured, in order to afford their assured the chance to continue business without embarrassment while awaiting outcome of the settlement of their claims.[30] According to PRUDENTIAL, the Trust and Loan Agreement did not subrogate to it whatever rights and/or actions TRANS-ASIA may have against third persons, and it cannot by no means be taken that by virtue thereof, PRUDENTIAL was granted irrevocable power of attorney by TRANS-ASIA, as the sole power to prosecute lies solely with the latter. The Court of Appeals held that the real character of the transaction between the parties as evidenced by the Loan and Trust Receipt is that of an advance payment by PRUDENTIAL of TRANS-ASIAs claim on the insurance, thus:

It is undisputed that TRANS-ASIA received from PRUDENTIAL the amount of P3,000,000.00. The same was evidenced by a transaction receipt denominated as a Loan and Trust Receipt, dated 29 May 1995, reproduced hereunder: LOAN AND TRUST RECEIPT Claim File No. MH-93-025 P3,000,000.00 Check No. PCIB066755 May 29, 1995

Received FROM PRUDENTIAL GUARANTEE AND ASSURANCE INC., the sum of PESOS THREE MILLION ONLY (P3,000,000.00) as a loan without interest, under Policy No. MH93/1353, repayable only in the event and to the extent that any net recovery is made by TRANS ASIA SHIPPING CORP., from any person or persons, corporation or corporations, or other parties, on account of loss by any casualty for which they may be liable, occasioned by the 25 October 1993: Fire on Board. As security for such repayment, we hereby pledge to PRUDENTIAL GUARANTEE AND ASSURANCE INC. whatever recovery we may make and deliver to it all documents necessary to prove our interest in said property. We also hereby agree to

The Philippine Insurance Code (PD 1460 as amended) was derived from the old Insurance Law Act No. 2427 of the Philippine Legislature during the American Regime. The Insurance Act was lifted verbatim from the law of California, except Chapter V thereof, which was taken largely from the insurance law of New York. Therefore, ruling case law in that jurisdiction is to Us persuasive in interpreting provisions of our own Insurance Code. In addition, the application of the adopted statute should correspond in fundamental points with the application in its country of origin x x x. xxxx Likewise, it is settled in that jurisdiction that the (sic) notwithstanding recitals in the Loan Receipt that the money was intended as a loan does not detract from its real character as payment of claim, thus:

The receipt of money by the insured employers from a surety company for losses on account of forgery of drafts by an employee where no provision or repayment of the money was made except upon condition that it be recovered from other parties and neither interest nor security for the asserted debts was provided for, the money constituted the payment of a liability and not a mere loan, notwithstanding recitals in the written receipt that the money was intended as a mere loan. What is clear from the wordings of the so-called Loan and Trust Receipt Agreement is that appellant is obligated to hand over to appellee whatever recovery (Trans Asia) may make and deliver to (Prudential) all documents necessary to prove its interest in the said property. For all intents and purposes therefore, the money receipted is payment under the policy, with Prudential having the right of subrogation to whatever net recovery Trans-Asia may obtain from third parties resulting from the fire. In the law on insurance, subrogation is an equitable assignment to the insurer of all remedies which the insured may have against third person whose negligence or wrongful act caused the loss covered by the insurance policy, which is created as the legal effect of payment by the insurer as an assignee in equity. The loss in the first instance is that of the insured but after reimbursement or compensation, it becomes the loss of the insurer. It has been referred to as the doctrine of substitution and rests on the principle that substantial justice should be attained regardless of form, that is, its basis is the doing of complete, essential, and perfect justice between all the parties without regard to form.[31] We agree. Notwithstanding its designation, the tenor of the Loan and Trust Receipt evidences that the real nature of the transaction between the parties was that the amount of P3,000,000.00 was not intended as a loan whereby TRANS-ASIA is obligated to pay PRUDENTIAL, but rather, the same was a partial payment or an advance on the policy of the claims due to TRANS-ASIA. First, the amount of P3,000,000.00 constitutes an advance payment to TRANS-ASIA by PRUDENTIAL, subrogating the former to the extent of any net recovery made by TRANS ASIA SHIPPING CORP., from any person or persons, corporation or corporations, or other parties, on account of loss by any casualty for which they may be liable, occasioned by the 25 October 1993: Fire on Board.[32] Second, we find that per the Loan and Trust Receipt, even as TRANS-ASIA agreed to promptly prosecute suit against such persons, corporation or corporations through

whose negligence the aforesaid loss was caused or who may otherwise be responsible therefore, with all due diligence in its name, the prosecution of the claims against such third persons are to be carried on at the expense of and under the exclusive direction and control of PRUDENTIAL GUARANTEE AND ASSURANCE INC.[33] The clear import of the phrase at the expense of and under the exclusive direction and control as used in the Loan and Trust Receipt grants solely to PRUDENTIAL the power to prosecute, even as the same is carried in the name of TRANS-ASIA, thereby making TRANS-ASIA merely an agent of PRUDENTIAL, the principal, in the prosecution of the suit against parties who may have occasioned the loss. Third, per the subject Loan and Trust Receipt, the obligation of TRANS -ASIA to repay PRUDENTIAL is highly speculative and contingent, i.e., only in the event and to the extent that any net recovery is made by TRANS-ASIA from any person on account of loss occasioned by the fire of 25 October 1993. The transaction, therefore, was made to benefit TRANS-ASIA, such that, if no recovery from third parties is made, PRUDENTIAL cannot be repaid the amount. Verily, we do not think that this is constitutive of a loan.[34] The liberality in the tenor of the Loan and Trust Receipt in favor of TRANS-ASIA leads to the conclusion that the amount of P3,000,000.00 was a form of an advance payment on TRANS-ASIAs claim on MH93/1353. III. A. PRUDENTIAL is directed to pay TRANS-ASIA the amount of P8,395,072.26, representing the balance of the loss suffered by TRANS-ASIA and covered by Marine Policy No. MH93/1363.

Our foregoing discussion supports the conclusion that TRANS-ASIA is entitled to the unpaid claims covered by Marine Policy No. MH93/1363, or a total amount of P8,395,072.26. B. Likewise, PRUDENTIAL is directed to pay TRANS-ASIA, damages in the form of attorneys fees equivalent to 10% of P8,395,072.26. The Court of Appeals denied the grant of attorneys fees. It held that attorneys fees cannot be awarded absent a showing of bad faith on the part of PRUDENTIAL in

rejecting TRANS-ASIAs claim, notwithstanding that the rejection was erroneous. According to the Court of Appeals, attorneys fees can be awarded only in the cases enumerated in Article 2208 of the Civil Code which finds no application in the instant case. We disagree. Sec. 244 of the Insurance Code grants damages consisting of attorneys fees and other expenses incurred by the insured after a finding by the Insurance Commissioner or the Court, as the case may be, of an unreasonable denial or withholding of the payment of the claims due. Moreover, the law imposes an interest of twice the ceiling prescribed by the Monetary Board on the amount of the claim due the insured from the date following the time prescribed in Section 242[35] or in Section 243,[36] as the case may be, until the claim is fully satisfied. Finally, Section 244 considers the failure to pay the claims within the time prescribed in Sections 242 or 243, when applicable, as prima facie evidence of unreasonable delay in payment. To the mind of this Court, Section 244 does not require a showing of bad faith in order that attorneys fees be granted. As earlier stated, under Section 244, a prima facie evidence of unreasonable delay in payment of the claim is created by failure of the insurer to pay the claim within the time fixed in both Sections 242 and 243 of the Insurance Code. As established in Section 244, by reason of the delay and the consequent filing of the suit by the insured, the insurers shall be adjudged to pay damages which shall consist of attorneys fees and other expenses incurred by the insured.[37] Section 244 reads: In case of any litigation for the enforcement of any policy or contract of insurance, it shall be the duty of the Commissioner or the Court, as the case may be, to make a finding as to whether the payment of the claim of the insured has been unreasonably denied or withheld; and in the affirmative case, the insurance company shall be adjudged to pay damages which shall consist of attorneys fees and other expenses incurred by the insured person by reason of such unreasonable denial or withholding of payment plus interest of twice the ceiling prescribed by the Monetary Board of the amount of the claim due the insured, from the date following the time prescribed in section two hundred forty-two or in section two hundred forty-three, as the case may be, until the claim is fully satisfied; Provided, That the failure to pay any such claim within the time prescribed in said sections shall be considered prima facie evidence of unreasonable delay in payment.

Sections 243 and 244 of the Insurance Code apply when the court finds an unreasonable delay or refusal in the payment of the insurance claims. In the case at bar, the facts as found by the Court of Appeals, and confirmed by the records show that there was an unreasonable delay by PRUDENTIAL in the payment of the unpaid balance of P8,395,072.26 to TRANS-ASIA. On 26 October 1993, a day after the occurrence of the fire in M/V Asia Korea, TRANS-ASIA filed its notice of claim. On 13 August 1996, the adjuster, Richards Hogg International (Phils.), Inc., completed its survey report recommending the amount of P11,395,072.26 as the total indemnity due to TRANS-ASIA.[38] On 21 April 1997, PRUDENTIAL, in a letter[39] addressed to TRANS-ASIA denied the latters claim for the amount of P8,395,072.26 representing the balance of the total indemnity. On 21 July 1997, PRUDENTIAL sent a second letter[40] to TRANS-ASIA seeking a return of the amount of P3,000,000.00. On 13 August 1997, TRANS-ASIA was constrained to file a complaint for sum of money against PRUDENTIAL praying, inter alia, for the sum of P8,395,072.26 representing the balance of the proceeds of the insurance claim. As can be gleaned from the foregoing, there was an unreasonable delay on the part of PRUDENTIAL to pay TRANS-ASIA, as in fact, it refuted the latters right to the insurance claims, from the time proof of loss was shown and the ascertainment of the loss was made by the insurance adjuster. Evidently, PRUDENTIALs unreasonable delay in satisfying TRANS-ASIAs unpaid claims compelled the latter to file a suit for collection. Succinctly, an award equivalent to ten percent (10%) of the unpaid proceeds of the policy as attorneys fees to TRANS-ASIA is reasonable under the circumstances, or otherwise stated, ten percent (10%) of P8,395,072.26. In the case of Cathay Insurance, Co., Inc. v. Court of Appeals,[41] where a finding of an unreasonable delay under Section 244 of the Insurance Code was made by this Court, we grant an award of attorneys fees equivalent to ten percent (10%) of the total proceeds. We find no reason to deviate from this judicial precedent in the case at bar. C. Further, the aggregate amount (P8,395,072.26 plus 10% thereof as attorneys fees) shall be imposed double interest in accordance with Section 244 of the Insurance Code.

Section 244 of the Insurance Code is categorical in imposing an interest twice the ceiling prescribed by the Monetary Board due the insured, from the date following the time prescribed in Section 242 or in Section 243, as the case may be, until the claim is fully satisfied. In the case at bar, we find Section 243 to be applicable as what is involved herein is a marine insurance, clearly, a policy other than life insurance. Section 243 is hereunder reproduced: SEC. 243. The amount of any loss or damage for which an insurer may be liable, under any policy other than life insurance policy, shall be paid within thirty days after proof of loss is received by the insurer and ascertainment of the loss or damage is made either by agreement between the insured and the insurer or by arbitration; but if such ascertainment is not had or made within sixty days after such receipt by the insurer of the proof of loss, then the loss or damage shall be paid within ninety days after such receipt. Refusal or failure to pay the loss or damage within the time prescribed herein will entitle the assured to collect interest on the proceeds of the policy for the duration of the delay at the rate of twice the ceiling prescribed by the Monetary Board, unless such failure or refusal to pay is based on the ground that the claim is fraudulent.

The contention fails to persuade. It is settled that an award of double interest is lawful and justified under Sections 243 and 244 of the Insurance Code.[43] In Finman General Assurance Corporation v. Court of Appeals,[44] this Court held that the payment of 24% interest per annum is authorized by the Insurance Code.[45] There is no gainsaying that the term double interest as used in Sections 243 and 244 can only be interpreted to mean twice 12% per annum or 24% per annum interest, thus: The term ceiling prescribed by the Monetary Board means the legal rate of interest of twelve per centum per annum (12%) as prescribed by the Monetary Board in C.B. Circular No. 416, pursuant to P.D. No. 116, amending the Usury Law; so that when Sections 242, 243 and 244 of the Insurance Code provide that the insurer shall be liable to pay interest twice the ceiling prescribed by the Monetary Board, it means twice 12% per annum or 24% per annum interest on the proceeds of the insurance.[46]

E.

The payment of double interest should be counted from 13 September 1996.

As specified, the assured is entitled to interest on the proceeds for the duration of the delay at the rate of twice the ceiling prescribed by the Monetary Board except when the failure or refusal of the insurer to pay was founded on the ground that the claim is fraudulent. D. The term double interest as used in the Decision of the Court of Appeals must be interpreted to mean 24% per annum.

The Court of Appeals, in imposing double interest for the duration of the delay of the payment of the unpaid balance due TRANS-ASIA, computed the same from 13 August 1996 until such time when the amount is fully paid. Although not raised by the parties, we find the computation of the duration of the delay made by the appellate court to be patently erroneous. To be sure, Section 243 imposes interest on the proceeds of the policy for the duration of the delay at the rate of twice the ceiling prescribed by the Monetary Board. Significantly, Section 243 mandates the payment of any loss or damage for which an insurer may be liable, under any policy other than life insurance policy, within thirty days after proof of loss is received by the insurer and ascertainment of the loss or damage is made either by agreement between the insured and the insurer or by arbitration. It is clear that under Section 243, the insurer has until the 30th day after proof of loss and ascertainment of the loss or damage to pay its liability under the insurance, and only after such time can the insurer be held to be in delay, thereby necessitating the imposition of double interest.

PRUDENTIAL assails the award of interest, granted by the Court of Appeals, in favor of TRANS-ASIA in the assailed Decision of 6 November 2001. It is PRUDENTIALs stance that the award is extortionate and grossly unsconscionable. In support thereto, PRUDENTIAL makes a reference to TRANS-ASIAs prayer in the Complaint filed with the court a quo wherein the latter sought, interest double the prevailing rate of interest of 21% per annum now obtaining in the banking business or plus 42% per annum pursuant to Article 243 of the Insurance Code x x x.[42]

In the case at bar, it was not disputed that the survey report on the ascertainment of the loss was completed by the adjuster, Richard Hoggs International (Phils.), Inc. on 13 August 1996. PRUDENTIAL had thirty days from 13 August 1996 within which to pay its liability to TRANS-ASIA under the insurance policy, or until 13 September 1996. Therefore, the double interest can begin to run from 13 September 1996 only. IV. A. An interest of 12% per annum is similarly imposed on the TOTAL amount of liability adjudged in section III herein, computed from the time of finality of judgment until the full satisfaction thereof in conformity with this Courts ruling in Eastern Shipping Lines, Inc. v. Court of Appeals.

1. PRUDENTIAL is DIRECTED to PAY TRANS-ASIA the amount of P8,395,072.26, representing the balance of the loss suffered by TRANS-ASIA and covered by Marine Policy No. MH93/1363; 2. PRUDENTIAL is DIRECTED further to PAY TRANS-ASIA damages in the form of attorneys fees equivalent to 10% of the amount of P8,395,072.26; 3. The aggregate amount (P8,395,072.26 plus 10% thereof as attorneys fees) shall be imposed double interest at the rate of 24% per annum to be computed from 13 September 1996 until fully paid; and 4. An interest of 12% per annum is similarly imposed on the TOTAL amount of liability adjudged as abovestated in paragraphs (1), (2), and (3) herein, computed from the time of finality of judgment until the full satisfaction thereof. No costs. SO ORDERED.

This Court in Eastern Shipping Lines, Inc. v. Court of Appeals,[47] inscribed the rule of thumb[48] in the application of interest to be imposed on obligations, regardless of their source. Eastern emphasized beyond cavil that when the judgment of the court awarding a sum of money becomes final and executory, the rate of legal interest, regardless of whether the obligation involves a loan or forbearance of money, shall be 12% per annum from such finality until its satisfaction, this interim period being deemed to be by then an equivalent to a forbearance[49] of credit. We find application of the rule in the case at bar proper, thus, a rate of 12% per annum from the finality of judgment until the full satisfaction thereof must be imposed on the total amount of liability adjudged to PRUDENTIAL. It is clear that the interim period from the finality of judgment until the satisfaction of the same is deemed equivalent to a forbearance of credit, hence, the imposition of the aforesaid interest. WHEREFORE, the Petition in G.R. No. 151890 is DENIED. However, the Petition in G.R. No. 151991 is GRANTED, thus, we award the grant of attorneys fees and make a clarification that the term double interest as used in the 6 November 2001 Decision of the Court of Appeals in CA GR CV No. 68278 should be construed to mean interest at the rate of 24% per annum, with a further clarification, that the same should be computed from 13 September 1996 until fully paid. The Decision and Resolution of the Court of Appeals, in CA-G.R. CV No. 68278, dated 6 November 2001 and 29 January 2002, respectively, are, thus, MODIFIED in the following manner, to wit:

G.R. No. L-20853

May 29, 1967

BONIFACIO BROS. v. MORA This is an appeal from the decision of the Court of First Instance of Manila, Branch XV, in civil case 48823, affirming the decision of the Municipal Court of Manila, declaring the H.S. Reyes, Inc. as having a better right than the Bonifacio Bros., Inc. and the Ayala Auto Parts Company, appellants herein, to the proceeds of motor insurance policy A-0615, in the sum of P2,002.73, issued by the State Bonding & Insurance Co. Inc., and directing payment of the said amount to the H. Reyes, Inc. Enrique Mora, owner of Oldsmobile sedan model 1956, bearing plate No. QCmortgaged the same to the H.S. Reyes, Inc., with the condition that the former would insure the automobile with the latter as beneficiary. The automobile was thereafter insured on June 23, 1959 with the State Bonding & Insurance Co., Inc., and motor car insurance policy A-0615 was issued to Enrique Mora, the pertinent provisions of which read: 1. The Company (referring to the State Bonding & Insurance Co., Inc.) will, subject to the Limits of Liability, indemnify the Insured against loss of or damages to the Motor Vehicle and its accessories and spare parts whilst thereon; (a) by accidental

collision or overturning or collision or overturning consequent upon mechanical breakdown or consequent upon wear and tear, xxx xxx xxx

2. At its own option the Company may pay in cash the amount of the loss or damage or may repair, reinstate, or replace the Motor Vehicle or any part thereof or its accessories or spare parts. The liability of the Company shall not exceed the value of the parts whichever is the less. The Insured's estimate of value stated in the schedule will be the maximum amount payable by the Company in respect of any claim for loss or damage.1wph1.t xxx xxx xxx

4. The Insured may authorize the repair of the Motor Vehicle necessitated by damage for which the Company may be liable under this Policy provided that: (a) The estimated cost of such repair does not exceed the Authorized Repair Limit, (b) A detailed estimate of the cost is forwarded to the Company without delay, subject to the condition that "Loss, if any is payable to H.S. Reyes, Inc.," by virtue of the fact that said Oldsmobile sedan was mortgaged in favor of the said H.S. Reyes, Inc. and that under a clause in said insurance policy, any loss was made payable to the H.S. Reyes, Inc. as Mortgagee; xxx xxx xxx

Upon the theory that the insurance proceeds should be paid directly to them, the Bonifacio Bros. Inc. and the Ayala Auto Parts Co. filed on May 8, 1961 a complaint with the Municipal Court of Manila against Enrique Mora and the State Bonding & Insurance Co., Inc. for the collection of the sum of P2,002.73 The insurance company filed its answer with a counterclaim for interpleader, requiring the Bonifacio Bros. Inc. and the H.S. Reyes, Inc. to interplead in order to determine who has better right to the insurance proceeds in question. Enrique Mora was declared in default for failure to appear at the hearing, and evidence against him was received ex parte. However, the counsel for the Bonifacio Bros. Inc., Ayala Auto Parts Co. and State Bonding & Insurance Co. Inc. submitted a stipulation of facts, on the basis of which are Municipal Court rendered a decision declaring the H.S. Reyes, Inc. as having a better right to the disputed amount and ordering State Bonding & Insurance Co. Inc. to pay to the H. S. Reyes, Inc. the said sum of P2,002.73. From this decision, the appellants elevated the case to the Court of First Instance of Manila which the stipulation of facts was reproduced. On October 19, 1962 the latter court rendered a decision, affirming the decision of the Municipal Court. The Bonifacio Bros. Inc. and the Ayala Auto Parts Co. moved for reconsideration of the decision, but the trial court denied the motion. Hence, this appeal. The main issue raised is whether there is privity of contract between the Bonifacio Bros. Inc. and the Ayala Auto Parts Co. on the one hand and the insurance company on the other. The appellants argue that the insurance company and Enrique Mora are parties to the repair of the car as well as the towage thereof performed. The authority for this assertion is to be found, it is alleged, in paragraph 4 of the insurance contract which provides that "the insured may authorize the repair of the Motor Vehicle necessitated by damage for which the company may be liable under the policy provided that (a) the estimated cost of such repair does not exceed the Authorized Repair Limit, and (b) a detailed estimate of the cost is forwarded to the company without delay." It is stressed that the H.H. Bayne Adjustment Company's recommendation of payment of the appellants' bill for materials and repairs for which the latter drew a check for P2,002.73 indicates that Mora and the H.H. Bayne Adjustment Co. acted for and in representation of the insurance company. This argument is, in our view, beside the point, because from the undisputed facts and from the pleadings it will be seen that the appellants' alleged cause of action rests exclusively upon the terms of the insurance contract. The appellants seek to recover the insurance proceeds, and for this purpose, they rely upon paragraph 4 of the insurance contract document executed by and between the State Bonding & Insurance Company, Inc. and Enrique Mora. The appellants are not mentioned in the contract as

During the effectivity of the insurance contract, the car met with an accident. The insurance company then assigned the accident to the Bayne Adjustment Co. for investigation and appraisal of the damage. Enrique Mora, without the knowledge and consent of the H.S. Reyes, Inc., authorized the Bonifacio Bros. Inc. to furnish the labor and materials, some of which were supplied by the Ayala Auto Parts Co. For the cost of labor and materials, Enrique Mora was billed at P2,102.73 through the H.H. Bayne Adjustment Co. The insurance company after claiming a franchise in the amount of P100, drew a check in the amount of P2,002.73, as proceeds of the insurance policy, payable to the order of Enrique Mora or H.S. Reyes,. Inc., and entrusted the check to the H.H. Bayne Adjustment Co. for disposition and delivery to the proper party. In the meantime, the car was delivered to Enrique Mora without the consent of the H.S. Reyes, Inc., and without payment to the Bonifacio Bros. Inc. and the Ayala Auto Parts Co. of the cost of repairs and materials.

parties thereto nor is there any clause or provision thereof from which we can infer that there is an obligation on the part of the insurance company to pay the cost of repairs directly to them. It is fundamental that contracts take effect only between the parties thereto, except in some specific instances provided by law where the contract contains some stipulation in favor of a third person.1 Such stipulation is known as stipulation pour autrui or a provision in favor of a third person not a pay to the contract. Under this doctrine, a third person is allowed to avail himself of a benefit granted to him by the terms of the contract, provided that the contracting parties have clearly and deliberately conferred a favor upon such person.2 Consequently, a third person not a party to the contract has no action against the parties thereto, and cannot generally demand the enforcement of the same.3 The question of whether a third person has an enforcible interest in a contract, must be settled by determining whether the contracting parties intended to tender him such an interest by deliberately inserting terms in their agreement with the avowed purpose of conferring a favor upon such third person. In this connection, this Court has laid down the rule that the fairest test to determine whether the interest of a third person in a contract is a stipulation pour autrui or merely an incidental interest, is to rely upon the intention of the parties as disclosed by their contract.4 In the instant case the insurance contract does not contain any words or clauses to disclose an intent to give any benefit to any repairmen or materialmen in case of repair of the car in question. The parties to the insurance contract omitted such stipulation, which is a circumstance that supports the said conclusion. On the other hand, the "loss payable" clause of the insurance policy stipulates that "Loss, if any, is payable to H.S. Reyes, Inc." indicating that it was only the H.S. Reyes, Inc. which they intended to benefit. We likewise observe from the brief of the State Bonding & Insurance Company that it has vehemently opposed the assertion or pretension of the appellants that they are privy to the contract. If it were the intention of the insurance company to make itself liable to the repair shop or materialmen, it could have easily inserted in the contract a stipulation to that effect. To hold now that the original parties to the insurance contract intended to confer upon the appellants the benefit claimed by them would require us to ignore the indespensable requisite that a stipulation pour autrui must be clearly expressed by the parties, which we cannot do. As regards paragraph 4 of the insurance contract, a perusal thereof would show that instead of establishing privity between the appellants and the insurance company, such stipulation merely establishes the procedure that the insured has to follow in order to be entitled to indemnity for repair. This paragraph therefore should not be

construed as bringing into existence in favor of the appellants a right of action against the insurance company as such intention can never be inferred therefrom. Another cogent reason for not recognizing a right of action by the appellants against the insurance company is that "a policy of insurance is a distinct and independent contract between the insured and insurer, and third persons have no right either in a court of equity, or in a court of law, to the proceeds of it, unless there be some contract of trust, expressed or implied between the insured and third person."5 In this case, no contract of trust, expressed or implied exists. We, therefore, agree with the trial court that no cause of action exists in favor of the appellants in so far as the proceeds of insurance are concerned. The appellants' claim, if at all, is merely equitable in nature and must be made effective through Enrique Mora who entered into a contract with the Bonifacio Bros. Inc. This conclusion is deducible not only from the principle governing the operation and effect of insurance contracts in general, but is clearly covered by the express provisions of section 50 of the Insurance Act which read: The insurance shall be applied exclusively to the proper interests of the person in whose name it is made unless otherwise specified in the policy. The policy in question has been so framed that "Loss, if any, is payable to H.S. Reyes, Inc.," which unmistakably shows the intention of the parties. The final contention of the appellants is that the right of the H.S. Reyes, Inc. to the insurance proceeds arises only if there was loss and not where there is mere damage as in the instant case. Suffice it to say that any attempt to draw a distinction between "loss" and "damage" is uncalled for, because the word "loss" in insurance law embraces injury or damage. Loss in insurance, defined. The injury or damage sustained by the insured in consequence of the happening of one or more of the accidents or misfortune against which the insurer, in consideration of the premium, has undertaken to indemnify the insured. (1 Bouv. Ins. No. 1215; Black's Law Dictionary; Cyclopedic Law Dictionary, cited in Martin's Phil. Commercial Laws, Vol. 1, 1961 ed. p. 608). Indeed, according to sec. 120 of the Insurance Act, a loss may be either total or partial. Accordingly, the judgment appealed from is hereby affirmed, at appellants' cost.

INSULAR LIFE VS. EBRADO 80 SCRA 181 Facts: > Buenaventura Ebrado was issued al life plan by Insular Company. He designated Capriona as his beneficiary, referring to her as his wife. > The insured then died and Carponia tried to claim the proceeds of the said plan. > She admitted to being only the common law wife of the insured. > Pascuala, the legal wife, also filed a claim asserting her right as the legal wife. The company then filed an action for interpleader. Issue: Whether or not the common law wife named as beneficiary can collect the proceeds. Held: NO. The civil code prohibitions on donations made between persons guilty of adulterous concubinage applies to insurance contracts. On matters not specifically provided for by the Insurance Law, the general rules on Civil law shall apply. A life insurance policy is no different from a civil donation as far as the beneficiary is concerned, since both are founded on liberality. VDA. DE CONSUEGRA V. GSIS - RETIREMENT INSURANCE BENEFITS 37 SCRA 315 Facts: > Jose Consuegra was employed as a shop foreman of the Office of the District Engineer in Surigao Del Norte. > When he was still alive, he contracted two marriages: o First Rosario Diaz; 2 children = Jose Consuegra Jr. and Pedro but both predeceased him o 2nd Basilia Berdin; 7 children. (this was contracted in GF while the first marriage subsisted)

> Being a GSIS member when he died, the proceeds of his life insurance were paid by the GSIS to Berdin and her children who were the beneficiaries named in the policy. > Since he was in the govt service for 22.5028 years, he was entitled to retirement insurance benefits, for which no beneficiary was designated. > Both families filed their claims with the GSIS, which ruled that the legal heirs were Diaz who is entitled to one-half or 8/16 of the retirement benefits and Berdin and her children were entitled to the remaining half, each to receive an equal share of 1/16. > Berdin went to CFI on appeal. CFI affirmed GSIS decision.

Issue: To whom should the retirement insurance benefits be paid?

Held: Both families are entitled to half of the retirement benefits. The beneficiary named in the life insurance does NOT automatically become the beneficiary in the retirement insurance. When Consuegra, during the early part of 1943, or before 1943, designated his beneficiaries in his life insurance, he could NOT have intended those beneficiaries of his life insurance as also the beneficiaries of his retirement insurance because the provisions on retirement insurance under the GSIS came about only when CA 186 was amended by RA 660 on June 18, 1951.

Sec. 11(b) clearly indicates that there is need for the employee to file an application for retirement insurance benefits when he becomes a GSIS member and to state his beneficiary. The life insurance and the retirement insurance are two separate and distinct systems of benefits paid out from 2 separate and distinct funds.

In case of failure to name a beneficiary in an insurance policy, the proceeds will accrue to the estate of the insured. And when there exists two marriages, each family will be entitled to one-half of the estate.

resulting in the total loss of the respondents stocks-in-trade, pieces of furnitures and fixtures, equipments and records.

COUNTRY BANKERS INSURANC, vs. LIANGA BAY Before us is a petition for review on certiorari of the Decision[1] of the Court of Appeals[2] dated December 29, 1998 in CA-G.R. CV Case No. 36902 affirming in toto the Decision[3] dated December 26, 1991 of the Regional Trial Court of Lianga, Surigao del Sur, Branch 28, in Civil Case No. L-518 which ordered petitioner Country Bankers Insurance Corporation to fully pay the insurance claim of respondent Lianga Bay and Community Multi-Purpose Cooperative, Inc., under Fire Insurance Policy No. F-1397, for loss sustained as a result of the fire that occurred on July 1, 1989 in the amount of Two Hundred Thousand Pesos (P200,000.00), with interest at twelve percent (12%) per annum from the date of filing of the complaint until fully paid, as well as Fifty Thousand Pesos (P50,000.00) as actual damages, Fifty Thousand Pesos (P50,000.00) as exemplary damages, Five Thousand Pesos (P5,000.00) as litigation expenses, Ten Thousand Pesos (P10,000.00) as attorneys fees, and the costs of suit. The facts are undisputed: The petitioner is a domestic corporation principally engaged in the insurance business wherein it undertakes, for a consideration, to indemnify another against loss, damage or liability from an unknown or contingent event including fire while the respondent is a duly registered cooperative judicially declared insolvent and represented by the elected assignee, Cornelio Jamero.

Due to the loss, the respondent filed an insurance claim with the petitioner under its Fire Insurance Policy No. F-1397, submitting: (a) the Spot Report of Pfc. Arturo V. Juarbal, INP Investigator, dated July 1, 1989; (b) the Sworn Statement of Jose Lomocso; and (c) the Sworn Statement of Ernesto Urbiztondo.

The petitioner, however, denied the insurance claim on the ground that, based on the submitted documents, the building was set on fire by two (2) NPA rebels who wanted to obtain canned goods, rice and medicines as provisions for their comrades in the forest, and that such loss was an excepted risk under paragraph No. 6 of the policy conditions of Fire Insurance Policy No. F-1397, which provides:

This insurance does not cover any loss or damage occasioned by or through or in consequence, directly or indirectly, of any of the following occurrences, namely: (d) Mutiny, riot, military or popular uprising, insurrection, rebellion, revolution, military or usurped power.

It appears that sometime in 1989, the petitioner and the respondent entered into a contract of fire insurance. Under Fire Insurance Policy No. F-1397, the petitioner insured the respondents stocks-in-trade against fire loss, damage or liability during the period starting from June 20, 1989 at 4:00 p.m. to June 20, 1990 at 4:00 p.m., for the sum of Two Hundred Thousand Pesos (P200,000.00). On July 1, 1989, at or about 12:40 a.m., the respondents building located at Barangay Diatagon, Lianga, Surigao del Sur was gutted by fire and reduced to ashes,

Any loss or damage happening during the existence of abnormal conditions (whether physical or otherwise) which are occasioned by or through or in consequence, directly or indirectly, of any of said occurrences shall be deemed to be loss or damage which is not covered by this insurance, except to the extent that the Insured shall prove that such loss or damage happened independently of the existence of such abnormal conditions.

Finding the denial of its claim unacceptable, the respondent then instituted in the trial court the complaint for recovery of loss, damage or liability against petitioner. The petitioner answered the complaint and reiterated the ground it earlier cited to deny the insurance claim, that is, that the loss was due to NPA rebels, an excepted risk under the fire insurance policy.

In due time, the trial court rendered its Decision dated December 26, 1991 in favor of the respondent, declaring that:

For being unsubstantiated with credible and positive evidence, the counterclaim is dismissed.

Based on its findings, it is therefore the considered opinion of this Court, as it so holds, that the defenses raised by defendant-Country Bankers has utterly crumbled on account of its inherent weakness, incredibility and unreliability, and after applying those helpful tools like common sense, logic and the Courts honest appraisal of the real and actual situation obtaining in this area, such defenses remains (sic) unimpressive and unconvincing, and therefore, the defendant-Country Bankers has to be irreversibly adjudged liable, as it should be, to plaintiff-Insolvent Cooperative, represented in this action by its Assignee, Cornelio Jamero, and thus, ordering said defendant-Country Bankers to pay the plaintiff-Insolvent Cooperative, as follows:

IT IS SO ORDERED.

Petitioner interposed an appeal to the Court of Appeals. On December 29, 1998, the appellate court affirmed the challenged decision of the trial court in its entirety. Petitioner now comes before us via the instant petition anchored on three (3) assigned errors,[4] to wit:

1. To fully pay the insurance claim for the loss the insured-plaintiff sustained as a result of the fire under its Fire Insurance Policy No. F-1397 in its full face value of P200,000.00 with interest of 12% per annum from date of filing of the complaint until the same is fully paid;

1. THE HONORABLE COURT OF APPEALS FAILED TO APPRECIATE AND GIVE CREDENCE TO THE SPOT REPORT OF PFC. ARTURO JUARBAL (EXH. 3) AND THE SWORN STATEMENT OF JOSE LOMOCSO (EXH. 4) THAT THE RESPONDENTS STOCK-IN-TRADE WAS BURNED BY THE NPA REBELS, HENCE AN EXCEPTED RISK UNDER THE FIRE INSURANCE POLICY.

2. To pay as and in the concept of actual or compensatory damages in the total sum of P50,000.00;

2. THE HONORABLE COURT OF APPEALS ERRED IN HOLDING PETITIONER LIABLE FOR 12% INTEREST PER ANNUM ON THE FACE VALUE OF THE POLICY FROM THE FILING OF THE COMPLAINT UNTIL FULLY PAID.

3. To pay as and in the concept of exemplary damages in the total sum of P50,000.00; 3. THE HONORABLE COURT OF APPEALS ERRED IN HOLDING THE PETITIONER LIABLE FOR ACTUAL AND EXEMPLARY DAMAGES, LITIGATION EXPENSES, ATTORNEYS FEES AND COST OF SUIT.

4. To pay in the concept of litigation expenses the sum of P5,000.00;

5. To pay by way of reimbursement the attorneys fees in the sum of P10,000.00; and

6. To pay the costs of the suit.

A party is bound by his own affirmative allegations. This is a well-known postulate echoed in Section 1 of Rule 131 of the Revised Rules of Court. Each party must prove his own affirmative allegations by the amount of evidence required by law which in civil cases, as in this case, is preponderance of evidence, to obtain a favorable judgment.[5]

In the instant case, the petitioner does not dispute that the respondents stocks-intrade were insured against fire loss, damage or liability under Fire Insurance Policy No. F- 1397 and that the respondent lost its stocks-in-trade in a fire that occurred on July 1, 1989, within the duration of said fire insurance. The petitioner, however, posits the view that the cause of the loss was an excepted risk under the terms of the fire insurance policy.

was told or read or heard the same. Such testimony is considered hearsay and may not be received as proof of the truth of what he has learned. Such is the hearsay rule which applies not only to oral testimony or statements but also to written evidence as well.[9]

Where a risk is excepted by the terms of a policy which insures against other perils or hazards, loss from such a risk constitutes a defense which the insurer may urge, since it has not assumed that risk, and from this it follows that an insurer seeking to defeat a claim because of an exception or limitation in the policy has the burden of proving that the loss comes within the purview of the exception or limitation set up. If a proof is made of a loss apparently within a contract of insurance, the burden is upon the insurer to prove that the loss arose from a cause of loss which is excepted or for which it is not liable, or from a cause which limits its liability.[6] Stated elsewise, since the petitioner in this case is defending on the ground of non-coverage and relying upon an exemption or exception clause in the fire insurance policy, it has the burden of proving the facts upon which such excepted risk is based, by a preponderance of evidence.[7] But petitioner failed to do so.

The hearsay rule is based upon serious concerns about the trustworthiness and reliability of hearsay evidence inasmuch as such evidence are not given under oath or solemn affirmation and, more importantly, have not been subjected to crossexamination by opposing counsel to test the perception, memory, veracity and articulateness of the out-of-court declarant or actor upon whose reliability on which the worth of the out-of-court statement depends.[10]

Thus, the Sworn Statements of Jose Lomocso and Ernesto Urbiztondo are inadmissible in evidence, for being hearsay, inasmuch as they did not take the witness stand and could not therefore be cross-examined.

There are exceptions to the hearsay rule, among which are entries in official records.[11] To be admissible in evidence, however, three (3) requisites must concur, to wit:

The petitioner relies on the Sworn Statements of Jose Lomocso and Ernesto Urbiztondo as well as on the Spot Report of Pfc. Arturo V. Juarbal dated July 1, 1989, more particularly the following statement therein:

(a) that the entry was made by a public officer, or by another person specially enjoined by law to do so;

xxx investigation revealed by Jose Lomocso that those armed men wanted to get can goods and rice for their consumption in the forest PD investigation further disclosed that the perpetrator are member (sic) of the NPA PD end x x x

(b) that it was made by the public officer in the performance of his duties, or by such other person in the performance of a duty specially enjoined by law; and

A witness can testify only to those facts which he knows of his personal knowledge, which means those facts which are derived from his perception.[8] Consequently, a witness may not testify as to what he merely learned from others either because he

(c) that the public officer or other person had sufficient knowledge of the facts by him stated, which must have been acquired by him personally or through official information.[12]

The third requisite was not met in this case since no investigation, independent of the statements gathered from Jose Lomocso, was conducted by Pfc. Arturo V. Juarbal. In fact, as the petitioner itself pointed out, citing the testimony of Pfc. Arturo Juarbal,[13] the latters Spot Report was based on the personal knowledge of the caretaker Jose Lomocso who witnessed every single incident surrounding the facts and circumstances of the case. This argument undeniably weakens the petitioners defense, for the Spot Report of Pfc. Arturo Juarbal relative to the statement of Jose Lomocso to the effect that NPA rebels allegedly set fire to the respondents building is inadmissible in evidence, for the purpose of proving the truth of the statements contained in the said report, for being hearsay.

damages. The provisions under Title XVIII on Damages of the Civil Code govern in determining the measure of recoverable damages.

II. With regard particularly to an award of interest in the concept of actual and compensatory damages, the rate of interest, as well as the accrual thereof, is imposed, as follows:

The said Spot Report is admissible only insofar as it constitutes part of the testimony of Pfc. Arturo V. Juarbal since he himself took the witness stand and was available for cross-examination. The portions of his Spot Report which were of his personal knowledge or which consisted of his perceptions and conclusions are not hearsay. The rest of the said report relative to the statement of Jose Lomocso may be considered as independently relevant statements gathered in the course of Juarbals investigation and may be admitted as such but not necessarily to prove the truth thereof. [14]

1. When the obligation is breached, and it consists in the payment of a sum of money, i.e., a loan or forbearance of money, the interest due should be that which may have been stipulated in writing. Furthermore, the interest due shall itself earn legal interest from the time it is judicially demanded. In the absence of stipulation, the rate of interest shall be 12% per annum to be computed from default, i.e., from judicial or extrajudicial demand under and subject to the provisions of Article 1169 of the Civil Code.

The petitioners evidence to prove its defense is sadly wanting and thus, gives rise to its liability to the respondent under Fire Insurance Policy No. F-1397. Nonetheless, we do not sustain the trial courts imposition of twelve percent (12%) interest on the insurance claim as well as the monetary award for actual and exemplary damages, litigation expenses and attorneys fees for lack of legal and valid basis.

2. When an obligation, not constituting a loan or forbearance of money, is breached, an interest on the amount of damages awarded may be imposed at the discretion of the court at the rate of 6% per annum. No interest, however, shall be adjudged on unliquidated claims or damages except when or until the demand can be established with reasonable certainty. Accordingly, where the demand is established with reasonable certainty, the interest shall begin to run from the time the claim is made judicially or extrajudicially (Art. 1169, Civil Code) but when such certainty cannot be so reasonably established at the time the demand is made, the interest shall begin to run only from the date the judgment of the court is made (at which time the quantification of damages may be deemed to have been reasonably ascertained). The actual base for the computation of legal interest shall, in any case, be on the amount finally adjudged.

Concerning the application of the proper interest rates, the following guidelines were set in Eastern Shipping Lines, Inc. v. Court of Appeals and Mercantile Insurance Co., Inc.:[15] I. When an obligation, regardless of its source, i.e., law, contracts, quasicontracts, delicts or quasi-delicts, is breached, the contravenor can be held liable for

3. When the judgment of the court awarding a sum of money becomes final and executory, the rate of legal interest, whether the case falls under paragraph 1 or paragraph 2, above, shall be 12% per annum from such finality until its satisfaction, this interim period being deemed to be by then an equivalent to a forbearance of credit.

In the said case of Eastern Shipping, the Court further observed that a forbearance in the context of the usury law is a contractual obligation of lender or creditor to refrain, during a given period of time, from requiring the borrower or debtor to repay a loan or debt then due and payable.

Considering the foregoing, the insurance claim in this case is evidently not a forbearance of money, goods or credit, and thus the interest rate should be as it is hereby fixed at six percent (6%) computed from the date of filing of the complaint.

With respect to the award of litigation expenses and attorneys fees, Article 2208 of the New Civil Code[17] enumerates the instances where such may be awarded and, in all cases, it must be reasonable, just and equitable if the same were to be granted. Attorneys fees as part of damages are not meant to enrich the winning party at the expense of the losing litigant. They are not awarded every time a party prevails in a suit because of the policy that no premium should be placed on the right to litigate.[18] The award of attorneys fees is the exception rather than the general rule. As such, it is necessary for the court to make findings of facts and law that would bring the case within the exception and justify the grant of such award. We find none in this case to warrant the award by the trial court of litigation expenses and attorneys fees in the amounts of Five Thousand Pesos (P5,000.00) and Ten Thousand Pesos (P10,000.00), respectively, and therefore, the same must also be deleted.

We find no justification for the award of actual damages of Fifty Thousand Pesos (P50,000.00). Well-entrenched is the doctrine that actual, compensatory and consequential damages must be proved, and cannot be presumed.[16] That part of the dispositive portion of the Decision of the trial court ordering the petitioner to pay actual damages of Fifty Thousand Pesos (P50,000.00) has no basis at all. The justification, if any, for such an award of actual damages does not appear in the body of the decision of the trial court. Neither is there any testimonial and documentary evidence on the alleged actual damages of Fifty Thousand Pesos (P50,000.00) to warrant such an award. Thus, the same must be deleted.

WHEREFORE, the appealed Decision is MODIFIED. The rate of interest on the adjudged principal amount of Two Hundred Thousand Pesos (P200,000.00) shall be six percent (6%) per annum computed from the date of filing of the Complaint in the trial court. The awards in the amounts of Fifty Thousand Pesos (P50,000.00) as actual damages, Fifty Thousand Pesos (P50,000.00) as exemplary damages, Five Thousand Pesos (P5,000.00) as litigation expenses, and Ten Thousand Pesos (P10,000.00) as attorneys fees are hereby DELETED. Costs against the petitioner. SO ORDERED.

PACIFIC TIMBER V. CA Concerning the award of exemplary damages for Fifty Thousand Pesos (P50,000.00), we likewise find no legal and valid basis for granting the same. Article 2229 of the New Civil Code provides that exemplary damages may be imposed by way of example or correction for the public good. Exemplary damages are imposed not to enrich one party or impoverish another but to serve as a deterrent against or as a negative incentive to curb socially deleterious actions. They are designed to permit the courts to mould behavior that has socially deleterious consequences, and its imposition is required by public policy to suppress the wanton acts of an offender. However, it cannot be recovered as a matter of right. It is based entirely on the discretion of the court. We find no cogent and valid reason to award the same in the case at bar. 112 SCRA 199 Facts: > On March 13, 1963, Pacific secured temporary insurance from the Workemens Insurance Co. for its exportation of logs to Japan. Workmen issued on said date Cover Note 1010 insuring said cargo. > The regular marine policies were issued by the company in favor of Pacific on Apr 2, 1963. The 2 marine policies bore the number 53H01032 and 53H01033. > After the issuance of the cover note but BEFORE the issuance of the 2 policies, some of the logs intended to be exported were lost due to a typhoon.

> Pacific filed its claim with the company, but the latter refused, contending that said loss may not be considered as covered under the cover note because such became null and void by virtue of the issuance of the marine policies. Issue: Whether or not the cover not was without consideration, thus null and void. Held: It was with consideration. SC upheld Pacifics contention that said cover not was with consideration. The fact that no separate premium was paid on the cover note before the loss was insured against occurred does not militate against the validity of Pacifics contention, for no such premium could have been paid, since by the nature of the cover note, it did not contain, as all cover notes do not contain, particulars of the shipment that would serve as basis for the computation of the premiums. As a logical consequence, no separate premiums are required to be paid on a cover note. If the note is to be treated as a separate policy instead of integrating it to the regular policies subsequently issued, its purpose would be meaningless for it is in a real sense a contract, not a mere application.

> Mondragon wrote on the bottom of the application form his strong recommendation for the approval of the insurance application. > On Apr 30, 1957, Mondragon received a letter from Grepalife Main office disapproving the insurance application of Ngo for the simple reason that the 20yr endowment plan is not available for minors below 7 yrs old. > Mondragon wrote back the main office again strongly recommending the approval of the endowment plan on the life of Helen, adding that Grepalife was the only insurance company NOT selling endowment plans to children. > On may 1957, Helen died of influenza with complication of broncho pneumonia. Ngo filed a claim with Gepalife, but the latter denied liability on the ground that there was no contract between the insurer and the insured and a binding receipt is NOT evidence of such contract. Issue: Whether or not the binding deposit receipt, constituted a temporary contract of life insurance. Held: NO. The binding receipt in question was merely an acknowledgement on behalf of the company, that the latters branch office had received from the applicant, the insurance premium and had accepted the application subject for processing by the insurance company, and that the latter will either approve or reject the same on the basis of whether or not the applicant is insurable on standard rates.

GREPALIFE V. CA Facts:

89 SCRA 543

> On March 14, 1957, respondent Ngo Hing filed an application with Grepalife for a 20-yr endowment policy for 50T on the life of his one year old daughter Helen Go. > All the essential data regarding Helen was supplied by Ngo to Lapu-Lapu Mondragon, the branch manager of Grepalife-Cebu. Mondragon then typed the data on the application form which was later signed by Ngo. > Ngo then paid the insurance premium and a binding deposit receipt was issued to him. The binding receipt contained the following provision: If the applicant shall not have been insurable xxx and the Company declines to approve the application, the insurance applied for shall not have been in force at any time and the sum paid shall be returned to the applicant upon the surrender of this receipt.

Since Grepalife disapproved the insurance application of Ngo, the binding deposit receipt had never became on force at any time, pursuant to par. E of the said receipt. A binding receipt is manifestly merely conditional and does NOT insure outright. Where an agreement is made between the applicant and the agent, NO liability shall attach until the principal approves the risk and a receipt is given by the agent.

The acceptance is merely conditional, and is subordinated to the act of the company in approving or rejecting the application. Thus in life insurance, a binding slip or binding receipt does NOT insure by itself.

[G.R. No. 139776. August 1, 2002] PHILAM LIFE vs. JUDGE VALENCIA-BAGALACSA

hypertensive since 1974; private respondents sent a letter dated May 25, 1983[2] requesting for reconsideration of the denial; in a letter dated July 11, 1983, it reiterated its decision to deny the claim for payment of the proceeds;[3] more than ten (10) years later, or on December 1, 1994, it received a letter from Jose C. Claro, a provincial board member of the province of Camarines Sur, reiterating the early request for reconsideration which it denied in a letter dated February 14, 1995.[4]

Private respondents opposed the motion to dismiss.[5] Before us is a petition for review on certiorari under Rule 45 of the Rules of Court. Petitioner Philippine American Life and General Insurance Company prays that the decision of the Court of Appeals promulgated on April 30, 1999 be reversed and set aside and that the Complaint filed against it by private respondents Eduardo Z. Lumaniog, Celso Z. Lumaniog and Ruben Z. Lumaniog before the Regional Trial Court of Libmanan, Camarines Sur, docketed as Civil Case No. L-787 be ordered dismissed on ground of prescription of action. The facts of the case: On June 20, 1995, private respondents, as legitimate children and forced heirs of their late father, Faustino Lumaniog, filed with the aforesaid RTC, a complaint for recovery of sum of money against petitioner alleging that: their father was insured by petitioner under Life Insurance Policy No. 1305486 with a face value of P50,000.00; their father died of coronary thrombosis on November 25, 1980; on June 22, 1981, they claimed and continuously claimed for all the proceeds and interests under the life insurance policy in the amount of P641,000.00, despite repeated demands for payment and/or settlement of the claim due from petitioner, the last of which is on December 1, 1994, petitioner finally refused or disallowed said claim on February 14, 1995;[1] and so, they filed their complaint on June 20, 1995. Petitioner filed an Answer with Counterclaim and Motion to Dismiss, contending that: the cause of action of private respondents had prescribed and they are guilty of laches; it had denied private respondents claim in a letter dated March 12, 1982, signed by its then Assistant Vice President, Amado Dimalanta, on ground of concealment on the part of the deceased insured Faustino when he asserted in his application for insurance coverage that he had not been treated for indication of chest pain, palpitation, high blood pressure, rheumatic fever, heart murmur, heart attack or other disorder of the heart or blood vessel when in fact he was a known

On June 7, 1996, the RTC issued an Order which reads:

After a perusal of the motion to dismiss filed by defendants counsel and the objection submitted by plaintiffs counsel, the Court finds that the matters treated in their respective pleadings are evidentiary in nature, hence, the necessity of a trial on the merits.

Set therefore the hearing in this case on August 1, 1996 at 8:30 a.m., considering that the calendar of the Court is already filled up until the end of July. Notify parties and counsels. SO ORDERED.[6] Petitioners motion for reconsideration was denied by the RTC in its Order dated December 12, 1997 upholding however in the same Order the claim of private respondents counsel that the running of the 10-year period was stopped on May 25, 1983 when private respondents requested for a reconsideration of the denial and it was only on February 14, 1995 when petitioner finally decided to deny their claim that the 10-year period began to run.[7]

Petitioner filed a petition for certiorari (docketed as CA-G.R. SP No. 47885) under Rule 65 of the Rules of Court in the Court of Appeals and after the comment of the

private respondents and reply of petitioner, the appellate court rendered its Decision, dated April 30, 1999, portions of which read as follows: Thus, this Court of the opinion and so holds that the prescriptive period to bring the present action commences to run only on February 14, 1995 (Rollo, pp. 25-26), the date when the petitioner finally rejected the claim of private respondents and not in 1983. The ten year period should instead be counted from the date of rejection by the insurer in this case February 14, 1995 since this is the time when the cause of action accrues. This fact was supported further by the letter of the petitioner to Atty. Claro dated December 20, 1994, stating that they were reviewing the claim and shall advise Atty. Claro of their action regarding his request for reconsideration (Id., p. 53). In the case of Summit Guaranty and Insurance Co., Inc. Vs. De Guzman (151 SCRA 389, 397-398), citing the case of Eagle Star Insurance Co., Ltd., et al. vs. Chia Yu, the Supreme Court held that: The plaintiffs cause of action did not accrue until his claim was finally rejected by the insurance company. This is because, before such final rejection, there was no real necessity for bringing suit. In the same case, the case of ACCFA vs. Alpha Insurance and Surety Co., was likewise cited where the Supreme Court ruled in this wise: Since a cause of action requires, as essential elements, not only a legal right of the plaintiff and a correlative of the defendant but also an act or omission of the defendant in violation of said legal right, the cause of action does not accrue until the party obligated refuses, expressly or impliedly, to comply with its duty. Hence, We find no grave abuse of discretion committed by the court a quo when it issued the Orders dated June 7, 1996 and dated December 12, 1997.

Hence, the present petition for review. Petitioner posits the following issues:

A. Whether or not the complaint filed by private respondents for payment of life insurance proceeds is already barred by prescription of action.

B. Whether or not an extrajudicial demand made after an action has prescribed shall cause the revival of the action.[9]

Private respondents filed their Comment and petitioners, their Reply.

Before we determine whether the Court of Appeals had committed any reversible error, we must necessarily first ascertain whether or not the RTC committed grave abuse of discretion in issuing the Orders dated June 7, 1996 and December 12, 1997.

Notably, the RTC was initially correct in issuing the Order dated June 7, 1996 when it set the case below for hearing as there are matters in the respective pleadings of the parties that are evidentiary in nature, hence the necessity of a trial on the merits[10], in effect, denying the motion to dismiss, pursuant to the then prevailing Section 3, Rule 16, of the Rules of Court, to wit:

WHEREFORE, the instant petition for certiorari with prayer for issuance of temporary restraining order and/or preliminary injunction is DENIED DUE COURSE and is accordingly DISMISSED by this Court for lack of merit. Costs against the petitioner. SO ORDERED.[8]

Sec. 3. Hearing and order. - After hearing the court may deny or grant the motion or allow amendment of pleading, or may defer the hearing and determination of the motion until the trial if the ground alleged therein does not appear to be indubitable.

before it was amended by the 1997 Rules of Civil Procedure, effective July 1, 1997.[11]

It must be emphasized that petitioner had specifically alleged in the Answer that it had denied private respondents claim per its letter dated July 11, 1983.[12] Hence, due process demands that it be given the opportunity to prove that private respondents had received said letter, dated July 11, 1983. Said letter is crucial to petitioners defense that the filing of the complaint for recovery of sum of money in June, 1995 is beyond the 10-year prescriptive period[13].

The ruling of the RTC that the cause of action of private respondents had not prescribed, is arbitrary and patently erroneous for not being founded on evidence on record, and therefore, the same is void.[15]

Consequently, while the Court of Appeals did not err in upholding the June 7, 1986 Order of the RTC, it committed a reversible error when it declared that the RTC did not commit any grave abuse of discretion in issuing the Order dated December 12, 1997.

It is for the above reason that the RTC committed a grave abuse of discretion when, in resolving the motion for reconsideration of petitioner, it arbitrarily ruled in its Order dated December 12, 1997, that the period of ten (10) years had not yet lapsed. It based its finding on a mere explanation of the private respondents counsel and not on evidence presented by the parties as to the date when to reckon the prescriptive period. Portions of the Order dated December 12, 1997 read:

A perusal of the record will likewise reveal that plaintiffs counsel explained that the running of the ten (10) year period was stopped on May 25, 1983, upon demand of Celso Lomaniog for the compliance of the contract and reconsideration of the decision. Counsel also wrote the President of the Company on December 1, 1994, asking for reconsideration. The letter was answered by the Assistant Vice President of the Claims Department of Philamlife, with the advise that the company is reviewing the claim. On February 14, 1995, Atty. Abis sent a letter to counsel, finally deciding the plaintiffs claim. Thus, the period of prescription should commence to run only from February 14, 1995, when Atty. Abis finally decided plaintiffs claim.

The appellate court should have granted the petition for certiorari assailing said Order of December 12, 1997. Certiorari is an appropriate remedy to assail an interlocutory order (1) when the tribunal issued such order without or in excess of jurisdiction or with grave abuse of discretion and (2) when the assailed interlocutory order is patently erroneous and the remedy of appeal would not afford adequate and expeditious relief.[16] Said Order was issued with grave abuse of discretion for being patently erroneous and arbitrary, thus, depriving petitioner of due process, as discussed earlier.

WHEREFORE, the petition is partly GRANTED. The assailed decision of the Court of Appeals dated April 30, 1999 insofar only as it upheld the Order dated December 12, 1997 is REVERSED and SET ASIDE. A new judgment is entered reversing and setting aside the Order dated December 12, 1997 of the Regional Trial Court of Libmanan, Camarines Sur (Branch 56) and affirming its Order dated June 20, 1995. Said RTC is directed to proceed with dispatch with Civil Case No. L-787. No costs.SO ORDERED.

It is evident from the foregoing that the ten (10) year period for plaintiffs to claim the insurance proceeds has not yet prescribed. The final determination denying the claim was made only on February 14, 1995. Hence, when the instant case was filed on June 20, 1995, the ten year period has not yet lapsed. Moreover, defendants counsel failed to comply with the requirements of the Rules in filing his motion for reconsideration.[14] (emphasis supplied)

TIBAY v. CA May a fire insurance policy be valid, binding and enforceable upon mere partial payment of premium? On 19 July 1990 the trial court ruled for petitioners and adjudged FORTUNE liable for the total value of the insured building and personal properties in the amount of P600,000.00 plus interest at the legal rate of 6% per annum from the filing of the complaint until full payment, and attorneys fees equivalent to 20% of the total amount claimed plus costs of suit.[2]

On 22 January 1987 private respondent Fortune Life and General Insurance Co., Inc. (FORTUNE) issued Fire Insurance Policy No. 136171 in favor of Violeta R. Tibay and/or Nicolas Roraldo on their two-storey residential building located at 5855 Zobel Street, Makati City, together with all their personal effects therein. The insurance was for P600,000.00 covering the period from 23 January 1987 to 23 January 1988. On 23 January 1987, of the total premium of P2,983.50, petitioner Violeta Tibay only paid P600.00 thus leaving a considerable balance unpaid.

On 24 March 1995 the Court of Appeals reversed the court a quo by declaring FORTUNE not to be liable to plaintiff-appellees therein but ordering defendantappellant to return to the former the premium of P2,983.50 plus 12% interest from 10 March 1987 until full payment.[3]

On 8 March 1987 the insured building was completely destroyed by fire. Two days later or on 10 March 1987 Violeta Tibay paid the balance of the premium. On the same day, she filed with FORTUNE a claim on the fire insurance policy. Her claim was accordingly referred to its adjuster, Goodwill Adjustment Services, Inc. (GASI), which immediately wrote Violeta requesting her to furnish it with the necessary documents for the investigation and processing of her claim. Petitioner forthwith complied. On 28 March 1987 she signed a non-waiver agreement with GASI to the effect that any action taken by the companies or their representatives in investigating the claim made by the claimant for his loss which occurred at 5855 Zobel Roxas, Makati on March 8, 1987, or in the investigating or ascertainment of the amount of actual cash value and loss, shall not waive or invalidate any condition of the policies of such companies held by said claimant, nor the rights of either or any of the parties to this agreement, and such action shall not be, or be claimed to be, an admission of liability on the part of said companies or any of them.[1]

Hence this petition for review with petitioners contending mainly that contrary to the conclusion of the appellate court, FORTUNE remains liable under the subject fire insurance policy inspite of the failure of petitioners to pay their premium in full.

We find no merit in the petition; hence, we affirm the Court of Appeals.

Insurance is a contract whereby one undertakes for a consideration to indemnify another against loss, damage or liability arising from an unknown or contingent event.[4] The consideration is the premium, which must be paid at the time and in the way and manner specified in the policy, and if not so paid, the policy will lapse and be forfeited by its own terms.[5]

In a letter dated 11 June 1987 FORTUNE denied the claim of Violeta for violation of Policy Condition No. 2 and of Sec. 77 of the Insurance Code. Efforts to settle the case before the Insurance Commission proved futile. On 3 March 1988 Violeta and the other petitioners sued FORTUNE for damages in the amount of P600,000.00 representing the total coverage of the fire insurance policy plus 12% interest per annum, P 100,000.00 moral damages, and attorneys fees equivalent to 20% of the total claim.

The pertinent provisions in the Policy on premium read

THIS POLICY OF INSURANCE WITNESSETH, THAT only after payment to the Company in accordance with Policy Condition No. 2 of the total premiums by the

insured as stipulated above for the period aforementioned for insuring against Loss or Damage by Fire or Lightning as herein appears, the Property herein described x x x

2. This policy including any renewal thereof and/or any endorsement thereon is not in force until the premium has been fully paid to and duly receipted by the Company in the manner provided herein.

Apparently the crux of the controversy lies in the phrase unless and until the premium thereof has been paid. This leads us to the manner of payment envisioned by the law to make the insurance policy operative and binding. For whatever judicial construction may be accorded the disputed phrase must ultimately yield to the clear mandate of the law. The principle that where the law does not distinguish the court should neither distinguish assumes that the legislature made no qualification on the use of a general word or expression. In Escosura v. San Miguel Brewery, inc.,[7] the Court through Mr. Justice Jesus G. Barrera, interpreting the phrase with pay used in connection with leaves of absence with pay granted to employees, ruled -

Any supplementary agreement seeking to amend this condition prepared by agent, broker or Company official, shall be deemed invalid and of no effect.

x x x the legislative practice seems to be that when the intention is to distinguish between full and partial payment, the modifying term is used x x x

Except only in those specific cases where corresponding rules and regulations which are or may hereafter be in force provide for the payment of the stipulated premiums in periodic installments at fixed percentage, it is hereby declared, agreed and warranted that this policy shall be deemed effective, valid and binding upon the Company only when the premiums therefor have actually been paid in full and duly acknowledged in a receipt signed by any authorized official or representative/agent of the Company in such manner as provided herein, (Italics supplied).[6]

Citing C. A. No. 647 governing maternity leaves of married women in government, R. A. No. 679 regulating employment of women and children, R.A. No. 843 granting vacation and sick leaves to judges of municipal courts and justices of the peace, and finally, Art. 1695 of the New Civil Code providing that every househelp shall be allowed four (4) days vacation each month, which laws simply stated with pay, the Court concluded that it was undisputed that in all these laws the phrase with pay used without any qualifying adjective meant that the employee was entitled to full compensation during his leave of absence.

Clearly the Policy provides for payment of premium in full. Accordingly, where the premium has only been partially paid and the balance paid only after the peril insured against has occurred, the insurance contract did not take effect and the insured cannot collect at all on the policy. This is fully supported by Sec. 77 of the Insurance Code which provides

SEC. 77. An insurer is entitled to payment of the premium as soon as the thing insured is exposed to the peril insured against. Notwithstanding any agreement to the contrary, no policy or contract of insurance issued by an insurance company is valid and binding unless and until the premium thereof has been paid, except in the case of a life or an industrial life policy whenever the grace period provision applies (Italics supplied).

Petitioners maintain otherwise. Insisting that FORTUNE is liable on the policy despite partial payment of the premium due and the express stipulation thereof to the contrary, petitioners rely heavily on the 1967 case of Philippine Phoenix and Insurance Co., Inc. v. Woodworks, Inc.[8] where the Court through Mr. Justice Arsenio P. Dizon sustained the ruling of the trial court that partial payment of the premium made the policy effective during the whole period of the policy. In that case, the insurance company commenced action against the insured for the unpaid balance on a fire insurance policy. In its defense the insured claimed that nonpayment of premium produced the cancellation of the insurance contract. Ruling otherwise the Court held

It is clear x x x that on April 1, 1960, Fire Insurance Policy No. 9652 was issued by appellee and delivered to appellant, and that on September 22 of the same year, the latter paid to the former the sum of P3,000.00 on account of the total premium of P6,051.95 due thereon. There is, consequently, no doubt at all that, as between the insurer and the insured, there was not only a perfected contract of insurance but a partially performed one as far as the payment of the agreed premium was concerned. Thereafter the obligation of the insurer to pay the insured the amount, for which the policy was issued in case the conditions therefor had been complied with, arose and became binding upon it, while the obligation of the insured to pay the remainder of the total amount of the premium due became demandable.

The obligation to pay premiums when due is ordinarily an indivisible obligation to pay the entire premium. Here, the parties x x x agreed to make the premiums payable in installments, and there is no pretense that the parties never envisioned to make the insurance contract binding between them. It was renewed for two succeeding years, the second and third policies being a renewal/replacement for the previous one. And the insured never informed the insurer that it was terminating the policy because the terms were unacceptable.

The 1967 Phoenix case is not persuasive; neither is it decisive of the instant dispute. For one, the factual scenario is different. In Phoenix it was the insurance company that sued for the balance of the premium, i.e., it recognized and admitted the existence of an insurance contract with the insured. In the case before us, there is, quite unlike in Phoenix, a specific stipulation that (t)his policy xxx is not in force until the premium has been fully paid and duly receipted by the Company x x x. Resultantly, it is correct to say that in Phoenix a contract was perfected upon partial payment of the premium since the parties had not otherwise stipulated that prepayment of the premium in full was a condition precedent to the existence of a contract.

While it maybe true that under Section 77 of the Insurance Code, the parties may not agree to make the insurance contract valid and binding without payment of premiums, there is nothing in said section which suggests that the parties may not agree to allow payment of the premiums in installment, or to consider the contract as valid and binding upon payment of the first premium. Otherwise we would allow the insurer to renege on its liability under the contract, had a loss incurred (sic) before completion of payment of the entire premium, despite its voluntary acceptance of partial payments, a result eschewed by basic considerations of fairness and equity x x x.

In Phoenix, by accepting the initial payment of P3,000.00 and then later demanding the remainder of the premium without any other precondition to its enforceability as in the instant case, the insurer in effect had shown its intention to continue with the existing contract of insurance, as in fact it was enforcing its right to collect premium, or exact specific performance from the insured. This is not so here. By express agreement of the parties, no vinculum juris or bond of law was to be established until full payment was effected prior to the occurrence of the risk insured against.

These two (2) cases, Phoenix and Tuscany, adequately demonstrate the waiver, either express or implied, of prepayment in full by the insurer: impliedly, by suing for the balance of the premium as inPhoenix, and expressly, by agreeing to make premiums payable in installments as in Tuscany. But contrary to the stance taken by petitioners, there is no waiver express or implied in the case at bench. Precisely, the insurer and the insured expressly stipulated that (t)his policy including any renewal thereof and/or any indorsement thereon is not in force until the premium has been fully paid to and duly receipted by the Company x x x and that this policy shall be deemed effective, valid and binding upon the Company only when the premiums therefor have actually been paid in full and duly acknowledged.

In Makati Tuscany Condominium Corp. v. Court of Appeals[9] the parties mutually agreed that the premiums could be paid in installments, which in fact they did for three (3) years, hence, this Court refused to invalidate the insurance policy. In giving effect to the policy, the Court quoted with approval the Court of Appeals

Conformably with the aforesaid stipulations explicitly worded and taken in conjunction with Sec. 77 of the Insurance Code the payment of partial premium by the assured in this particular instance should not be considered the payment required by the law and the stipulation of the parties. Rather, it must be taken in the concept of a deposit to be held in trust by the insurer until such time that the full amount has been tendered and duly receipted for. In other words, as expressly agreed upon in the

contract, full payment must be made before the risk occurs for the policy to be considered effective and in force.

Thus, no vinculum juris whereby the insurer bound itself to indemnify the assured according to law ever resulted from the fractional payment of premium. The insurance contract itself expressly provided that the policy would be effective only when the premium was paid in full. It would have been altogether different were it not so stipulated. Ergo, petitioners had absolute freedom of choice whether or not to be insured by FORTUNE under the terms of its policy and they freely opted to adhere thereto.

A maxim of recognized practicality is the rule that the expressed exception or exemption excludes others. Exceptio firm at regulim in casibus non exceptis. The express mention of exceptions operates to exclude other exceptions; conversely, those which are not within the enumerated exceptions are deemed included in the general rule. Thus, under Sec. 77, as well as Sec. 78, until the premium is paid, and the law has not expressly excepted partial payments, there is no valid and binding contract. Hence, in the absence of clear waiver of prepayment in full by the insurer, the insured cannot collect on the proceeds of the policy.

Indeed, and far more importantly, the cardinal polestar in the construction of an insurance contract is the intention of the parties as expressed in the policy.[10] Courts have no other function but to enforce the same. The rule that contracts of insurance will be construed in favor of the insured and most strongly against the insurer should not be permitted to have the effect of making a plain agreement ambiguous and then construe it in favor of the insured.[11] Verily, it is elemental law that the payment of premium is requisite to keep the policy of insurance in force. If the premium is not paid in the manner prescribed in the policy as intended by the parties the policy is ineffective. Partial payment even when accepted as a partial payment will not keep the policy alive even for such fractional part of the year as the part payment bears to the whole payment.[12]

Applying further the rules of statutory construction, the position maintained by petitioners becomes even more untenable. The case of South Sea Surety and Insurance Company, Inc. v. Court of Appeals,[13] speaks only of two (2) statutory exceptions to the requirement of payment of the entire premium as a prerequisite to the validity of the insurance contract. These exceptions are: (a) in case the insurance coverage relates to life or industrial life (health) insurance when a grace period applies, and (b) when the insurer makes a written acknowledgment of the receipt of premium, this acknowledgment being declared by law to, be then conclusive evidence of the premium payment.[14]

In the desire to safeguard the interest of the assured, itmust not be ignored that the contract of insurance is primarily a risk-distributing device, a mechanism by which all members of a group exposed to a particular risk contribute premiums to an insurer. From these contributory funds are paid whatever losses occur due to exposure to the peril insured against. Each party therefore takes a risk: the insurer, that of being compelled upon the happening of the contingency to pay the entire sum agreed upon, and the insured, that of parting with the amount required as premium, without receiving anything therefor in case the contingency does not happen. To ensure payment for these losses, the law mandates all insurance companies to maintain a legal reserve fund in favor of those claiming under their policies.[15] It should be understood that the integrity of this fund cannot be secured and maintained if by judicial fiat partial offerings of premiums were to be construed as a legal nexus between the applicant and the insurer despite an express agreement to the contrary. For what could prevent the insurance applicant from deliberately or wilfully holding back full premium payment and wait for the risk insured against to transpire and then conveniently pass on the balance of the premium to be deducted from the proceeds of the insurance? Worse, what if the insured makes an initial payment of only 10%, or even 1%, of the required premium, and when the risk occurs simply points to the proceeds from where to source the balance? Can an insurance company then exist and survive upon the payment of 1%, or even 10%, of the premium stipulated in the policy on the basis that, after all, the insurer can deduct from the proceeds of the insurance should the risk insured against occur?

Interpreting the contract of insurance stringently against the insurer but liberally in favor of the insured despite clearly defined obligations of the parties to the policy can be carried out to extremes that there is the danger that we may, so to speak, kill the goose that lays the golden egg. We are well aware of insurance companies falling

into the despicable habit of collecting premiums promptly yet resorting to all kinds of excuses to deny or delay payment of just insurance claims. But, in this case, the law is manifestly on the side of the insurer. For as long as the current Insurance Code remains unchanged and partial payment of premiums is not mentioned at all as among the exceptions provided in Secs. 77 and 78, no policy of insurance can ever pretend to be efficacious or effective until premium has been fully paid.

WHEREFORE, the petition is DENIED and the assailed Decision of the Court of Appeals dated 24 March 1995 is AFFIRMED. SO ORDERED.

UCPB GENERAL INSURANCE [UCPB] v. MASAGANA TELAMART FACTS

And so it must be. For it cannot be disputed that premium is the elixir vitae of the insurance business because by law the insurer must maintain a legal reserve fund to meet its contingent obligations to the public, hence, the imperative need for its prompt payment and full satisfaction.[16] It must be emphasized here that all actuarial calculations and various tabulations of probabilities of losses under the risks insured against are based on the sound hypothesis of prompt payment of premiums. Upon this bedrock insurance firms are enabled to offer the assurance of security to the public at favorable rates. But once payment of premium is left to the whim and caprice of the insured, as when the courts tolerate the payment of a mere P600.00 as partial undertaking out of the stipulated total premium of P2,983.50 and the balance to be paid even after the risk insured against has occurred, as petitioners have done in this case, on the principle that the strength of the vinculumjuris is not measured by any specific amount of premium payment, we will surely wreak havoc on the business and set to naught what has taken actuarians centuries to devise to arrive at a fair and equitable distribution of risks and benefits between the insurer and the insured.

In 1991, UCPB issued 5 fire insurance policies covering Masagana Telamarts various properties for the period from 22 May 1991 to 22 May 1992. On March 1992 [~2 months before policy expiration], UCPB evaluated the policies and decided not to renew them upon expiration of their terms on 22 May 1992. UCPB advised Masaganas broker of its intention not to renew the policies. On April 1992 [~1 month before policy expiration], UCPB gave written notice to Masagana of the non-renewal of the policies. On June 1992 [policy already expired], Masaganas property covered by 3 UCPB-issued policies was razed by fire. On 13 July 1992, Masagana presented to UCPBs cashier 5 manager's checks, representing premium for the renewal of the policies for another year. It was only on the following day, 14 July 1992, when Masagana filed with UCPB a formal claim for indemnification of the insured property razed by fire. On the same day, UCPB returned the 5 manager's checks, and rejected Masaganas claim since the policies had expired and were not renewed, and the fire occurred on 13 June 1992 (or before tender of premium payment). Masagana filed a civil complaint for recovery of the face value of the policies covering the insured property razed by fire. RTC ruled in favor of Masagana, as it found it to have complied with the obligation to pay the premium; hence, the replacement-renewal policy of these policies are effective and binding for another year [22 May 1992 22 May 1993]. CA affirmed RTC, holding that following previous practice, Masagana was allowed a 60-90 day credit term for the renewal of its policies, and that the acceptance of the late premium payment suggested that payment could be made later.

The terms of the insurance policy constitute the measure of the insurers liability. In the absence of statutory prohibition to the contrary, insurance companies have the same rights as individuals to limit their liability and to impose whatever conditions they deem best upon their obligations not inconsistent with public policy.[17] The validity of these limitations is by law passed upon by the Insurance Commissioner who is empowered to approve all forms of policies, certificates or contracts of insurance which insurers intend to issue or deliver. That the policy contract in the case at bench was approved and allowed issuance simply reaffirms the validity of such policy, particularly the provision in question.

ISSUE & HOLDING WON the fire insurance policies had expired on 22 May 1992, or had been extended or renewed by an implied credit arrangement though actual payment of premium was tendered on a later date after the occurrence of the risk insured against [fire]. FIRE INSURANCE POLICIES HAD EXPIRED

In March 1992, petitioner evaluated the policies and decided not to renew them upon expiration of their terms on May 22, 1992. Petitioner advised respondent's broker, Zuellig Insurance Brokers, Inc. of its intention not to renew the policies.

On April 6, 1992, petitioner gave written notice to respondent of the non-renewal of the policies at the address stated in the policies.

RATIO An insurance policy, other than life is not valid and binding until actual payment of the premium. Any agreement to the contrary is void. The parties may not agree expressly or impliedly on the extension of credit or time to pay the premium and consider the policy binding before actual payment. On June 13, 1992, fire razed respondent's property covered by three of the insurance policies petitioner issued.

The case of Malayan Insurance v. Cruz-Arnaldo cited by the CA is not applicable. In that case, payment of the premium was made on before the occurrence of the fire. In the present case, the payment of the premium for renewal of the policies was tendered a month after the fire occurred. Masagana did not even give UCPB a notice of loss within a reasonable time after occurrence of the fire. CA DECISION REVERSED.

On July 13, 1992, respondent presented to petitioner's cashier at its head office five (5) manager's checks in the total amount of P225,753.95, representing premium for the renewal of the policies from May 22, 1992 to May 22, 1993. No notice of loss was filed by respondent under the policies prior to July 14, 1992.

On July 14, 1992, respondent filed with petitioner its formal claim for indemnification of the insured property razed by fire.

UCPB GENERAL INSURANCE CO., INC., petitioner, vs. MASAGANA TELAMART, INC., respondent. [G.R. No. 137172. June 15, 1999 The case is an appeal via certiorari seeking to set aside the decision of the Court of Appeals,[1] affirming with modification that of the Regional Trial Court, Branch 58, Makati, ordering petitioner to pay respondent the sum of P18,645,000.00, as the proceeds of the insurance coverage of respondent's property razed by fire; 25% of the total amount due as attorney's fees and P25,000.00 as litigation expenses, and costs. The facts are undisputed and may be related as follows: On April 15, 1991, petitioner issued five (5) insurance policies covering respondent's various property described therein against fire, for the period from May 22, 1991 to May 22, 1992.

On the same day, July 14, 1992, petitioner returned to respondent the five (5) manager's checks that it tendered, and at the same time rejected respondent's claim for the reasons (a) that the policies had expired and were not renewed, and (b) that the fire occurred on June 13, 1992, before respondent's tender of premium payment.

On July 21, 1992, respondent filed with the Regional Trial Court, Branch 58, Makati City, a civil complaint against petitioner for recovery of P18,645,000.00, representing the face value of the policies covering respondent's insured property razed by fire, and for attorney's fees.[2]

On October 23, 1992, after its motion to dismiss had been denied, petitioner filed an answer to the complaint. It alleged that the complaint "fails to state a cause of action"; that petitioner was not liable to respondent for insurance proceeds under the policies because at the time of the loss of respondent's property due to fire, the policies had long expired and were not renewed.[3]

CA-G.R. CV No. 40751, which affirmed in toto the decision of the Regional Trial Court, Makati City, Branch 150 (hereafter trial court), in Civil Case No. 91-1009. Petitioner is a domestic corporation engaged in the insurance business. Sometime in 1990, respondent obtained from petitioner a fire insurance covering the stock-in-trade of his business, Moonlight Enterprises, located at Valencia, Bukidnon. The insurance was due to expire on 25 March 1990. On 5 April 1990 respondent issued PCIBank Check No. 352123 in the amount of P2,983.50 to petitioners agent, James Uy, as payment for the renewal of the policy. In turn, the latter delivered Renewal Certificate No. 00099047 to respondent. The check was drawn against a Manila bank and deposited in petitioners bank account in Cagayan de Oro City. The corresponding official receipt was issued on 10 April. Subsequently, a new insurance policy, Policy No. 206-4234498-7, was issued, whereby petitioner undertook to indemnify respondent for any damage or loss arising from fire up to P200,000 for the period 25 March 1990 to 25 March 1991.

After due trial, on March 10, 1993, the Regional Trial Court, Branch 58, Makati, rendered decision, the dispositive portion of which reads: "WHEREFORE, premises considered, judgment is hereby rendered in favor of the plaintiff and against the defendant, as follows: "(1) Authorizing and allowing the plaintiff to consign/deposit with this Court the sum of P225,753.95 (refused by the defendant) as full payment of the corresponding premiums for the replacement-renewal policies for Exhibits A, B, C, D and E; "(2) Declaring plaintiff to have fully complied with its obligation to pay the premium thereby rendering the replacement-renewal policy of Exhibits A, B, C, D and E effective and binding for the duration May 22, 1992 until May 22, 1993; and, ordering defendant to deliver forthwith to plaintiff the said replacement-renewal policies; "(3) Declaring Exhibits A & B, in force from August 22, 1991 up to August 23, 1992 and August 9, 1991 to August 9, 1992, respectively; and "(4) Ordering the defendant to pay plaintiff the sums of: (a) P18,645,000.00 representing the latter's claim for indemnity under Exhibits A, B & C and/or its replacement-renewal policies; (b) 25% of the total amount due as and for attorney's fees; (c) P25,000.00 as necessary litigation expenses; and, (d) the costs of suit. "All other claims and counterclaims asserted by the parties are denied and/or dismissed, including plaintiff's claim for interests. "SO ORDERED.

On 6 April 1990 Moonlight Enterprises was completely razed by fire. Total loss was estimated between P4,000,000 and P5,000,000. Respondent filed an insurance claim with petitioner and four other co-insurers, namely, Pioneer Insurance and Surety Corporation, Prudential Guarantee and Assurance, Inc., Filipino Merchants Insurance Co. and Domestic Insurance Company of the Philippines. Petitioner refused to honor the claim notwithstanding several demands by respondent, thus, the latter filed an action against petitioner before the trial court.

AMERICAN HOME ASSURANCE vs. CHUA, In this petition for review on certiorari under Rule 45 of the 1997 Rules of Civil Procedure, petitioner seeks the reversal of the decision[1] of the Court of Appeals in

In its defense, petitioner claimed there was no existing insurance contract when the fire occurred since respondent did not pay the premium. It also alleged that even assuming there was a contract, respondent violated several conditions of the policy, particularly: (1) his submission of fraudulent income tax return and financial statements; (2) his failure to establish the actual loss, which petitioner assessed at P70,000; and (3) his failure to notify to petitioner of any insurance already effected to cover the insured goods. These violations, petitioner insisted, justified the denial of the claim.

The trial court ruled in favor of respondent. It found that respondent paid by way of check a day before the fire occurred. The check, which was deposited in petitioners bank account, was even acknowledged in the renewal certificate issued by petitioners agent. It declared that the alleged fraudulent documents were limited to the disparity between the official receipts issued by the Bureau of Internal Revenue (BIR) and the income tax returns for the years 1987 to 1989. All the other documents were found to be genuine. Nonetheless, it gave credence to the BIR certification that respondent paid the corresponding taxes due for the questioned years.

On appeal, the assailed decision was affirmed in toto by the Court of Appeals. The Court of Appeals found that respondents claim was substantially proved and petitioners unjustified refusal to pay the claim entitled respondent to the award of damages.

Its motion for reconsideration of the judgment having been denied, petitioner filed the petition in this case. Petitioner reiterates its stand that there was no existing insurance contract between the parties. It invokes Section 77 of the Insurance Code, which provides:

As to respondents failure to notify petitioner of the other insurance contracts covering the same goods, the trial court held that petitioner failed to show that such omission was intentional and fraudulent. Finally, it noted that petitioners investigation of respondent's claim was done in collaboration with the representatives of other insurance companies who found no irregularity therein. In fact, Pioneer Insurance and Surety Corporation and Prudential Guarantee and Assurance, Inc. promptly paid the claims filed by respondent.

An insurer is entitled to payment of the premium as soon as the thing insured is exposed to the peril insured against. Notwithstanding any agreement to the contrary, no policy or contract of insurance issued by an insurance company is valid and binding unless and until the premium thereof has been paid, except in the case of life or an industrial life policy whenever the grace period provision applies.

The trial court decreed as follows: WHEREFORE, judgment is hereby rendered in favor of [respondent] and against the [petitioner] ordering the latter to pay the former the following: 1. P200,000.00, representing the amount of the insurance, plus legal interest from the date of filing of this case; 2. P200,000.00 as moral damages; 3. P200,000.00 as loss of profit; 4. P100,000.00 as exemplary damages; 5. P50,000.00 as attorneys fees; and 6. Cost of suit.

and cites the case of Arce v. Capital Insurance & Surety Co., Inc.,[2] where we ruled that unless and until the premium is paid there is no insurance.

Petitioner emphasizes that when the fire occurred on 6 April 1990 the insurance contract was not yet subsisting pursuant to Article 1249[3] of the Civil Code, which recognizes that a check can only effect payment once it has been cashed. Although respondent testified that he gave the check on 5 April to a certain James Uy, the check, drawn against a Manila bank and deposited in a Cagayan de Oro City bank, could not have been cleared by 6 April, the date of the fire. In fact, the official receipt issued for respondents check payment was dated 10 April 1990, four days after the fire occurred.

Citing jurisprudence,[4] petitioner also contends that respondents non-disclosure of the other insurance contracts rendered the policy void. It underscores the trial courts neglect in considering the Commission on Audits certification that the BIR receipts

submitted by respondent were, in effect, fake since they were issued to other persons. Finally, petitioner argues that the award of damages was excessive and unreasonable considering that it did not act in bad faith in denying respondents claim.

In its reply, petitioner alleges that the petition questions the conclusions of law made by the trial court and the Court of Appeals.

Respondent counters that the issue of non-payment of premium is a question of fact which can no longer be assailed. The trial courts finding on the matter, which was affirmed by the Court of Appeals, is conclusive.

Respondent refutes the reason for petitioners denial of his claim. As found by the trial court, petitioners loss adjuster admitted prior knowledge of respondents existing insurance contracts with the other insurance companies. Nonetheless, the loss adjuster recommended the denial of the claim, not because of the said contracts, but because he was suspicious of the authenticity of certain documents which respondent submitted in filing his claim.

Petitioner invokes respondents admission that his check for the renewal of the policy was received only on 10 April 1990, taking into account that the policy period was 25 March 1990 to 25 March 1991. The official receipt was dated 10 April 1990. Anent respondents testimony that the check was given to petitioners agent, a certain James Uy, the latter points out that even respondent was not sure if Uy was indeed its agent. It faults respondent for not producing Uy as his witness and not taking any receipt from him upon presentment of the check. Even assuming that the check was received a day before the occurrence of the fire, there still could not have been any payment until the check was cleared.

Moreover, petitioner denies respondents allegation that it intended a renewal of the contract for the renewal certificate clearly specified the following conditions:

To bolster his argument, respondent cites Section 66 of the Insurance Code,[5] which requires the insurer to give a notice to the insured of its intention to terminate the policy forty-five days before the policy period ends. In the instant case, petitioner opted not to terminate the policy. Instead, it renewed the policy by sending its agent to respondent, who was issued a renewal certificate upon delivery of his check payment for the renewal of premium. At this precise moment the contract of insurance was executed and already in effect. Respondent also claims that it is standard operating procedure in the provinces to pay insurance premiums by check when collected by insurance agents.

Subject to the payment by the assured of the amount due prior to renewal date, the policy shall be renewed for the period stated. Any payment tendered other than in cash is received subject to actual cash collection. Subject to no loss prior to premium payment. If there be any loss, and is not covered [sic].

On the issue of damages, respondent maintains that the amounts awarded were reasonable. He cites numerous trips he had to make from Cagayan de Oro City to Manila to follow up his rightful claim. He imputes bad faith on petitioner who made enforcement of his claim difficult in the hope that he would eventually abandon it. He further emphasizes that the adjusters of the other insurance companies recommended payment of his claim, and they complied therewith.

Petitioner asserts that an insurance contract can only be enforced upon the payment of the premium, which should have been made before the renewal period.

Finally, in assailing the excessive damages awarded to respondent petitioner stresses that the policy in issue was limited to a liability of P200,000; but the trial court granted the following monetary awards: P200,000 as actual damages; P200,000 as moral damages; P100,000 as exemplary damages; and P50,000 as attorneys fees.

The following issues must be resolved: first, whether there was a valid payment of premium, considering that respondents check was cashed after the occurrence of the fire; second, whether respondent violated the policy by his submission of fraudulent documents and non-disclosure of the other existing insurance contracts; and finally, whether respondent is entitled to the award of damages.

An acknowledgment in a policy or contract of insurance of the receipt of premium is conclusive evidence of its payment, so far as to make the policy binding, notwithstanding any stipulation therein that it shall not be binding until the premium is actually paid. This Section establishes a legal fiction of payment and should be interpreted as an exception to Section 77.[9] Is respondent guilty of the policy violations imputed against him? We are not convinced by petitioners arguments. The submission of the alleged fraudulent documents pertained to respondents income tax returns for 1987 to 1989. Respondent, however, presented a BIR certification that he had paid the proper taxes for the said years. The trial court and the Court of Appeals gave credence to the certification and it being a question of fact, we hold that said finding is conclusive.

The general rule in insurance laws is that unless the premium is paid the insurance policy is not valid and binding. The only exceptions are life and industrial life insurance.[6] Whether payment was indeed made is a question of fact which is best determined by the trial court. The trial court found, as affirmed by the Court of Appeals, that there was a valid check payment by respondent to petitioner. Wellsettled is the rule that the factual findings and conclusions of the trial court and the Court of Appeals are entitled to great weight and respect, and will not be disturbed on appeal in the absence of any clear showing that the trial court overlooked certain facts or circumstances which would substantially affect the disposition of the case.[7] We see no reason to depart from this ruling.

According to the trial court the renewal certificate issued to respondent contained the acknowledgment that premium had been paid. It is not disputed that the check drawn by respondent in favor of petitioner and delivered to its agent was honored when presented and petitioner forthwith issued its official receipt to respondent on 10 April 1990. Section 306 of the Insurance Code provides that any insurance company which delivers a policy or contract of insurance to an insurance agent or insurance broker shall be deemed to have authorized such agent or broker to receive on its behalf payment of any premium which is due on such policy or contract of insurance at the time of its issuance or delivery or which becomes due thereon.[8] In the instant case, the best evidence of such authority is the fact that petitioner accepted the check and issued the official receipt for the payment. It is, as well, bound by its agents acknowledgment of receipt of payment.

Ordinarily, where the insurance policy specifies as a condition the disclosure of existing co-insurers, non-disclosure thereof is a violation that entitles the insurer to avoid the policy. This condition is common in fire insurance policies and is known as the other insurance clause. The purpose for the inclusion of this clause is to prevent an increase in the moral hazard. We have ruled on its validity and the case of Geagonia v. Court of Appeals[10] clearly illustrates such principle. However, we see an exception in the instant case.

Citing Section 29[11] of the Insurance Code, the trial court reasoned that respondents failure to disclose was not intentional and fraudulent. The application of Section 29 is misplaced. Section 29 concerns concealment which is intentional. The relevant provision is Section 75, which provides that:

A policy may declare that a violation of specified provisions thereof shall avoid it, otherwise the breach of an immaterial provision does not avoid the policy.

Section 78 of the Insurance Code explicitly provides:

To constitute a violation the other existing insurance contracts must be upon the same subject matter and with the same interest and risk.[12] Indeed, respondent acquired

several co-insurers and he failed to disclose this information to petitioner. Nonetheless, petitioner is estopped from invoking this argument. The trial court cited the testimony of petitioners loss adjuster who admitted previous knowledge of the co-insurers. Thus, COURT: Q The matter of additional insurance of other companies, was that ever discussed in your investigation?

Indubitably, it cannot be said that petitioner was deceived by respondent by the latters non-disclosure of the other insurance contracts when petitioner actually had prior knowledge thereof. Petitioners loss adjuster had known all along of the other existing insurance contracts, yet, he did not use that as basis for his recommendation of denial. The loss adjuster, being an employee of petitioner, is deemed a representative of the latter whose awareness of the other insurance contracts binds petitioner. We, therefore, hold that there was no violation of the other insurance clause by respondent.

A Yes, sir.

Petitioner is liable to pay its share of the loss. The trial court and the Court of Appeals were correct in awarding P200,000 for this. There is, however, merit in petitioners grievance against the damages and attorneys fees awarded.

Q In other words, from the start, you were aware the insured was insured with other companies like Pioneer and so on?

A Yes, Your Honor.

There is no legal and factual basis for the award of P200,000 for loss of profit. It cannot be denied that the fire totally gutted respondents business; thus, respondent no longer had any business to operate. His loss of profit cannot be shouldered by petitioner whose obligation is limited to the object of insurance, which was the stockin-trade, and not the expected loss in income or profit.

Q But in your report you never recommended the denial of the claim simply because of the non-disclosure of other insurance? [sic]

A Yes, Your Honor.

Q In other words, to be emphatic about this, the only reason you recommended the denial of the claim, you found three documents to be spurious. That is your only basis?

A Yes, Your Honor.[13] [Emphasis supplied]

Neither can we approve the award of moral and exemplary damages. At the core of this case is petitioners alleged breach of its obligation under a contract of insurance. Under Article 2220 of the Civil Code, moral damages may be awarded in breaches of contracts where the defendant acted fraudulently or in bad faith. We find no such fraud or bad faith. It must again be stressed that moral damages are emphatically not intended to enrich a plaintiff at the expense of the defendant. Such damages are awarded only to enable the injured party to obtain means, diversion or amusements that will serve to obviate the moral suffering he has undergone, by reason of the defendants culpable action. Its award is aimed at the restoration, within the limits of the possible, of the spiritual status quo ante, and it must be proportional to the suffering inflicted.[14] When awarded, moral damages must not be palpably and scandalously excessive as to indicate that it was the result of passion, prejudice or corruption on the part of the trial court judge.[15]

The law[16] is likewise clear that in contracts and quasi-contracts the court may award exemplary damages if the defendant acted in a wanton, fraudulent, reckless, oppressive, or malevolent manner. Nothing thereof can be attributed to petitioner which merely tried to resist what it claimed to be an unfounded claim for enforcement of the fire insurance policy.

As to attorneys fees, the general rule is that attorneys fees cannot be recovered as part of damages because of the policy that no premium should be placed on the right to litigate.[17] In short, the grant of attorneys fees as part of damages is the exception rather than the rule; counsels fees are not awarded every time a party prevails in a suit. It can be awarded only in the cases enumerated in Article 2208 of the Civil Code, and in all cases it must be reasonable.[18] Thereunder, the trial court may award attorneys fees where it deems just and equitable that it be so granted. While we respect the trial courts exercise of its discretion in this case, the award of P50,000 is unreasonable and excessive. It should be reduced to P10,000.

. . . Lope Maglana was an employee of the Bureau of Customs whose work station was at Lasa, here in Davao City. On December 20, 1978, early morning, Lope Maglana was on his way to his work station, driving a motorcycle owned by the Bureau of Customs. At Km. 7, Lanang, he met an accident that resulted in his death. He died on the spot. The PUJ jeep that bumped the deceased was driven by Pepito Into, operated and owned by defendant Destrajo. From the investigation conducted by the traffic investigator, the PUJ jeep was overtaking another passenger jeep that was going towards the city poblacion. While overtaking, the PUJ jeep of defendant Destrajo running abreast with the overtaken jeep, bumped the motorcycle driven by the deceased who was going towards the direction of Lasa, Davao City. The point of impact was on the lane of the motorcycle and the deceased was thrown from the road and met his untimely death. 1

WHEREFORE, the instant petition is partly GRANTED. The challenged decision of the Court of Appeals in CA-G.R. No. 40751 is hereby MODIFIED by a) deleting the awards of P200,000 for loss of profit, P200,000 as moral damages and P100,000 as exemplary damages, and b) reducing the award of attorneys fees from P50,000 to P10,000. No pronouncement as to costs. SO ORDERED.

Consequently, the heirs of Lope Maglana, Sr., here petitioners, filed an action for damages and attorney's fees against operator Patricio Destrajo and the Afisco Insurance Corporation (AFISCO for brevity) before the then Court of First Instance of Davao, Branch II. An information for homicide thru reckless imprudence was also filed against Pepito Into.

G.R. No. 60506

August 6, 1992

VDA. DE MAGLANA, v. HON. CONSOLACION,

During the pendency of the civil case, Into was sentenced to suffer an indeterminate penalty of one (1) year, eight (8) months and one (1) day of prision correccional, as minimum, to four (4) years, nine (9) months and eleven (11) days of prision correccional, as maximum, with all the accessory penalties provided by law, and to indemnify the heirs of Lope Maglana, Sr. in the amount of twelve thousand pesos (P12,000.00) with subsidiary imprisonment in case of insolvency, plus five thousand pesos (P5,000.00) in the concept of moral and exemplary damages with costs. No appeal was interposed by accused who later applied for probation. 2

The nature of the liability of an insurer sued together with the insured/operator-owner of a common carrier which figured in an accident causing the death of a third person is sought to be defined in this petition for certiorari.

On December 14, 1981, the lower court rendered a decision finding that Destrajo had not exercised sufficient diligence as the operator of the jeepney. The dispositive portion of the decision reads:

The facts as found by the trial court are as follows:

WHEREFORE, the Court finds judgment in favor of the plaintiffs against defendant Destrajo, ordering him to pay plaintiffs the sum of P28,000.00 for loss of income; to pay plaintiffs the sum of P12,000.00 which amount shall be deducted in the event judgment in Criminal Case No. 3527-D against the driver, accused Into, shall have been enforced; to pay plaintiffs the sum of P5,901.70 representing funeral and burial expenses of the deceased; to pay plaintiffs the sum of P5,000.00 as moral damages which shall be deducted in the event judgment (sic) in Criminal Case No. 3527-D against the driver, accused Into; to pay plaintiffs the sum of P3,000.00 as attorney's fees and to pay the costs of suit.

petitioners became direct beneficiaries under the provision of the policy which, in effect, is a stipulation pour autrui. 6 This motion was likewise denied for lack of merit.

Hence, petitioners filed the instant petition for certiorari which, although it does not seek the reversal of the lower court's decision in its entirety, prays for the setting aside or modification of the second paragraph of the dispositive portion of said decision. Petitioners reassert their position that the insurance company is directly and solidarily liable with the negligent operator up to the extent of its insurance coverage.

The defendant insurance company is ordered to reimburse defendant Destrajo whatever amounts the latter shall have paid only up to the extent of its insurance coverage. SO ORDERED. 3 We grant the petition. Petitioners filed a motion for the reconsideration of the second paragraph of the dispositive portion of the decision contending that AFISCO should not merely be held secondarily liable because the Insurance Code provides that the insurer's liability is "direct and primary and/or jointly and severally with the operator of the vehicle, although only up to the extent of the insurance coverage." 4 Hence, they argued that the P20,000.00 coverage of the insurance policy issued by AFISCO, should have been awarded in their favor. In its comment on the motion for reconsideration, AFISCO argued that since the Insurance Code does not expressly provide for a solidary obligation, the presumption is that the obligation is joint. The particular provision of the insurance policy on which petitioners base their claim is as follows:

Sec. 1 LIABILITY TO THE PUBLIC 1. The Company will, subject to the Limits of Liability, pay all sums necessary to discharge liability of the insured in respect of

(a)

death of or bodily injury to any THIRD PARTY

In its Order of February 9, 1982, the lower court denied the motion for reconsideration ruling that since the insurance contract "is in the nature of suretyship, then the liability of the insurer is secondary only up to the extent of the insurance coverage." 5

(b)

....

2. Petitioners filed a second motion for reconsideration reiterating that the liability of the insurer is direct, primary and solidary with the jeepney operator because the

....

3. In the event of the death of any person entitled to indemnity under this Policy, the Company will, in respect of the liability incurred to such person indemnify his personal representatives in terms of, and subject to the terms and conditions hereof. 7

two (2) respondents by reason of the indemnity contract against third party liability under which an insurer can be directly sued by a third party this will result in a violation of the principles underlying solidary obligation and insurance contracts. (emphasis supplied)

The above-quoted provision leads to no other conclusion but that AFISCO can be held directly liable by petitioners. As this Court ruled in Shafer vs. Judge, RTC of Olongapo City, Br. 75, "[w]here an insurance policy insures directly against liability, the insurer's liability accrues immediately upon the occurrence of the injury or even upon which the liability depends, and does not depend on the recovery of judgment by the injured party against the insured." 8 The underlying reason behind the third party liability (TPL) of the Compulsory Motor Vehicle Liability Insurance is "to protect injured persons against the insolvency of the insured who causes such injury, and to give such injured person a certain beneficial interest in the proceeds of the policy . . ." 9 Since petitioners had received from AFISCO the sum of P5,000.00 under the no-fault clause, AFISCO's liability is now limited to P15,000.00.

The Court then proceeded to distinguish the extent of the liability and manner of enforcing the same in ordinary contracts from that of insurance contracts. While in solidary obligations, the creditor may enforce the entire obligation against one of the solidary debtors, in an insurance contract, the insurer undertakes for a consideration to indemnify the insured against loss, damage or liability arising from an unknown or contingent event. 11 Thus, petitioner therein, which, under the insurance contract is liable only up to P20,000.00, can not be made solidarily liable with the insured for the entire obligation of P29,013.00 otherwise there would result "an evident breach of the concept of solidary obligation."

However, we cannot agree that AFISCO is likewise solidarily liable with Destrajo. In Malayan Insurance Co., Inc. v. Court of Appeals, 10 this Court had the opportunity to resolve the issue as to the nature of the liability of the insurer and the insured vis-a-vis the third party injured in an accident. We categorically ruled thus:

Similarly, petitioners herein cannot validly claim that AFISCO, whose liability under the insurance policy is also P20,000.00, can be held solidarily liable with Destrajo for the total amount of P53,901.70 in accordance with the decision of the lower court. Since under both the law and the insurance policy, AFISCO's liability is only up to P20,000.00, the second paragraph of the dispositive portion of the decision in question may have unwittingly sown confusion among the petitioners and their counsel. What should have been clearly stressed as to leave no room for doubt was the liability of AFISCO under the explicit terms of the insurance contract.

While it is true that where the insurance contract provides for indemnity against liability to third persons, such third persons can directly sue the insurer, however, the direct liability of the insurer under indemnity contracts against third party liability does not mean that the insurer can be held solidarily liable with the insured and/or the other parties found at fault. The liability of the insurer is based on contract; that of the insured is based on tort.

In fine, we conclude that the liability of AFISCO based on the insurance contract is direct, but not solidary with that of Destrajo which is based on Article 2180 of the Civil Code. 12 As such, petitioners have the option either to claim the P15,000 from AFISCO and the balance from Destrajo or enforce the entire judgment from Destrajo subject to reimbursement from AFISCO to the extent of the insurance coverage.

In the case at bar, petitioner as insurer of Sio Choy, is liable to respondent Vallejos (the injured third party), but it cannot, as incorrectly held by the trial court, be made "solidarily" liable with the two principal tortfeasors, namely respondents Sio Choy and San Leon Rice Mill, Inc. For if petitioner-insurer were solidarily liable with said,

While the petition seeks a definitive ruling only on the nature of AFISCO's liability, we noticed that the lower court erred in the computation of the probable loss of income. Using the formula: 2/3 of (80-56) x P12,000.00, it awarded P28,800.00. 13

Upon recomputation, the correct amount is P192,000.00. Being a "plain error," we opt to correct the same. 14 Furthermore, in accordance with prevailing jurisprudence, the death indemnity is hereby increased to P50,000.00. 15

Issues & Resolutions: Filipino contends that an "all risks" marine policy has a technical meaning in insurance in that before a claim can be compensable it is essential that there must be "some fortuity," "casualty" or "accidental cause" to which the alleged loss is attributable and the failure of herein private respondent, upon whom lay the burden, to adduce evidence showing that the alleged loss to the cargo in question was due to a fortuitous event precludes his right to recover from the insurance policy.

WHEREFORE, premises considered, the present petition is hereby GRANTED. The award of P28,800.00 representing loss of income is INCREASED to P192,000.00 and the death indemnity of P12,000.00 to P50,000.00. SO ORDERED.

FILIPINO MERCHANTS V. CA- INSURABLE INTEREST 179 SCRA 638 Facts: > The Chao Tiek Seng a consignee of the shipment of fishmeal loaded on board the vessel SS Bougainville and unloaded at the Port of Manila on or about December 11, 1976 and seeks to recover from Filipino the amount of P51,568.62 representing damages to said shipment which has been insured by Filipino. > Filipino brought a third party complaint against Compagnie Maritime Des Chargeurs Reunis and/or E. Razon, Inc. seeking judgment against the third party defendants in case judgment is rendered against it. > It appears from the evidence presented that Chao insured said shipment with Filipino for the sum of P267,653.59 for the goods described as 600 metric tons of fishmeal in gunny bags of 90 kilos each from Bangkok, Thailand to Manila against all risks under warehouse to warehouse terms. > Actually, what was imported was 59.940 metric tons not 600 tons at $395.42 a ton. > The fishmeal in 666 gunny bags were unloaded from the ship on December 11, 1976 at Manila unto the arrastre contractor E. Razon, Inc. and Filipinos surveyor ascertained and certified that in such discharge 105 bags were in bad order condition as jointly surveyed by the ship's agent and the arrastre contractor. > Based on said computation the Chao made a formal claim against the Filipino for P51,568.62. A formal claim statement was also presented by the plaintiff against the vessel, but the Filipino refused to pay the claim.

SC did not uphold this contention. An "all risks policy" should be read literally as meaning all risks whatsoever and covering all losses by an accidental cause of any kind. The terms "accident" and "accidental", as used in insurance contracts, have not acquired any technical meaning. They are construed by the courts in their ordinary and common acceptance. Thus, the terms have been taken to mean that which happens by chance or fortuitously, without intention and design, and which is unexpected, unusual and unforeseen. An accident is an event that takes place without one's foresight or expectation; an event that proceeds from an unknown cause, or is an unusual effect of a known cause and, therefore, not expected.

Coverage under an "all risks" provision of a marine insurance policy creates a special type of insurance which extends coverage to risks not usually contemplated and avoids putting upon the insured the burden of establishing that the loss was due to the peril falling within the policy's coverage; the insurer can avoid coverage upon demonstrating that a specific provision expressly excludes the loss from coverage. A marine insurance policy providing that the insurance was to be "against all risks" must be construed as creating a special insurance and extending to other risks than are usually contemplated, and covers all losses except such as arise from the fraud of the insured. The burden of the insured, therefore, is to prove merely that the goods he transported have been lost, destroyed or deteriorated. Thereafter, the burden is shifted to the insurer to prove that the loss was due to excepted perils. To impose on the insured the burden of proving the precise cause of the loss or damage would be inconsistent with the broad protective purpose of "all risks" insurance.

In the present case, there being no showing that the loss was caused by any of the excepted perils, the insurer is liable under the policy

Filipino contends that Chao does not have insurable interest, being only a consignee of the goods. Anent the issue of insurable interest, SC upheld the ruling of the CA that Chao, as consignee of the goods in transit under an invoice containing the terms under "C & F Manila," has insurable interest in said goods. Section 13 of the Insurance Code defines insurable interest in property as every interest in property, whether real or personal, or any relation thereto, or liability in respect thereof, of such nature that a contemplated peril might directly damnify the insured. In principle, anyone has an insurable interest in property who derives a benefit from its existence or would suffer loss from its destruction whether he has or has not any title in, or lien upon or possession of the property. Insurable interest in property may consist in (a) an existing interest; (b) an inchoate interest founded on an existing interest; or (c) an expectancy, coupled with an existing interest in that out of which the expectancy arises. Chao, as vendee/consignee of the goods in transit has such existing interest therein as may be the subject of a valid contract of insurance. His interest over the goods is based on the perfected contract of sale. The perfected contract of sale between him and the shipper of the goods operates to vest in him an equitable title even before delivery or before he performed the conditions of the sale. The contract of shipment, whether under F.O.B., C.I.F., or C. & F. as in this case, is immaterial in the determination of whether the vendee has an insurable interest or not in the goods in transit. The perfected contract of sale even without delivery vests in the vendee an equitable title, an existing interest over the goods sufficient to be the subject of insurance. Oriental Assurance Corporation vs. CA [G.R. No. 94052 August 9, 1991, 200 SCRA 459] Facts: Panama bought, in Palawan, 1,208 pieces of apitong logs, with a total volume of 2,000 cubic meters. It hired Transpacific Towage, Inc., to transport said logs by sea to Manila and insured it against loss for P1-M with Oriental Assurance.

stipulated in the policy that the insurance is against TOTAL LOSS only, and it is subject to the following clauses, to wit: Civil Code Article 1250 Waiver clause, Typhoon warranty clause, and Omnibus clause.

The logs were loaded on the 2 barges: (1) on barge PCT-7000, 610 pieces of logs with a volume of 1,000 cubicmeters; and (2) on Barge TPAC-1000, 598 pieces of logs, also with a volume of 1,000 cubic meters. On 28 January 1986, the 2 barges were towed by MT 'Seminole'(tugboat), during the voyage, rough seas and strong winds caused damage to Barge TPAC-1000 resulting in the loss of 497 pieces of logs out of the 598 pieces loaded thereon.

Panama demanded payment for the loss but Oriental Assurance refused on the ground that its contracted liability was for "TOTAL LOSS ONLY." Consequently, Panama filed a Complaint for Damages against Ever Insurance Agency (allegedly, also liable), Benito Sy Lee Yong and Oriental Assurance, before the RTC-Kalookan.

RTC rendered a decision ordering Oriental Assurance to pay Panama P415,000.00 as insurance indemnity. Both parties appealed. The appellate court affirmed the RTC decision. Both RTC and CA shared the view that the insurance contract should be liberally construed in order to avoid a denial of substantial justice; that the logs loaded in the two barges should be treated separately such that the loss sustained by the shipment in one of them may be considered as "constructive total loss" and correspondingly compensable.

Oriental Assurance filed a petition for review on certiorari challenging the aforesaid dispositions. Issue: Is Oriental Assurance liable?

The policy was issued. It is stipulated there, among others, that the subject matter insured is 2,000 cubic meters of apitong logs and that the vessels to be utilized are the following: MT. 'Seminole', Barge PCT-7000 for the 1,000 cubic meter of apitong logs and Barge Transpac-1000 for the other 1,000 cubic meter of apitong logs. It is also

Held: No. The SC held that the terms of the contract constitute the measure of the insurers liability and compliance therewith is a condition precedent to the insured's right to recovery from the insurer. That whether a contract is entire or severable is a question of intention to be determined by the language employed by the parties. The

policy in question shows that the subject matter insured was the entire shipment of 2,000 cubic meters of apitong logs. The fact that the logs were loaded on two different barges did not make the contract several and divisible as to the items insured. The logs on the two barges were not separately valued or separately insured. Only one premium was paid for the entire shipment, making for only one cause or consideration. The insurance contract must, therefore, be considered indivisible. The law provides that a constructive total loss, is one which gives to a person insured by a contract of marine insurance a right to abandon thing insured, or any particular portion thereof separately valued by the policy, or otherwise separately insured, and recover for a total loss thereof, when the cause of the loss is a peril injured against: (a) If more than three-fourths thereof in value is actually lost, or would have to be expended to recover it from the peril; (b) If it is injured to such an extent as to reduce its value more than three-fourths. The logs involved, although placed in two barges, were not separately valued by the policy, nor separately insured. Resultantly, the logs lost in barge TPAC-1000 in relation to the total number of logs loaded on the same barge cannot be made the basis for determining constructive total loss. The logs having been insured as one inseparable unit, the correct basis for determining the existence of constructive total loss is the totality of the shipment of logs. Of the entirety of 1,208, pieces of logs, only 497 pieces thereof were lost or 41.45% of the entire shipment. Since the cost of those 497 pieces does not exceed 75% of the value of all 1,208 pieces of logs, the shipment cannot be said to have sustained a constructive total loss. Hence, no recovery can be had against Oriental Assurance. The latter has no liability under the policy.

result in death or disability.On 22 May 1982, within the life of the policy, Gabriel died in Iraq. On 12 July 1983,Emerald Construction reported Gabriels death to Fortune Insurance by telephone. Amongthe documents thereafter submitted to Fortune Insurance were a copy of the deathcertificate issued by the Ministry of Health of the Republic of Iraq which stated that anautopsy report by the National Bureau of Investigation was conducted to the effect that dueto advanced state of postmortem decomposition, the cause of death of Gabriel couldnot be determined (emphasis added).Because of this development Fortune Insurance ultimately denied the claim of Emerald Construction on the ground of prescription. Gabriels widow, Jacqueline Jimenez,went to the to the lower court. In her complaint against Emerald Construction and FortuneInsurance, she averred that her husband died of electrocution while in the performance of his work.Fortune Insurance alleged that since both the death certificate issued by the IraqiMinistry of Health and the autopsy report of the NBI failed to disclose the cause of Gabrielsdeath, it denied liability under the policy. In addition, private respondent raised the defenseof prescription, invoking Section 384 of the Insurance Code. ISSUE: WON Jacqueline Jimenez vda. de Gabriels claim against Fortune Insurance should bedenied on the ground of prescription HELD:

Vda. De Gabriel v. CA G.R. No. 103883 November 14, 1996 FACTS: Marcelino Gabriel was employed by Emerald Construction & DevelopmentCorporation (Emerald Construction for brevity) at its construction project in Iraq. He wascovered by a personal accident insurance in the amount of P100,000.00 under a grouppolicy procured from Fortune Insurance & Surety Company (Fortune Insurance for brevity)by Emerald Construction for its overseas workers. The insured risk was for bodily injurycaused by violent accidental external and visible means which injury would solely andindependently of any other cause

Yes. Section 384 of the Insurance Code provides:Sec. 384. Any person having any claim upon the policy issued pursuantto this chapter shall, without any unnecessary delay, present to theinsurance company concerned a written notice of claim setting forththe nature, extent and duration of the injuries sustained as certified bya duly licensed physician. Notice of claim must be filed within sixmonths from date of the accident, otherwise, the claim shall bedeemed waived. Action or suit for recovery of damage due to loss orinjury must be brought, in proper cases, with the Commissioner or theCourts within one year from denial of the claim, otherwise, theclaimants right of action shall prescribe. The notice of death was given to Fortune Insurance, concededly, more than a yearafter the death of vda. de Gabriels husband. Fortune Insurance, in invoking prescription, was not referring to the one-year period from the denial of the claim within which to file anaction against an insurer but obviously to the written notice of claim that had to

besubmitted within six months from the time of the accident.Vda. de Gabriel argues that Fortune Insurance must be deemed to have waived itsright to show that the cause of death is an excepted peril, by failing to have its answers dulyverified. It is true that a matter of which a written request for admission is made shall bedeemed impliedly admitted unless, within a period designated in the request, which shallnot be less than 10 days after service thereof, or within such further time as the court mayallow on motion and notice, the party to whom the request is directed serves upon the partyrequesting the admission a sworn statement either denying specifically the matters of whichan admission is requested or setting forth in detail the reasons why he cannot truthfullyeither admit or deny those matters; however, the verification, like in most cases required bythe rules of procedure, is a formal, not jurisdictional, requirement, and mainly intended tosecure an assurance that matters which are alleged are done in good faith or are true andcorrect and not of mere speculation. When circumstances warrant, the court may simplyorder the correction of unverified pleadings or act on it and waive strict compliance with therules in order that the ends of justice may thereby be served. In the case of answers towritten requests for admission particularly, the court can allow the party making theadmission, whether made expressly or deemed to have been made impliedly, to withdraw oramend it upon such terms as may be just. The insurance policy expressly provided that to be compensable, the injury or deathshould be caused by violent accidental external and visible means. In attempting to provethe cause of her husbands death, all that vda. de Gabriel could submit were a letter sent toher by her husbands co-worker, stating that Gabriel died when he tried to haul water out of a tank while its submerged motor was still functioning, and vda. de Gabriels sworn affidavit. The said affidavit, however, suffers from procedural infirmity as it was not even testified toor identified by vda. de Gabriel herself. This affidavit therefore is a mere hearsay under thelaw.In like manner, the letter allegedly written by the deceaseds co-worker which wasnever identified to in court by the supposed author, suffers from the same defect as theaffidavit of vda. de Gabriel. Not one of the other documents submitted, to wit, the POEAdecision, the death certificate issued by the Ministry of Health of Iraq and the NBI autopsyreport, could give any probative value to vda. de Gabriels claim. The POEA decision did notmake any categorical holding on the specific cause of Gabriels death.In summary, evidence is utterly wanting to establish that the insured suffered froman accidental death, the risk covered by the policy

William Tiu vs Arriesgado (GR No 138060, Sept 1, 2004, Callejo)

Spouses Arriesgado were passengers of a bus owned by the petitioner. The respondents sustained injures when the bus collided with a cargo truck. In its defense, petitioner invoked the defense of last clear chance.

The SC held that Doctrine of last clear chance applies to a suit involving the owners of the two colliding vehicle. It does not apply to a suit involving breach for a contract of carriage. Philippine Rabbit Bus Lines vs Macalinao (GR No 141856, Feb 11, 2005, Sandoval-Gutierrez) Doctrine of last clear chance was applied in this case because it involves a suit between two colliding vehicles. Nostradamus Villanueva vs Priscilla and Leandro Domingo (GR No 144274, Sept 20, 2006, Corona)

Application of the Registered Owner Rule

The registered owner of a vehicle is directly and principally responsible for any accident, injury or death caused by the operation of the vehicle in the streets and highways.

The purpose is to protect the public in general and for easy identification of the persons who could be held responsible for the injury sustained.

Addendum: Extraordinary diligence is required to common carriers in transporting goods and passengers Reasons: 1. nature of the business 2. public policy

premium was paid on installments on 12 March 1982, 20 May 1982, 21 June 1982 and 16 November 1982, all of which were accepted by private respondent.

On 10 February 1983, private respondent issued to petitioner Insurance Policy No. AH-CPP-9210596, which replaced and renewed the previous policy, for a term covering 1 March 1983 to 1 March 1984. The premium in the amount of P466,103.05 was again paid on installments on 13 April 1983, 13 July 1983, 3 August 1983, 9 September 1983, and 21 November 1983. All payments were likewise accepted by private respondent.

Registered owner primarily and solidarily liable with driver under the KABIT SYSTEM. Kabit system is contrary to public policy; therefore, void and inexistent.

G.R. No. 95546

November 6, 1992

MAKATI TUSCANY CONDOMINIUM CORPORATION, v. CA This case involves a purely legal question: whether payment by installment of the premiums due on an insurance policy invalidates the contract of insurance, in view of Sec. 77 of P.D. 612, otherwise known as the Insurance Code, as amended, which provides: Sec. 77. An insurer is entitled to the payment of the premium as soon as the thing is exposed to the peril insured against. Notwithstanding any agreement to the contrary, no policy or contract of insurance issued by an insurance company is valid and binding unless and until the premium thereof has been paid, except in the case of a life or an industrial life policy whenever the grace period provision applies.

On 20 January 1984, the policy was again renewed and private respondent issued to petitioner Insurance Policy No. AH-CPP-9210651 for the period 1 March 1984 to 1 March 1985. On this renewed policy, petitioner made two installment payments, both accepted by private respondent, the first on 6 February 1984 for P52,000.00 and the second, on 6 June 1984 for P100,000.00. Thereafter, petitioner refused to pay the balance of the premium.

Consequently, private respondent filed an action to recover the unpaid balance of P314,103.05 for Insurance Policy No. AH-CPP-9210651.

In its answer with counterclaim, petitioner admitted the issuance of Insurance Policy No. AH-CPP-9210651. It explained that it discontinued the payment of premiums because the policy did not contain a credit clause in its favor and the receipts for the installment payments covering the policy for 1984-85, as well as the two (2) previous policies, stated the following reservations:

Sometime in early 1982, private respondent American Home Assurance Co. (AHAC), represented by American International Underwriters (Phils.), Inc., issued in favor of petitioner Makati Tuscany Condominium Corporation (TUSCANY) Insurance Policy No. AH-CPP-9210452 on the latter's building and premises, for a period beginning 1 March 1982 and ending 1 March 1983, with a total premium of P466,103.05. The

2. Acceptance of this payment shall not waive any of the company rights to deny liability on any claim under the policy arising before such payments or after the expiration of the credit clause of the policy; and

3. Subject to no loss prior to premium payment. If there be any loss such is not covered.

Petitioner further claimed that the policy was never binding and valid, and no risk attached to the policy. It then pleaded a counterclaim for P152,000.00 for the premiums already paid for 1984-85, and in its answer with amended counterclaim, sought the refund of P924,206.10 representing the premium payments for 1982-85. After some incidents, petitioner and private respondent moved for summary judgment. On 8 October 1987, the trial court dismissed the complaint and the counterclaim upon the following findings:

The obligation to pay premiums when due is ordinarily as indivisible obligation to pay the entire premium. Here, the parties herein agreed to make the premiums payable in installments, and there is no pretense that the parties never envisioned to make the insurance contract binding between them. It was renewed for two succeeding years, the second and third policies being a renewal/replacement for the previous one. And the insured never informed the insurer that it was terminating the policy because the terms were unacceptable.

While it is true that the receipts issued to the defendant contained the aforementioned reservations, it is equally true that payment of the premiums of the three aforementioned policies (being sought to be refunded) were made during the lifetime or term of said policies, hence, it could not be said, inspite of the reservations, that no risk attached under the policies. Consequently, defendant's counterclaim for refund is not justified. As regards the unpaid premiums on Insurance Policy No. AH-CPP-9210651, in view of the reservation in the receipts ordinarily issued by the plaintiff on premium payments the only plausible conclusion is that plaintiff has no right to demand their payment after the lapse of the term of said policy on March 1, 1985. Therefore, the defendant was justified in refusing to pay the same. 1

While it may be true that under Section 77 of the Insurance Code, the parties may not agree to make the insurance contract valid and binding without payment of premiums, there is nothing in said section which suggests that the parties may not agree to allow payment of the premiums in installment, or to consider the contract as valid and binding upon payment of the first premium. Otherwise, we would allow the insurer to renege on its liability under the contract, had a loss incurred (sic) before completion of payment of the entire premium, despite its voluntary acceptance of partial payments, a result eschewed by a basic considerations of fairness and equity.

To our mind, the insurance contract became valid and binding upon payment of the first premium, and the plaintiff could not have denied liability on the ground that payment was not made in full, for the reason that it agreed to accept installment payment. . . . 3

Both parties appealed from the judgment of the trial court. Thereafter, the Court of Appeals rendered a decision 2 modifying that of the trial court by ordering herein petitioner to pay the balance of the premiums due on Policy No. AH-CPP-921-651, or P314,103.05 plus legal interest until fully paid, and affirming the denial of the counterclaim. The appellate court thus explained

Petitioner now asserts that its payment by installment of the premiums for the insurance policies for 1982, 1983 and 1984 invalidated said policies because of the provisions of Sec. 77 of the Insurance Code, as amended, and by the conditions stipulated by the insurer in its receipts, disclaiming liability for loss for occurring before payment of premiums.

It argues that where the premiums is not actually paid in full, the policy would only be effective if there is an acknowledgment in the policy of the receipt of premium pursuant to Sec. 78 of the Insurance Code. The absence of an express acknowledgment in the policies of such receipt of the corresponding premium payments, and petitioner's failure to pay said premiums on or before the effective

dates of said policies rendered them invalid. Petitioner thus concludes that there cannot be a perfected contract of insurance upon mere partial payment of the premiums because under Sec. 77 of the Insurance Code, no contract of insurance is valid and binding unless the premium thereof has been paid, notwithstanding any agreement to the contrary. As a consequence, petitioner seeks a refund of all premium payments made on the alleged invalid insurance policies.

understanding to allow insured to pay premiums in installments not so proscribed. At the very least, both parties should be deemed in estoppel to question the arrangement they have voluntarily accepted. 4

The reliance by petitioner on Arce vs. Capital Surety and Insurance Co. 5 is unavailing because the facts therein are substantially different from those in the case at bar. In Arce, no payment was made by the insured at all despite the grace period given. In the case before Us, petitioner paid the initial installment and thereafter made staggered payments resulting in full payment of the 1982 and 1983 insurance policies. For the 1984 policy, petitioner paid two (2) installments although it refused to pay the balance.

We hold that the subject policies are valid even if the premiums were paid on installments. The records clearly show that petitioner and private respondent intended subject insurance policies to be binding and effective notwithstanding the staggered payment of the premiums. The initial insurance contract entered into in 1982 was renewed in 1983, then in 1984. In those three (3) years, the insurer accepted all the installment payments. Such acceptance of payments speaks loudly of the insurer's intention to honor the policies it issued to petitioner. Certainly, basic principles of equity and fairness would not allow the insurer to continue collecting and accepting the premiums, although paid on installments, and later deny liability on the lame excuse that the premiums were not prepared in full.

We therefore sustain the Court of Appeals. We quote with approval the well-reasoned findings and conclusion of the appellate court contained in its Resolution denying the motion to reconsider its Decision

It appearing from the peculiar circumstances that the parties actually intended to make three (3) insurance contracts valid, effective and binding, petitioner may not be allowed to renege on its obligation to pay the balance of the premium after the expiration of the whole term of the third policy (No. AH-CPP-9210651) in March 1985. Moreover, as correctly observed by the appellate court, where the risk is entire and the contract is indivisible, the insured is not entitled to a refund of the premiums paid if the insurer was exposed to the risk insured for any period, however brief or momentary.

While the import of Section 77 is that prepayment of premiums is strictly required as a condition to the validity of the contract, We are not prepared to rule that the request to make installment payments duly approved by the insurer, would prevent the entire contract of insurance from going into effect despite payment and acceptance of the initial premium or first installment. Section 78 of the Insurance Code in effect allows waiver by the insurer of the condition of prepayment by making an acknowledgment in the insurance policy of receipt of premium as conclusive evidence of payment so far as to make the policy binding despite the fact that premium is actually unpaid. Section 77 merely precludes the parties from stipulating that the policy is valid even if premiums are not paid, but does not expressly prohibit an agreement granting credit extension, and such an agreement is not contrary to morals, good customs, public order or public policy (De Leon, the Insurance Code, at p. 175). So is an

WHEREFORE, finding no reversible error in the judgment appealed from, the same is AFFIRMED. Costs against petitioner. SO ORDERED.

G.R. No. L-22684

August 31, 1967 That the defendant paid P3,000.00 on September 22, 1960 under official receipt No. 30245 of plaintiff;

PHILIPPINE PHOENIX SURETY & INSURANCE, INC., vs.WOODWORKS, INC., defendant-appellant.

Appeal upon a question of law taken by Woodworks, Inc. from the judgment of the Court of First Instance of Manila in Civil Case No. 50710 "ordering the defendant, Woodworks, Inc. to pay to the plaintiff, Philippine Phoenix Surety & Insurance, Inc., the sum of P3,522.09 with interest thereon at the legal rate of 6% per annum from the date of the filing of the complaint until fully paid, and costs of the suit."

That plaintiff made several demands on defendant to pay the amount of P3,522.09.1wph1.t

In the present appeal, appellant claims that the court a quo committed the following errors:

Appellee Philippine Phoenix Surety & Insurance Co., Inc. commenced this action in the Municipal Court of Manila to recover from appellant Woodworks, Inc. the sum of P3,522.09, representing the unpaid balance of the premiums on a fire insurance policy issued by appellee in favor of appellant for a term of one year from April 1, 1960 to April 1, 1961. From an adverse decision of said court, Woodworks, Inc. appealed to the Court of First Instance of Manila (Civil Case No. 50710) where the parties submitted the following stipulation of facts, on the basis of which the appealed decision was rendered:

I. The lower court erred in stating that in fire insurance policies the risk attached upon the issuance and delivery of the policy to the insured.

II. The lower court erred in deciding that in a perfected contract of insurance nonpayment of premium does not cancel the policy.

That plaintiff and defendant are both corporations duly organized and existing under and by virtue of the laws of the Philippines;

III. The lower court erred in deciding that the premium in the policy was still collectible when the complaint was filed.

That on April 1, 1960, plaintiff issued to defendant Fire Policy No. 9652 for the amount of P300,000.00, under the terms and conditions therein set forth in said policy a copy of which is hereto attached and made a part hereof as Annex "A";

IV. The lower court erred in deciding that a partial payment of the premium made the policy effective during the whole period of the policy.

That the premiums of said policy as stated in Annex "A" amounted to P6,051.95; the margin fee pursuant to the adopted plan as an implementation of Republic Act 2609 amounted to P363.72, copy of said adopted plan is hereto attached as Annex "B" and made a part hereof, the documentary stamps attached to the policy was P96.42;

It is clear from the foregoing that on April 1, 1960 Fire Insurance Policy No. 9652 was issued by appellee and delivered to appellant, and that on September 22 of the same year, the latter paid to the former the sum of P3,000.00 on account of the total premium of P6,051.95 due thereon. There is, consequently, no doubt at all that, as between the insurer and the insured, there was not only a perfected contract of insurance but a partially performed one as far as the payment of the agreed premium

was concerned. Thereafter the obligation of the insurer to pay the insured the amount for which the policy was issued in case the conditions therefor had been complied with, arose and became binding upon it, while the obligation of the insured to pay the remainder of the total amount of the premium due became demandable.

would load and carry on board its barge Mable 10 about 422.18 cubic meters of logs from Malampaya Sound, Palawan to North Harbor, Manila. The petitioners insured the logs against loss for P100,000.00 with respondent Pioneer Insurance and Surety Corporation (Pioneer).

We can not agree with appellant's theory that non-payment by it of the premium due, produced the cancellation of the contract of insurance. Such theory would place exclusively in the hands of one of the contracting parties the right to decide whether the contract should stand or not. Rather the correct view would seem to be this: as the contract had become perfected, the parties could demand from each other the performance of whatever obligations they had assumed. In the case of the insurer, it is obvious that it had the right to demand from the insured the completion of the payment of the premium due or sue for the rescission of the contract. As it chose to demand specific performance of the insured's obligation to pay the balance of the premium, the latter's duty to pay is indeed indubitable.

On February 29, 1972, the petitioners loaded on the barge, 811 pieces of logs at Malampaya Sound, Palawan for carriage and delivery to North Harbor, Port of Manila, but the shipment never reached its destination because Mable 10 sank with the 811 pieces of logs somewhere off Cabuli Point in Palawan on its way to Manila. As alleged by the petitioners in their complaint and as found by both the trial and appellate courts, the barge where the logs were loaded was not seaworthy such that it developed a leak. The appellate court further found that one of the hatches was left open causing water to enter the barge and because the barge was not provided with the necessary cover or tarpaulin, the ordinary splash of sea waves brought more water inside the barge.

Having thus resolved that the fourth and last assignment of error submitted in appellant's brief is without merit, the first three assignments of error must likewise be overruled as lacking in merit.

Wherefore, the appealed decision being in accordance with law and the evidence, the same is hereby affirmed, with costs.

On March 8, 1972, the petitioners wrote a letter to Manila Bay demanding payment of P150,000.00 for the loss of the shipment plus P100,000.00 as unrealized profits but the latter ignored the demand. Another letter was sent to respondent Pioneer claiming the full amount of P100,000.00 under the insurance policy but respondent refused to pay on the ground that its hability depended upon the "Total loss by Total Loss of Vessel only". Hence, petitioners commenced Civil Case No. 86599 against Manila Bay and respondent Pioneer.

ISABELA ROQUE, , vs. IAC

G.R. No. L-66935

November 11, 1985

After hearing, the trial court found in favor of the petitioners. The dispositive portion of the decision reads: FOR ALL THE FOREGOING, the Court hereby rendered judgment as follows: (a) Condemning defendants Manila Bay Lighterage Corporation and Pioneer Insurance and Surety Corporation to pay plaintiffs, jointly and severally, the sum of P100,000.00;

This petition for certiorari asks for the review of the decision of the Intermediate Appellate Court which absolved the respondent insurance company from liability on the grounds that the vessel carrying the insured cargo was unseaworthy and the loss of said cargo was caused not by the perils of the sea but by the perils of the ship.

On February 19, 1972, the Manila Bay Lighterage Corporation (Manila Bay), a common carrier, entered into a contract with the petitioners whereby the former

(b) Sentencing defendant Manila Bay Lighterage Corporation to pay plaintiff, in addition, the sum of P50,000.00, plus P12,500.00, that the latter advanced to the former as down payment for transporting the logs in question;

On January 30, 1984, the appellate court modified the trial court's decision and absolved Pioneer from liability after finding that there was a breach of implied warranty of seaworthiness on the part of the petitioners and that the loss of the insured cargo was caused by the "perils of the ship" and not by the "perils of the sea". It ruled that the loss is not covered by the marine insurance policy.

(c) Ordering the counterclaim of defendant Insurance against plaintiffs, dismissed, for lack of merit, but as to its cross-claim against its co-defendant Manila Bay Lighterage Corporation, the latter is ordered to reimburse the former for whatever amount it may pay the plaintiffs as such surety;

After the appellate court denied their motion for reconsideration, the petitioners filed this petition with the following assignments of errors: I

(d) Ordering the counterclaim of defendant Lighterage against plaintiffs, dismissed for lack of merit;

THE INTERMEDIATE APPELLATE COURT ERRED IN HOLDING THAT IN CASES OF MARINE CARGO INSURANCE, THERE IS A WARRANTY OF SEAWORTHINESS BY THE CARGO OWNER. II THE INTERMEDIATE APPELLATE COURT ERRED IN HOLDING THAT THE LOSS OF THE CARGO IN THIS CASE WAS CAUSED BY "PERILS OF THE SHIP" AND NOT BY "PERILS OF THE SEA." III

(e) Plaintiffs' claim of not less than P100,000.00 and P75,000.00 as exemplary damages are ordered dismissed, for lack of merits; plaintiffs' claim for attorney's fees in the sum of P10,000.00 is hereby granted, against both defendants, who are, moreover ordered to pay the costs; and

(f) The sum of P150,000.00 award to plaintiffs, shall bear interest of six per cent (6%) from March 25, 1975, until amount is fully paid.

THE INTERMEDIATE APPELLATE COURT ERRED IN NOT ORDERING THE RETURN TO PETITIONER OF THE AMOUNT OF P8,000.00 WHICH WAS DEPOSITED IN THE TRIAL COURT AS SALVAGE VALUE OF THE LOGS THAT WERE RECOVERED. In their first assignment of error, the petitioners contend that the implied warranty of seaworthiness provided for in the Insurance Code refers only to the responsibility of the shipowner who must see to it that his ship is reasonably fit to make in safety the contemplated voyage.

Respondent Pioneer appealed to the Intermediate Appellate Court. Manila Bay did not appeal. According to the petitioners, the transportation company is no longer doing business and is without funds.

During the initial stages of the hearing, Manila Bay informed the trial court that it had salvaged part of the logs. The court ordered them to be sold to the highest bidder with the funds to be deposited in a bank in the name of Civil Case No. 86599.

The petitioners state that a mere shipper of cargo, having no control over the ship, has nothing to do with its seaworthiness. They argue that a cargo owner has no control over the structure of the ship, its cables, anchors, fuel and provisions, the manner of loading his cargo and the cargo of other shippers, and the hiring of a sufficient number of competent officers and seamen. The petitioners' arguments have no merit.

There is no dispute over the liability of the common carrier Manila Bay. In fact, it did not bother to appeal the questioned decision. However, the petitioners state that Manila Bay has ceased operating as a firm and nothing may be recovered from it. They are, therefore, trying to recover their losses from the insurer.

Moreover, the fact that the unseaworthiness of the ship was unknown to the insured is immaterial in ordinary marine insurance and may not be used by him as a defense in order to recover on the marine insurance policy. As was held in Richelieu and Ontario Nav. Co. v. Boston Marine, Inc., Co. (136 U.S. 406): There was no look-out, and both that and the rate of speed were contrary to the Canadian Statute. The exception of losses occasioned by unseaworthiness was in effect a warranty that a loss should not be so occasioned, and whether the fact of unseaworthiness were known or unknown would be immaterial. Since the law provides for an implied warranty of seaworthiness in every contract of ordinary marine insurance, it becomes the obligation of a cargo owner to look for a reliable common carrier which keeps its vessels in seaworthy condition. The shipper of cargo may have no control over the vessel but he has full control in the choice of the common carrier that will transport his goods. Or the cargo owner may enter into a contract of insurance which specifically provides that the insurer answers not only for the perils of the sea but also provides for coverage of perils of the ship. We are constrained to apply Section 113 of the Insurance Code to the facts of this case. As stated by the private respondents: In marine cases, the risks insured against are "perils of the sea" (Chute v. North River Ins. Co., Minn214 NW 472, 55 ALR 933). The purpose of such insurance is protection against contingencies and against possible damages and such a policy does not cover a loss or injury which must inevitably take place in the ordinary course of things. There is no doubt that the term 'perils of the sea' extends only to losses caused by sea damage, or by the violence of the elements, and does not embrace all losses happening at sea. They insure against losses from extraordinary occurrences only, such as stress of weather, winds and waves, lightning, tempests, rocks and the like. These are understood to be the "perils of the sea" referred in the policy, and not those ordinary perils which every vessel must encounter. "Perils of the sea" has been said to include only such losses as are of extraordinary nature, or arise from some overwhelming power, which cannot be guarded against by the ordinary exertion of human skill and prudence. Damage done to a vessel by perils of the sea includes every species of damages done to a vessel at sea, as distinguished from the ordinary wear and tear of the voyage, and distinct from injuries suffered by the vessel in consequence of her not being seaworthy at the outset of her voyage (as in this case). It is also the general rule that everything which happens thru the inherent vice of the

The liability of the insurance company is governed by law. Section 113 of the Insurance Code provides:

In every marine insurance upon a ship or freight, or freightage, or upon any thing which is the subject of marine insurance, a warranty is implied that the ship is seaworthy. Section 99 of the same Code also provides in part. Marine insurance includes: (1) (a) Insurance against loss of or damage to: Vessels, craft, aircraft, vehicles, goods, freights, cargoes, merchandise, ...

From the above-quoted provisions, there can be no mistaking the fact that the term "cargo" can be the subject of marine insurance and that once it is so made, the implied warranty of seaworthiness immediately attaches to whoever is insuring the cargo whether he be the shipowner or not. As we have ruled in the case of Go Tiaoco y Hermanos v. Union Insurance Society of Canton (40 Phil. 40): The same conclusion must be reached if the question be discussed with reference to the seaworthiness of the ship. It is universally accepted that in every contract of insurance upon anything which is the subject of marine insurance, a warranty is implied that the ship shall be seaworthy at the time of the inception of the voyage. This rule is accepted in our own Insurance Law (Act No. 2427, sec. 106). ...

thing, or by the act of the owners, master or shipper, shall not be reputed a peril, if not otherwise borne in the policy. (14 RCL on Insurance, Sec. 384, pp. 1203- 1204; Cia. de Navegacion v. Firemen's Fund Ins. Co., 277 US 66, 72 L. ed. 787, 48 S. Ct. 459). With regard to the second assignment of error, petitioners maintain, that the loss of the cargo was caused by the perils of the sea, not by the perils of the ship because as found by the trial court, the barge was turned loose from the tugboat east of Cabuli Point "where it was buffeted by storm and waves." Moreover, petitioners also maintain that barratry, against which the cargo was also insured, existed when the personnel of the tugboat and the barge committed a mistake by turning loose the barge from the tugboat east of Cabuli Point. The trial court also found that the stranding and foundering of Mable 10 was due to improper loading of the logs as well as to a leak in the barge which constituted negligence. On the contention of the petitioners that the trial court found that the loss was occasioned by the perils of the sea characterized by the "storm and waves" which buffeted the vessel, the records show that the court ruled otherwise. It stated: xxx xxx xxx

It is quite unmistakable that the loss of the cargo was due to the perils of the ship rather than the perils of the sea. The facts clearly negate the petitioners' claim under the insurance policy. In the case of Go Tiaoco y Hermanos v. Union Ins. Society of Canton, supra, we had occasion to elaborate on the term "perils of the ship." We ruled: It must be considered to be settled, furthermore, that a loss which, in the ordinary course of events, results from the natural and inevitable action of the sea, from the ordinary wear and tear of the ship, or from the negligent failure of the ship's owner to provide the vessel with proper equipment to convey the cargo under ordinary conditions, is not a peril of the sea. Such a loss is rather due to what has been aptly called the "peril of the ship." The insurer undertakes to insure against perils of the sea and similar perils, not against perils of the ship. As was well said by Lord Herschell in Wilson, Sons & Co. v. Owners of Cargo per the Xantho ([1887], 12 A. C., 503, 509), there must, in order to make the insurer liable, be some casualty, something which could not be foreseen as one of the necessary incidents of the adventure. The purpose of the policy is to secure an indemnity against accidents which may happen, not against events which must happen. In the present case the entrance of the sea water into the ship's hold through the defective pipe already described was not due to any accident which happened during the voyage, but to the failure of the ship's owner properly to repair a defect of the existence of which he was apprised. The loss was therefore more analogous to that which directly results from simple unseaworthiness than to that which result from the perils of the sea. xxx xxx xx

... The other affirmative defense of defendant Lighterage, 'That the supposed loss of the logs was occasioned by force majeure... "was not supported by the evidence. At the time Mable 10 sank, there was no typhoon but ordinary strong wind and waves, a condition which is natural and normal in the open sea. The evidence shows that the sinking of Mable 10 was due to improper loading of the logs on one side so that the barge was tilting on one side and for that it did not navigate on even keel; that it was no longer seaworthy that was why it developed leak; that the personnel of the tugboat and the barge committed a mistake when it turned loose the barge from the tugboat east of Cabuli point where it was buffeted by storm and waves, while the tugboat proceeded to west of Cabuli point where it was protected by the mountain side from the storm and waves coming from the east direction. ..." In fact, in the petitioners' complaint, it is alleged that "the barge Mable 10 of defendant carrier developed a leak which allowed water to come in and that one of the hatches of said barge was negligently left open by the person in charge thereof causing more water to come in and that "the loss of said plaintiffs' cargo was due to the fault, negligence, and/or lack of skill of defendant carrier and/or defendant carrier's representatives on barge Mable 10."

Suffice it to say that upon the authority of those cases there is no room to doubt the liability of the shipowner for such a loss as occurred in this case. By parity of reasoning the insurer is not liable; for generally speaking, the shipowner excepts the perils of the sea from his engagement under the bill of lading, while this is the very perils against which the insurer intends to give protection. As applied to the present case it results that the owners of the damaged rice must look to the shipowner for redress and not to the insurer. Neither can petitioners allege barratry on the basis of the findings showing negligence on the part of the vessel's crew.

Barratry as defined in American Insurance Law is "any willful misconduct on the part of master or crew in pursuance of some unlawful or fraudulent purpose without the consent of the owners, and to the prejudice of the owner's interest." (Sec. 171, U.S. Insurance Law, quoted in Vance, Handbook on Law of Insurance, 1951, p. 929.) Barratry necessarily requires a willful and intentional act in its commission. No honest error of judgment or mere negligence, unless criminally gross, can be barratry. (See Vance on Law of Insurance, p. 929 and cases cited therein.) In the case at bar, there is no finding that the loss was occasioned by the willful or fraudulent acts of the vessel's crew. There was only simple negligence or lack of skill. Hence, the second assignment of error must likewise be dismissed. Anent the third assignment of error, we agree with the petitioners that the amount of P8,000.00 representing the amount of the salvaged logs should have been awarded to them. However, this should be deducted from the amounts which have been adjudicated against Manila Bay Lighterage Corporation by the trial court. WHEREFORE, the decision appealed from is AFFIRMED with the modification that the amount of P8,000.00 representing the value of the salvaged logs which was ordered to be deposited in the Manila Banking Corporation in the name of Civil Case No. 86599 is hereby awarded and ordered paid to the petitioners. The liability adjudged against Manila Bay Lighterage Corporation in the decision of the trial court is accordingly reduced by the same amount. SO ORDERED.

mariners, and of all other perils, losses, andmisfortunes that have or shall come to the hurt, detriment, or damage of the said goods andmerchandise or any part thereof." It was found out that the drain pipe which served as adischarge from the water closet passed down through the compartment where the rice inquestion was stowed and thence out to sea through the wall of the compartment, which wasa part of the wall of the ship. The joint or elbow where the pipe changed its direction was ofcast iron; and in course of time it had become corroded and abraded until a longitudinalopening had appeared in the pipe about one inch in length. This hole had been in existencebefore the voyage was begun, and an attempt had been made to repair it by filling withcement and bolting over it a strip of iron. The effect of loading the boat was to submerge thevent, or orifice, of the pipe until it was about 18 inches or 2 feet below the level of the sea. Asa consequence the sea water rose in the pipe. Navigation under these conditions resulted inthe washing out of the cement-filling from the action of the sea water, thus permitting thecontinued flow of the salt water into the compartment of rice. An action on a policy of marineinsurance issued by the Union Insurance Society of Canton, Ltd., upon the cargo of ricebelonging to the Go Tiaoco Brothers was filed. The trial court found that the inflow of the seawater during the voyage was due to a defect in one of the drain pipes of the ship andconcluded that the loss was not covered by the policy of insurance. Judgment wasaccordingly entered in favor of Union Insurance and Go Tiaoco Brothers appealed. Issue 1: Whether perils of the sea includes entrance of water into the ships hold through adefective pipe. Held 1: NO. It is determined that the words "all other perils, losses, and misfortunes" are tobe interpreted as covering risks which are of like kind (ejusdem generis) with the particularrisks which are enumerated in the preceding part of the same clause of the contract.According to the ordinary rules of construction these words must be interpreted withreference to the words which immediately precede them. They were no doubt inserted inorder to prevent disputes founded on nice distinctions. Their office is to cover in termswhatever may be within the spirit of the cases previously enumerated, and so they have agreater or less effect as a narrower or broader view is taken of those cases. For example, ifthe expression "perils of the seas" is given its widest sense the general words have little or noeffect as applied to that case. If on the other hand that expression is to receive a limitedconstruction and loss by perils of the seas is to be confined to loss ex marine tempestatisdiscrimine, the general words

La Razon Social "Go Tiaoco y Hermanos" vs. Union Insurance Society of Canton Ltd.[GR 13983, 1 September 1919] Facts: A cargo of rice belonging to the Go Tiaoco Brothers, was transported in the early daysof May, 1915, on the steamship Hondagua from the port of Saigon to Cebu. On dischargingthe rice from one of the compartments in the after hold, upon arrival at Cebu, it wasdiscovered that 1,473 sacks had been damaged by sea water. The loss so resulting to theowners of rice, after proper deduction had been made for the portion saved, was P3,875. Thepolicy of insurance, covering the shipment, was signed upon a form long in use amongcompanies engaged in maritime insurance. It purports to insure the cargo from the followingamong other risks: "Perils . . . of the seas, men, of war, fire, enemies, pirates, rovers, thieves,.jettisons, . . . barratry of the master and

become most important. But still, when they first became thesubject of judicial construction, they have always been held or assumed to be restricted tocases "akin to" or "resembling" or "of the same kind as" those specially mentioned. I see noreason for departing from this settled rule. In marine insurance it is above all thingsnecessary to abide by settled rules and to avoid anything like novel refinements or a newdeparture. It must be considered to be settled, furthermore, that a loss which, in the ordinarycourse of events, results from the natural and inevitable action of the sea, from the ordinarywear and tear of the ship, or from the negligent failure of the ship's owner to provide thevessel with proper equipment to convey the cargo under ordinary conditions, is not a peril ofthe sea. Such a loss is rather due to what has been aptly called the "peril of the ship." Theinsurer undertakes to insure against perils of the sea and similar perils, not against perils ofthe ship. There must, in order to make the insurer liable, be "some casualty, something whichcould not be foreseen as one of the necessary incidents of the adventure. The purpose of thepolicy is to secure an indemnity against accidents which may happen, not against eventswhich must happen." Herein, the entrance of the sea water into the ship's hold through thedefective pipe already described was not due to any accident which happened during thevoyage, but to the failure of the ship's owner properly to repair a defect of the existence ofwhich he was apprised. The loss was therefore more analogous to that which directly resultsfrom simple unseaworthiness than to that which results from perils of the sea. Issue 2: Whether there is an implied warranty on the seaworthy of the vessel in every marineinsurance contract. Held 2: YES. It is universally accepted that in every contract of insurance upon anythingwhich is the subject of marine insurance, a warranty is implied that the ship shall beseaworthy at the time of the inception of the voyage. This rule is accepted in our ownInsurance Law (Act No. 2427, sec. 106). It is also well settled that a ship which is seaworthyfor the purpose of insurance upon the ship may yet be unseaworthy for the purpose ofinsurance upon the cargo (Act No. 2427, sec. 106)

CATHAY INSURANCE CO. vs. CA

G.R. No. 76145 June 30, 1987

This petition seeks the review of the decision of the Court of Appeals 1 in CA-G.R. CV No. 06559 affirming the decision of the Regional Trial Court (RTC), 2 National Capital Region (NCR) Manila, Branch 38 and the Resolution of the said appellate court denying petitioner's motion for reconsideration.

Originally, this was a complaint filed by private respondent corporation against petitioner (then defendant) company seeking collection of the sum of P868,339.15 representing private respondent's losses and damages incurred in a shipment of seamless steel pipes under an insurance contract in favor of the said private respondent as the insured, consignee or importer of aforesaid merchandise while in transit from Japan to the Philippines on board vessel SS "Eastern Mariner." The total value of the shipment was P2,894,463.83 at the prevailing rate of P7.95 to a dollar in June and July 1984, when the shipment was made.

The trial court decided in favor of private respondent corporation by ordering petitioner to pay it the sum of P866,339.15 as its recoverable insured loss equivalent to 30% of the value of the seamless steel pipes; ordering petitioner to pay private respondent interest on the aforecited amount at the rate of 34% or double the ceiling prescribed by the Monetary Board per annum from February 3, 1982 or 90 days from private respondent's submission of proof of loss to petitioner until paid as provided in the settlement of claim provision of the policy; and ordering petitioner to pay private respondent certain amounts for marine surveyor's fee, attorney's fees and costs of the suit. Respondent in its comment on the petition, contends that: 1. Coverage of private respondent's loss under the insurance policy issued by petitioner is unmistakable. 2. Alleged contractual limitations contained in insurance policies are regarded with extreme caution by courts and are to be strictly construed against the insurer; obscure phrases and exceptions should not be allowed to defeat the very purpose for which the policy was procured.

3. Rust is not an inherent vice of the seamless steel pipes without interference of external factors. 4. No matter how petitioner might want it otherwise, the 15-day clause of the policy had been foreclosed in the pre-trial order and it was not even raised in petitioner's answer to private respondent's complaint. 5. The decision was correct in not holding that the heavy rusting of the seamless steel pipes did not occur during the voyage of 7 days from July 1 to July 7, 1981. 6. The alleged lack of supposed bad order survey from the arrastre capitalized on by petitioner was more than clarified by no less than 2 witnesses. 7. The placing of notation "rusty" in the way bills is not only private respondent's right but a natural and spontaneous reaction of whoever received the seamless steel pipes in a rusty condition at private respondent's bodega. 8. The Court of Appeals did not engage in any guesswork or speculation in concluding a loss allowance of 30% in the amount of P868,339.15. 9. The rate of 34% per annum double the ceiling prescribed by the Monetary Board is the rate of interest fixed by the Insurance Policy itself and the Insurance Code. The petitioner however maintains that: (1) Private respondent does not dispute the fact that, contrary to the finding of the respondent Court (the petitioner has failed "to present any evidence of any viable exeption to the application of the policy") there is in fact an express exeption to the application of the policy. (2) As adverted to in the Petition for Review, private respondent has admitted that the question shipment in not covered bya " square provision of the contract," but private respondent claims implied coverage from the phrase " perils of the sea" mentioned in the opening sentenced of the policy. (3) The insistence of private respondent that rusting is a peril of the sea is erroneous. (4) Private respondent inaccurately invokes the rule of strict construction against insurer under the guise of construction in order to impart a non-existing ambiguity or doubt into the policy so as to resolve it against the insurer.

(5) Private respondent while impliedly admitting that a loss occasioned by an inherent defect or vice in the insured article is not within the terms of the policy, erroneously insists that rusting is not an inherent vice or in the nature of steel pipes. (6) Rusting is not a risk insured against, since a risk to be insured against should be a casualty or some casualty, something which could not be foreseen as one of the necessary incidents of adventure. (7) A fact capable of unquestionable demonstration or of public knowledge needs no evidence. This fact of unquestionable demonstration or of public knowledge is that heavy rusting of steel or iron pipes cannot occur within a period of a seven (7) day voyage. Besides, petitioner had introduced the clear cargo receipts or tally sheets indicating that there was no damage on the steel pipes during the voyage. (8) The evidence of private respondent betrays the fact that the account of P868,339.15 awarded by the respondent Court is founded on speculation, surmises or conjectures and the amount of less has not been proven by competent, satisfactory and clear evidence. We find no merit in this petition. There is no question that the rusting of steel pipes in the course of a voyage is a "peril of the sea" in view of the toll on the cargo of wind, water, and salt conditions. At any rate if the insurer cannot be held accountable therefor, We would fail to observe a cardinal rule in the interpretation of contracts, namely, that any ambiguity therein should be construed against the maker/issuer/drafter thereof, namely, the insurer. Besides the precise purpose of insuring cargo during a voyage would be rendered fruitless. Be it noted that any attack of the 15-day clause in the policy was foreclosed right in the pre-trial conference.

Finally, it is a cardinal rule that save for certain exceptions, findings of facts of the appellate tribunal are binding on Us. Not one of said exceptions can apply to this case.

WHEREFORE, this petition is hereby DENIED, and the assailed decision of the Court of Appeals is hereby AFFIRMED. SO ORDERED.

Go vs. Redfern, GR 47705, 25 April 1941 Note: Original decision in Spanish [Rough translation] Facts: In October 1937, Edward K. Redfern obtained an insurance policy against accidents from the International Assurance Co, Ltd. On 31 August 1938, Redfern died from an accident. The mother of the deceased, presenting the necessary evidence of the death of Redfern, sought to claim the proceeds of the insurance policy from the insurance company. The company, however, denied such claim, on the ground that the insurance policy was amended on 22 November 1937 to include another beneficiary, Concordia Go. Hence, an action was filed to determine who has the right to collect the insurance proceeds of the deceased Redfern. The mother claimed that the addition of the co-beneficiary is illegal. Go, on her part, alleged the contrary. The trial court ruled in favor of Angela Redfern, the mother. Go appealed.

Whether the addition of Gos name as co-beneficiary can be allowed for her share in the insurance proceeds

Held: When designated in a policy, the beneficiary acquires a right of which he cannot be deprived of without his consent, unless the right has been reserved specifically to the insured to modify the policy. The same doctrine was enunciated by the Court in the cases of Gercio vs. Sun Life Assurance Co. of Canada (48 Phil. 55) and Insular Life vs. Suva (34 Off. Gaz. 861). Thus, unless the insured has reserved specifically the right to change or to modify the policy, with respect to the beneficiary, said policy constitutes an acquired right of the beneficiary, which cannot be modified except with the consent of the latter. Herein, it is admitted that Redfern did not reserve expressly his right to change or modify the policy. Change implies the idea of an alteration. The addition of Go's name as one of the beneficiaries of the policy constitutes change as all addition is an alteration. The addition of Go's name changed the policy inasmuch as there are two beneficiaries instead of one, and thus in effect the original beneficiary cannot receive the full amount of the policy. The Supreme Court affirmed the appealed judgment in all of its parts, with costs against Go.
19. Tibay vs. Court of Appeals; 257 SCRA 126; 20. Phil. Phoenix Surety & Ins. Co. vs. Woodworks Inc.; 92 SCRA 419 21. Roque vs. IAC; 139 SCRA 596; 22. Go Tiaco vs. Union ins. Society of Canton; 40 Phil. 401;

Issue:

1. Great Pacific Life Assurance vs. Court of Appeals; 316 SCRA 678; 2. Sun Life Assurance Co. of Canada vs. Court of Appeals; June 22, 1995; 3. Philamcare Health Systems Inc. vs. Court of Appeals; 379 SCRA 356; 4. Vda. De Canilang vs. Court of Appeals; June 17, 1993; 5. Tan vs. Court of Appeals; June 29, 1989; 6. Prudential Guarantee vs. Trans-Asia Shipping Lines inc.; June 20, 2006 7. Bonifacio Bros. inc. vs. Mora; May 29, 1967; 8. The Insular Life Assurance vs. Ebrado; 80 SCRA 181; 9. Vda. De Consuegra vs. GSIS; 37 SCRA 315 10. Go vs. Redfern; 72 Phil. 71;

11. Country Bankers Ins. Corp. vs. Lianga Bay; January 25, 2002 12. Pacific Timber Export Corp. vs. Court of Appeals; 112 SCRA 199; 13. Great Pacific Life Insurance Corp. vs. Court of Appeals; 89 SCRA 543; 14. Philam Life and General Assurance Co. vs. Judge ValenciaBagalacsa; August 1, 2002; 15. Makati Tuscany condominium Corp. vs. Court of Appeals; 215 SCRA 462 16. UCPB General Insurance vs. Masagana Telamart; June 5, 1999 17. UCPB General Insurance vs. Masagana Telamart; 356 SCRA 307; 18. American Home Insurance vs. Chua; June 28, 1999;

23. Cathay Ins. Vs. Court of Appeals; 151 SCRA 710; 24. Filipino Merchants Ins. Co. vs. Court of Appeals 179 SCRA 638; 25. Oriental Assurance Corp. vs. Court of Appeals; 200 SCRA 459 26. Vda. De Gabriel vs. Court of Appeals; November 14, 1996 27. Vda. De Maglana vs. Hon. Consolacion; August 6, 1992 28. Tiu vs. Arriesgado; September 1, 2004

You might also like